10.07.2015 Views

n - Recreaţii Matematice

n - Recreaţii Matematice

n - Recreaţii Matematice

SHOW MORE
SHOW LESS

You also want an ePaper? Increase the reach of your titles

YUMPU automatically turns print PDFs into web optimized ePapers that Google loves.

Anul XI, Nr. 2Iulie – Decembrie 2009RECREAŢ IIMATEMATICEREVISTĂ DE MATEMATICĂ PENTRU ELEVI Ş I PROFESORI10 ani de la apariţiarevistei “Recreaţii <strong>Matematice</strong>”e i 1Asociaţ ia “Recreaţ ii <strong>Matematice</strong>”IAŞ I - 2009


Semnificaţia formulei de pe copertă:iÎntr-o formă concisă, formula e 1leagă cele patru ramuri fundamentaleale matematicii:ARITMETICA reprezentată de 1GEOMETRIAreprezentată de ALGEBRAreprezentată de iANALIZA MATEMATICĂreprezentată de eRedacţia revistei :Petru ASAFTEI, Dumitru BĂTINEŢU-GIURGIU (Bucureşti), Temistocle BÎRSAN, DanBRÂNZEI, Alexandru CĂRĂUŞU, Constantin CHIRILĂ, Eugenia COHAL, AdrianCORDUNEANU, Mihai CRĂCIUN (Paşcani), Paraschiva GALIA, Paul GEORGESCU,Mihai HAIVAS, Gheorghe IUREA, Lucian-Georges Lăduncă, Mircea LUPAN, GabrielMÎRŞANU, Alexandru NEGRESCU (student, Iaşi), Gabriel POPA, Dan POPESCU(Suceava), Florin POPOVICI (Braşov), Maria RACU, Neculai ROMAN (Mirceşti), IoanSĂCĂLEANU (Hârlău), Ioan ŞERDEAN (Orăştie), Dan TIBA (Bucureşti), Marian TETIVA(Bârlad), Lucian TUŢESCU (Craiova), Adrian ZANOSCHI, Titu ZVONARU (Comăneşti).Materialele vor fi trimise la una dintre adresele e-mail : t-birsan@yahoo.comprofgpopa@yahoo.co.ukvpgeo@go.comCOPYRIGHT © 2008, ASOCIAŢIA “RECREAŢII MATEMATICE”. Toate drepturile aparţin Asociaţiei“Recreaţii <strong>Matematice</strong>”. Reproducerea integrală sau parţială a textului sau a ilustraţiilor dinaceastă revistă este posibilă numai cu acordul prealabil scris al acesteia.TIPĂRITĂ LA SL&F IMPEX IAŞIBd. Carol I, nr. 3-5Tel. 0788 498933E-mail: simonaslf@yahoo.comISSN 1582 - 1765


Revista semestrială RECREAŢII MATEMATICE este editată deASOCIAŢIA “RECREAŢII MATEMATICE”. Apare la datele de 1 martie şi1 septembrie şi se adresează elevilor, profesorilor, studenţilor şi tuturor celorpasionaţi de matematica elementară.În atenţia tuturor colaboratorilorMaterialele trimise redacţiei spre publicare (note şi articole, chestiuni demetodică, probleme propuse etc.) trebuie prezentate îngrijit, clar şi concis; eletrebuie să prezinte interes pentru un cerc cât mai larg de cititori. Se recomandă catextele să nu depăşească patru pagini. Evident, ele trebuie să fie originale şi sănu fi apărut sau să fi fost trimise spre publicare altor reviste. Rugăm ca materialeletehnoredactate să fie însoţite de fişierele lor.Problemele destinate rubricilor: Probleme propuse şi Probleme pentrupregătirea concursurilor vor fi redactate pe foi separate cu enunţ şi demonstraţie/rezolvare(câte una pe fiecare foaie) şi vor fi însoţite de numele autorului, şcoalaşi localitatea unde lucrează/învaţă.Redacţia va decide asupra oportunităţii publicării materialelor primite.În atenţia elevilorNumele elevilor ce vor trimite redacţiei soluţii corecte la problemele dinrubricile de Probleme propuse şi Probleme pentru pregatirea concursurilorvor fi menţionate în Pagina rezolvitorilor. Se va ţine seama de regulile:1. Pot trimite soluţii la minimum cinci probleme propuse în numărulprezent şi cel anterior al revistei; pe o foaie va fi redactată soluţia unei singureprobleme.2. Elevii din clasele VI-XII au dreptul să trimită soluţii la problemelepropuse pentru clasa lor, pentru orice clasă mai mare, din două clase mai mici şiimediat anterioare. Elevii din clasa a V-a pot trimite soluţii la problemele propusepentru clasele a IV-a, a V-a şi orice clasă mai mare, iar elevii claselor I-IV pottrimite soluţii la problemele propuse pentru oricare din clasele primare şi orice clasămai mare. Orice elev poate trimite soluţii la problemele de concurs (tip G şi L).3. Vor fi menţionate următoarele date personale: numele şi prenumele,clasa, şcoala şi localitatea, precum şi de numele profesorului cu care învaţă.4. Plicul cu probleme rezolvate se va trimite prin poştă (sau va fi adusdirect) la adresa Redacţiei:Prof. dr. Temistocle BîrsanStr. Aurora, nr. 3, sc. D, ap. 6,700 474, IaşiJud. IAŞIE-mail: t_birsan@yahoo.com


CUPRINSCătre cititori dupǎ zece ani de apariţie a revistei .......................................................................... 85NECULAI GHEORGHIU (1930-2009) ............................................................................................ 87ARTICOLE ŞI NOTET. BÎRSAN, G. DOSPINESCU – Conjectura Beal pentru polinoame ..................................... 89C.-L. BEJAN – Teorema lui Brouwer - un caz particular elementar ........................................ 94D. MĂRGHIDANU – Exponentul de triangularitate al unui triunghi ...................................... 96D. POPESCU – Asupra unui şir de integrale Riemann ............................................................. 98A. VERNESCU – Dualitatea unor sume combinatoriale ........................................................ 101NOTA ELEVULUID. M. MOCANU – Asupra problemei C.O: 5004 din G.M. ................................................... 105CORESPONDENŢEA. REISNER – Matrices à coefficients dans un corps fini ..................................................... 107CHESTIUNI METODICEM. MIHEŢ – O metodǎ de rezolvare a problemelor ..............................................................111S. BOGA – Metoda identificǎrii.............................................................................................115CUM CONCEPEM ... CUM REZOLVĂMD. MIHALACHE, M. TETIVA– Bisectoarele exterioare nu sunt ca bisectoarele interioare..117MATEMATICA ÎN CLASELE PRIMARED.M. BĂTINEŢU-GIURGIU – Metoda falsei ipoteze - variante de utilizare.........................121ŞCOLI ŞI DASCĂLILiceul Teoretic "Garabet Ibrăileanu"........................................................................................123LIDIA COHAL (1930-2009) ...................................................................................................126CONCURSURI ŞI EXAMENEConcursul de matematică "Al. Myller", ed. a VII-a, 2009 .................................................. 127Concursul de matematică "Florica T. Câmpan", 2009 ......................................................... 130PROBLEME ŞI SOLUŢIISoluţiile problemelor propuse în nr. 2/2008 .......................................................................... 135Soluţiile problemelor pentru pregătirea concursurilor din nr. 2/2008 ................................ 152Probleme propuse ...................................................................................................................... 161Probleme pentru pregătirea concursurilor .............................................................................. 167Training problems for mathematical contests ....................................................................... 169Pagina rezolvitorilor ............................................................................................................... 172ISSN 1582 – 17657 lei


Către cititoridupă zece ani de apariţie a revisteiCascada cântă:Îmi revărs cu bucurie apele,dacă un strop din ele ajunge însetatuluiRabindranath TagoreÎn toamna anului 1999 apare la Iaşi primul număr al revistei Recreaţii <strong>Matematice</strong>.Continuându-şi apariţia în mod constant cu două numere anual, revista a atins zeceani de existenţă în primăvara acestui an. Revista este gândită de iniţiatorii ei să fieo continuatoare a vechii reviste Recreaţii Ştiinţifice (1883–1888), de la care a preluatşi numele, puţin modificat, noua revistă având un conţinut strict matematic.Revista Recreaţii Ştiinţifice este prima revistă ştiinţifică din ţară care s-a adresattineretului cu chestiuni din toate ramurile ştiinţei, dar cu un conţinut predominantmatematic; în cuvântul de început al celui de-al şaselea an de apariţie se spune, cumodestie, dar şi cu mândrie: Credem că noi am tras cea întăi brazdă [...] Brazda ′ imică şi îngustă, dar există!Este o mare îndrăzneală să-ţi propui a fi continuator al unei reviste care a făcutoperă de pionerat în cultura românească şi a avut o contribuţie importantă în educaţiamatematică a tineretului. Preluând scopurile generoase ale creatorilor RecreaţiilorŞtiinţifice, actuale şi astăzi, dorinţa noastră supremă este de a asigura noii reviste unnivel de calitate şi de utilitate la înălţimea celui avut de ”străbuna” sa.Acum, la împlinirea a zece ani de apariţie, Recreaţiile <strong>Matematice</strong> este o revistăconsiderată printre primele publicaţii de gen din întreaga ţară. Fondatorii revisteiRecreaţii <strong>Matematice</strong> sunt (în ordine alfabetică): T. Bîrsan, C. Calistru, Al.Cărăuşu, C. Cocea, A. Corduneanu, Gh. Iurea, care au format primul comitetde redacţie. În timp, li s-au alăturat alţi profesori şi elevi. De Alin Spumă şiMihai Gârtan, dispăruţi din rândurile noastre, ne vom aminti cu pietate, dar şi curecunoştinţă pentru aportul lor entuziast la apariţia revistei în perioada de începuta ei. Cu nr. 2/2001 intră în redacţie Gabriel Popa, care devine în scurt timp unuldintre principalii realizatori ai revistei.Revista este distribuită şi are colaboratori, elevi şi profesori în toată ţara: Bârlad,Braşov, Bucureşti, Craiova, Galaţi, Hârlău, Orăştie, Paşcani, Piteşti, Rm. Vâlcea,Satu Mare, Sf. Gheorghe (Tulcea), Suceava, Timişoara, Vaslui etc. Menţionăm câţivacolaboratori din ţară ataşaţi Recreaţiilor <strong>Matematice</strong>: D. Popescu, F. Popovici, I.Şerdean, M. Tetiva, D. Tiba, L. Tuţescu, T. Zvonaru ş.a.Structura revistei comportă trei părţi mari: I) evenimente din lumea matematicii(conferinţe, aniversări, comemorări etc.), II) partea ”teoretică” cu rubricile: articoleşi note, nota elevului, chestiuni metodice, istoria matematicii, corespondenţe ş.a.,III) partea aplicativă cu rubricile: concursuri şi examene, probleme şi soluţii, paginarezolvitorilor. Mai sunt presărate în cuprinsul numerelor de revistă amuzamentematematice, diverse informaţii, premii şi elevi premiaţi.Se găsesc prezentate în paginile revistei figuri de mari matematicieni: P. Fermat,N.H. Abel, H. Poincaré, L. Euler, A.N. Kolmogorov, W.R. Hamilton ş.a., ilustre numeale matematicii româneşti: Spiru Haret, Gh. Vrănceanu, Al. Myller, Gr. C. Moisil,85


M. Haimovici ş.a., probleme celebre ale matematicii din toate timpurile: postulatul Val lui Euclid, construcţii cu rigla şi compasul, problema celor patru culori, conjecturalui Poincaré ş.a., instituţii şi reviste care au contribuit la progresul învăţământuluişi cercetării matematice româneşti: Seminarul Matematic ”Al. Myller” din Iaşi, Observatorulastronomic din Iaşi, revista Recreaţii Ştiinţifice, Revista Ştiinţifică ”V.Adamachi” ş.a., evenimente importante: Congresele internaţionale ale matematicienilorromâni din 2003 şi 2007, cât şi reflectarea altor aspecte ale vieţii matematice.Revista Recreaţii Ştiinţifice, pe care încercăm să o continuăm, a fost sărbătorităîn 2003, la 120 de ani de la apariţia sa, prin publicarea unor materiale omagiale, darşi a unor articole selectate din numerele acestei vechi reviste. A fost sărbătorită maiamplu în 2008, la 125 de ani de la apariţie, în cadrul unui simpozion desfăşurat înaula filialei Iaşi a Academiei Române. O importantă acţiune a fost, reeditarea celorşase tomuri de Recreaţii Ştiinţifice prin redarea textului originar cu mijloace moderneşi prin producerea unei variante electronice a lui de către firma Kepler Systèmesd ′ Information (director Marinela Ghigea).În rubricile părţii ”teoretice” a revistei au fost publicate în aceşti 10 ani, un numărde 216 articole: 121 note matematice, 26 note ale elevilor, 34 chestiuni metodice, 10”corespondenţe” (articole primite din afara ţării), 9 articole adresate claselor primareşi 16 articole repartizate în alte rubrici.Datorită strădaniilor unor membri ai redacţiei revistei, au fost atraşi, ca rezolvitorisau colaboratori cu note matematice, elevi talentaţi, olimpici, care, fără îndoială, sevor afirma în scurt timp ca valoroşi cercetători în domeniul matematicii: GabrielDospinescu, Adrian Zahariuc, Marius Pachiţariu, Irina Mustaţă, Vlad Emanuel,Cezar Lupu, Iurie Boreico (Chişinău), Marius Tiba, recompensaţi cu premii în banişi diplome.La rubrica Corespondenţe articolele au apărut în limba în care au fost redactate(română, franceză şi engleză) şi provin de la corespondenţi din ţările: RepublicaMoldova, Germania, Kazahstan, Franţa şi China.Date statistice sumare dau o imagine asupra părţii de probleme şi soluţii (toateproblemele au primit soluţii după un an de la publicare): ciclul primar–173 probleme,gimnaziu–432, liceu–400, de tip G (avansate pentru gimnaziu)–165, de tip L (avansatepentru liceu)–165. La aceastea trebuie adăugate şi problemele date la diferite concursurinaţionale care sunt prezentate în revistă ediţie de ediţie: Concursul ”Al. Myller”,Concursul ”Florica T. Câmpan”, Concursul ”Recreaţii <strong>Matematice</strong>” ş.a. Pentru treiapariţii la rubrica Pagina rezolvitorilor, elevii harnici sunt premiaţi cu câte o diplomăşi cărţi; în acest interval de 10 ani au fost premiaţi 234 de elevi rezolvitori, cei maimulţi fiind din ciclul primar.Eforturile colectivului de redacţie au fost uşurate datorită colaborării unui numărmare de profesori din întreaga ţară, care prin materialele oferite au contribuit laridicarea nivelului revistei.Revista Recreaţii <strong>Matematice</strong> este recenzată în G.M.-A şi în revista de referateZentralblatt für Didaktik der Mathematik (ZDM). Pagina web a revistei poate fivizitată la adresa: http://www.recreatiimatematice.ro86Redacţia revistei


NECULAI GHEORGHIU(1930-2009)În luna ianuarie 2009, Profesorul nostru,Neculai Gheorghiu a plecat dintre noi pentrutotdeauna. Va sta de acum încolo la masa umbrelor.Scriu aceste rânduri, nu numai în calitate dedecan al Facultăţii de matematică din Iaşi, facultatepe care Profesorul Gheorghiu a slujit-o cudăruire şi pasiune o viaţă, ci mai ales în calitatede discipol al său.Am urmat două cursuri ţinute de domnia sa,cel de analiză matematică din semestrul I al anuluiI şi cel de analiză funcţională din anul al III-lea.În plus, Profesorul Gheorghiu a fost îndrumătorulştiinţific al lucrării mele de licenţă. Ca asistentuniversitar, am făcut mulţi ani la rând seminarulla cursul de analiză funcţională din anul al III-lea.Perioada studenţiei mele a reprezentat o etapăde vârf a carierei Profesorului. Astfel, în anul 1973 a apărut, în colaborare cu prof.Teodor Precupanu, cursul litografiat Analiză matematică, publicat în 1976 la Edituradidactică şi pedagogică, iar în 1974 a apărut, la Editura Academiei Române, carteaIntroducere în analiză funcţională. În aceeaşi perioadă, 1972-1976, a fost prodecan alFacultăţii de matematică a Universităţii ”Al.I. Cuza” Iaşi, punând în slujba acesteiacalităţile sale de bun organizator şi om cu gândire limpede şi de perspectivă. Anterior,fusese decanul Facultăţii de matematică a Institutului Pedagogic de 3 ani din Iaşi,unde a fost colaborator apropiat al prof. Ilie Popa, rectorul institutului, şi a militatneobosit pentru o calitate superioară a învăţământului. A făcut parte din comitetulde conducere al Seminarului matematic ”Al. Myller” din Iaşi.Profesorul Neculai Gheorghiu a fost înzestrat cu un talent didactic de excepţie.Lecţiile sale erau de o claritate, de o rigoare şi de o naturaleţe deosebite. Nu ampredat decât lucruri simple, obişnuia să spună Profesorul; în realitate, concepte cuun înalt grad de abstractizare şi teorii complexe deveneau simple şi transparentedatorită harului său pedagogic. Ele au contribuit decisiv la orientarea mea spredomeniul analizei matematice. Nu puţini au fost şi mai sunt aceia care, la rândullor, au afirmat şi afirmă cu recunoştinţă că, precumpănitor, şi-au modelat gândireamatematică tocmai datorită acestor lecţii.Avea şi un mod original de a se comporta cu studenţii şi cu cei din jur. Eradeschis, direct, sever şi uneori dur, dar aceste asperităţi ale firii sale erau completateşi compensate de bogăţia sufletului său şi omenia sa. Faţă de studenţi dovedea largidisponibilităţi de a-i asculta şi înţelege, dar şi de a-i ajuta, în măsura în care o87


putea face. Era întotdeauna corect şi cinstit. Toate acestea au făcut ca ProfesorulNeculai Gheorghiu să fie foarte mult apreciat şi iubit de multe generaţii de studenţi.Aproape de fiecare dată când eram undeva unde se găseau foşti studenţi ai săi, apăreaîntrebarea Ce mai face Profesorul Gheorghiu? sau, mai direct şi mai sugestiv, Ce maiface Nae?, ca şi cum Nae era un fost coleg sau un prieten comun.Avea nostalgia locului natal; vorbea cu afecţiune de Mileanca (fostul judeţ Dorohoi),satul în care s-a născut şi a urmat clasele primare şi nu uita să adauge cu mândriecă la Mileanca se află cel mai bun cernoziom. Îşi amintea cu plăcere de şcolile pe carele-a urmat şi de faptul că a fost propunător de probleme la Gazeta Matematică (toatede geometrie, ţinea să precizeze).O dovadă a posibilităţilor întinse de comunicare a Profesorului NeculaiGheorghiu cu semenii săi din domenii diverse de activitate este implicarea sa înviaţa sportivă a Iaşului, la un moment dat având funcţii de conducere în baschetul şifotbalul ieşean.S-a preocupat şi de învăţământul matematic preuniversitar. În repetate rânduri aonorat ca preşedinte comisia judeţeană a Olimpiadei naţionale de matematică, a ţinutprelegeri pentru elevi, a propus probleme la concursuri şi olimpiade şi este coautorla o frumoasă culegere de probleme pentru elevi intitulată Matematici elementare;probleme de sinteză (Editura Junimea, Iaşi, 1983). Evident, ca orice culegere deprobleme bine scrisă şi aceasta conţine rezolvări integrale şi multe comentarii utile.Există acolo o superbă problemă propusă de Profesorul Gheorghiu la etapa finală aOlimpiadei de matematică din 1963.Profesorul Neculai Gheorghiu avea o inteligenţă sclipitoare. A avut rezultatenotabile şi în activitatea de cercetare ştiinţifică, domeniul său predilect de preocupărifiind acela al ecuaţiilor diferenţiale ordinare. Teza sa de doctorat, sub conducerea prof.dr. doc. Ilie Popa, cu titlul Despre comportarea asimptotică a soluţiilor ecuaţiilordiferenţiale de ordinul doi a fost foarte bine apreciată. Rezultatele cu totul deosebitedin activitatea didactică împreună cu rezultatele din activitatea de cercetare au făcutca Profesorul Gheorghiu să avanseze rapid în cariera universitară, astfel că la 40 deani era deja profesor universitar.Profesorul Neculai Gheorghiu continuă să fie prezent printre noi şi după dispariţiasa fizică; este de găsit în rafturile de cărţi ale Seminarului Matematic ”Al.Myller”, dar şi păstrat cu recunoştinţă şi iubire în inimile numeroaselor promoţii destudenţi care l-au avut ca dascăl.Va rămâne mereu în amintirea noastră ca un profesor reprezentativ al Facultăţiide matematică a universităţii ieşene, cu o contribuţie importantă în progresul învăţământuluimatematic românesc!Prof. dr. Ovidiu CÂRJĂDecan al Facultăţii de MatematicăUniversitatea ”Al.I. Cuza” Iaşi88


Conjectura Beal pentru polinoameTemistocle BÎRSAN 1 , Gabriel DOSPINESCU 2Abstract. This paper deals with the solution of equation (1) in the set of polynomials withinteger coefficients and it has an informative aim. As well as in the case of Fermat ′ s Conjecture, thesolution to Beal ′ s Conjecture (i.e., the claim that Eq. (1) with p, q, r- integer numbers greater than2 has no solution with x, y, z positive and mutually prime) in the set C [X]is elementary (Proposition1). The triples (p, q, r) with p, q, r in N ∗ such that equation (1) has solutions in C [X] are: (1, q, r),(2, 2, r), (2, 3, 3), (2, 3, 4), (2, 3, 5).Keywords: polynomials, Beal ′ s Conjecture, Fermat ′ s Conjecture, regular polyhedra.MSC 2000: 11D41.În 1966, Andrew Beal instituie un premiu pentru demonstrarea sau infirmareaaşa-numitei acum Conjecturi Beal [1]:Ecuaţia(1) x p + y q = z r ,unde p, q, r sunt numere întregi mai mari ca 2, nu are nici o soluţie cu x, y, z întregipozitivi şi relativ primi.Pentru p = q = r (= n) ecuaţia devine(2) x n + y n = z n ,ecuaţie care a fost subiect de preocupări pentru lumea matematică încă din antichitateşi mai este şi acum, deşi a fost realizată rezolvarea ei completă. Şcoala luiPitagora a demonstrat că ecuaţia x 2 +y 2 = z 2 admite o infinitate de triplete (x, y, z)formate cu numere întregi, pozitive şi prime între ele ce o verifică. Pierre Fermat aafirmat că ecuaţia (2) pentru n ≥ 3 nu admite nici o soluţie cu numere x, y, z întregişi nenule şi a notat pe marginea unei pagini a Aritmeticii lui Diofant că posedă odemonstraţie minunată a acestui fapt. Generaţii de matematicieni, aflaţi pe urmeleacestei demonstraţii, n-au reuşit să o găsească sau să lămurească care ar fi pututsă fie aceasta, ceea ce a făcut pe unii să presupună că Fermat a greşit pe parcursuldemonstraţiei sale. În sfârşit, în 1995, Andrew Wiles demonstrează Marea teoremăa lui Fermat, aşa cum istoria matematicii reţine afirmaţia lui Pierre Fermat [6];demonstraţia lui Wiles este, însă, accesibilă unui cerc foarte restrâns de specialişti.În acest context, apare cu atât mai uimitor şi remarcabil faptul că rezolvareaecuaţiei (2) în mulţimea C [X] a polinoamelor cu coeficienţi complecşi este elementară,1 Prof. dr., Catedra de matematică, Univ. Tehnică ”Gh. Asachi”, Iaşi2 Student, École Normale Supérieure, Paris 89


accesibilă unui elev de liceu. Mai precis, cu ajutorul teoremei Mason - Stothers,enunţată mai jos, se poate dovedi afirmaţia următoare:Dacă n ≥ 3, atunci ecuaţia (2) nu are soluţii în C [X] cu polinoame neconstanteşi relativ prime.O prezentare a acestor fapte este făcută în [4]; cititorii revistei sunt informaţiasupra acestui subiect în [2].În această ordine de idei, se impune de la sine înlocuirea ecuaţiei (2) cu (1) şirezolvarea acesteia din urmă în mulţimea C [X]. Este tocmai ceea ce ne propunem.Lăsăm, însă, cititorilor plăcerea de a rezolva Conjectura lui Beal propriu-zisă (în Z)(informaţii asupra premiului oferit de A. Beal sunt date în [1]).Vom apela şi în acest caz laTeorema Mason - Stothers ([4], [2]). Fie f, g, h ∈ C [X] neconstante şi relativprime. Dacă are loc egalitatea f + g = h, atunci(3) max {deg f, deg g, deg h} ≤ n 0 (fgh) − 1kQ(pentru f ∈ C [X] cu descompunerea f (X) = α (X − a i ) m i, se notează deg f =i=1m 1 + m 2 + · · · + m k – gradul lui f şi n 0 (f) = k – numărul rădăcinilor sale distincte).Demonstraţia este elementară şi poate fi găsită în [2].Propoziţia 1 (Conjectura Beal pentru polinoame). Dacă p, q, r ∈ Z şip ≥ 3, q ≥ 3, r ≥ 3, atunci ecuaţia (1) nu are soluţii în C [X] cu polinoame neconstanteşi relativ prime.Demonstraţie. Procedăm ca şi în cazul ecuaţiei (2). Presupunem că ecuaţia(1) cu exponenţii p, q, r satisfăcând condiţiile din enunţ admite o soluţie (f, g, h) cupolinoame neconstante relativ prime. Ca urmare, putem aplica Teorema Mason –Stothers polinoamelor (f (X)) p , (g (X)) q , (h (X)) r şi scriede undeşi, deci,deg (f (X)) p ≤ n 0 (f (X)) p (g (X)) q (h (X)) r − 1,p deg f (X) ≤ n 0 (f (X) g (X) h (X)) − 1p deg f (X) ≤ n 0 (f (X)) + n 0 (g (X)) + n 0 (h (X)) − 1.Scriind şi inegalităţile analoage acesteia relativ la polinoamele g şi h şi adunându-lemembru cu membru, obţinem relaţia(p − 3) deg f (X) + (q − 3) deg g (X) + (r − 3) deg h (X) ≤ −3,care este falsă, căci p ≥ 3, q ≥ 3, r ≥ 3, şi demonstraţia este încheiată.Înainte de a vedea ce se întâmplă atunci când unul (cel puţin) dintre exponenţiiecuaţiei (1) este mai mic ca 3, vom face câteva90


Observaţii. 1) În prezenţa egalităţii f + g = h, faptul că polinoamele f, g, h suntprime între ele este echivalent cu condiţia ca f, g, h să fie relativ prime două câtedouă.2) Dacă ε p 1 = 1, εq 2 = 1 şi εr 3 = 1 şi (x, y, z) este o soluţie în C [X] a ecuaţiei(1), atunci sunt soluţii ale acestei ecuaţii şi tripletele: (ε 1 x, ε 2 y, ε 3 z), (x, ε 2 y, ε 3 z),(x, y, ε 3 z) etc. Vom face abstracţie de aceste soluţii derivate ale unei soluţii găsite.3) Fără a restrânge generalitatea, putem considera că în tripleta (p, q, r) a exponenţilorecuaţiei (1) avem(4) p ≤ q ≤ r.Într-adevăr, putem lua p ≤ q schimbând, eventual, x şi y între ele în ecuaţia (1).Dacă r ≤ p, scriem (1) sub forma z r + (ε 1 x) p = y q , unde ε p 1 = −1. Dacă p ≤ r ≤ q,punem (1) sub forma x p + (ε 2 z) r = (ε 3 y) q , unde ε r 2 = ε q 3 = −1.Conform Propoziţiei 1 şi ţinând seama de (4), rezultă că p ∈ {1, 2}.I Cazul (1,q,r) este banal: soluţia generală a ecuaţiei(5) x + y q = z reste dată de(6) x = h r − g q , y = g, z = h, ∀g, h ∈ C [X] ,pentru orice exponenţi q, r ∈ N ∗ .II Cazul (2,q,r). Cu teorema Mason - Stothers vom obţine limitări importanteîn privinţa exponenţilor q şi r. Fie (x, y, z) o soluţie în C [X] a ecuaţiei x 2 + y q = z r(cu x, y, z neconstante şi relativ prime) şi fie a = deg x, b = deg y, c = deg z. Teoremaamintită ne spune cămax {2a, qb, rc} ≤ n 0€x 2 y q z rŠ−1,de undedeci avemmax {2a, qb, rc} ≤ a + b + c − 1,(7) a < b + c − 1 şi max {qb, rc} ≤ 2 (b + c − 1) .Cum max {qb, rc} ≥ 1 2 (qb + rc) ≥ q (b + c), combinând cu a doua relaţie din (7) vom2obţine q (b + c) ≤ 2 (b + c − 1), ceea ce conduce la q ∈ {2, 3}.2II.1 Subcazul (2,2,r), cu r ≥ 2. Ecuaţia(8) x 2 + y 2 = z rse mai scrie(x + iy) (x − iy) = z r91


şi cum x+iy şi x−iy sunt polinoame prime între ele rezultă că fiecare este puterea deexponent r a unui polinom: x + iy = f r şi x − iy = g r . În final, ecuaţia x2 + y 2 = z rare soluţia dată de(9) x = 1 2 (f r + g r ) , y = 1 2i (f r − g r ) , z = fg, ∀f, g ∈ C [X] .Verificarea faptului că (9) este o soluţie a ecuaţiei (8) este imediată.II.2 Subcazul (2,3,r), cu r ≥ 3. Din (7), avem max {3b, rc} ≤ 2 (b + c − 1), dincare rezultă căb < 2 (c − 1) şi rc < 6 (c − 1) ,deci r ∈ {3, 4, 5}.(2, 3, 5).Aşadar, au rămas trei situaţii de analizat: (2, 3, 3), (2, 3, 4) şiII.2.1 Ecuaţia x 2 + y 3 = z 3 . Vom vedea că rezolvarea acestei ecuaţii se reducela un caz anterior studiat. Într-adevăr, scriem ecuaţia în forma(10) (z − y) (z − εy)€z − ε 2 yŠ=x 2 ,unde ε 2 + ε + 1 = 0. Întrucât z − y, z − εy şi z − ε2 y sunt relativ prime două câtedouă, din egalitatea precedentă deducem că fiecare dintre acestea este pătratul unuipolinom:z − y = f 2 , z − εy = g 2 , z − ε 2 y = h 2 .Pentru ca acest sistem liniar în z şi y să fie compatibil, impunem polinoamelor f, g,h condiţia−εf 2 + (1 + ε) g 2 = h 2 .Aceasta, însă, se reduce după substituţii evidente la ecuaţia x 2 + y 2 = z 2 , care serezolvă conform cazului II.1.II.2.2 Ecuaţia x 2 + y 3 = z 4 . Procedăm ca în cazul precedent. Scriem ecuaţia îndiscuţie sub forma(11)€z 2 − xŠ€z 2 + xŠ=y 3 .Cum z 2 − x şi z 2 + x sunt relativ prime, urmează că ele sunt cuburi de polinoame:z 2 − x = f 3 , z 2 + x = g 3 .Astfel, suntem conduşi la ecuaţia 2z 2 = f 3 + g 3 care se reduce la rândul ei la ecuaţiax 2 + y 3 = z 3 întâlnită în subcazul II.2.1. Practic, soluţiile ecuaţiei x 2 + y 3 = z 4 vorfi obţinute parametric şi se vor exprima în funcţie de soluţiile cazului (2, 2, r).II.2.3 Ecuaţia x 2 + y 3 = z 5 . Rezolvarea acestei ecuaţii pare să fie deosebit dedificilă. În lipsa unei descompuneri de tipul (10) sau (11), nu putem proceda ca maisus.92


Stabilim doar faptul că ecuaţia are soluţii. Se poate verifica direct că tripleta(x, y, z) dată deXŠx = X 30 + 522X 25 − 10005X 20 − 10005X 10 − 522X 5 + 1,(12) y = −X 20 + 228X 15 − 494X 10 − 228X 5 − 1,z = 5√ 1728€X 11 + 11X 6 −este o soluţie a acestei ecuaţii ([3] , [5]).Observaţie. În [3], Felix Klein pune în evidenţă legătura strânsă care existăîntre poliedrele regulate şi soluţiile în C(X) ale ecuaţiei (1). Astfel, cazul (2, 3, 3)este legat de tetraedrul regulat, cazul (2, 3, 4) de cub şi octogonul regulat, iar cazul(2, 3, 5), cu soluţia (12), de dodecaedrul şi de icosaedrul regulat.Această legătură este menţionată şi discutată în [5].În concluzie, tripletele de exponenţi (p, q, r), p, q, r ∈ N ∗ , pentru care ecuaţia (1)are soluţii în C [X] sunt: (1, q, r), (2, 2, r), (2, 3, 3), (2, 3, 4), (2, 3, 5) şi toate permutărileacestora.Bibliografie1. *** - Beals Conjecture, The New Zeland Math. Mag., 35(1998), no.2, 38.2. T. Bîrsan - Marea teoremă a lui Fermat pentru polinoame, RecMat - 1/2004, 5-9.3. F. Klein - Vorlesungen über das Ikosaeder und die Auflösung der Gleichungen vomfünften Graden, Teubner, Leipsig, 1884.4. S. Lang - Math Talks for Undergraduates, Springer, 1999.5. V. V. Prasolov - Essays on Numbers and Figures, Amer. Math. Soc., MathematicalWorld, v. 16, 2000.6. A. Wiles - Modular elliptic curves and Fermat ′ s Last Theorem, Annals of Math.,142(1995), 443-551.În 1962 Bachet de Méziriac publică o versiune latină a Aritmeticii ce includeapeste o sută de probleme şi avea margini largi ale textului. Pe paginile unui astfelde exemplar, Pierre Fermat îşi nota soluţiile, comentariile şi rezultatele proprii. Pemarginea Cărţii a II-a Fermat notă afirmaţia :ecuaţia x n + y n = z n nu are soluţii în numere întregi şi nenule pentru n ≥ 3.cunoscută acum ca Marea Teoremă a lui Fermat. Apoi a scris comentariul:(continuare la pagina 20)93


Teorema lui Brouwer - un caz particular elementarCornelia-Livia BEJAN 1Abstract. By following the ideas developed by T. Traynor in [2], an elementary proof ofBrouwer ′ s fixed point theorem is presented for the restricted case of continuously differentiablefunctions.Keywords: closed ball, ball, fixed point, continuously differentiable function.MSC 2000: 54H25.Teorema de punct fix a lui Brouwer are o istorie lungă. Ideile ce conduc lademonstraţia acesteia se găsesc la Henri Poincaré înainte de 1886. L.E.J. Brouwera demonstrat teorema pentru n = 3 în 1909. În 1910 J. Hadamard dă primademonstraţie pentru n arbitrar, iar Brouwer dă o alta în 1912.În [2], autorul a dat o prezentare elementară a teoremei de punct fix a lui Brouwer.În această notă, adaptând demonstraţia elementară dată în [2], ne propunem săprezentăm un caz particular al teoremei lui Brouwer, într-o formă şi mai accesibilăprofesorilor şi studenţilor.Peste tot vom nota cu D discul unitate din plan şi cu C frontiera sa, adică cercul.Teorema de punct fix a lui Brouwer afirmă că orice aplicaţie continuă f : D → Dadmite măcar un punct fix, adică există măcar un element x ∈ D astfel încât f(x) = x.Altfel spus, dacă deformăm în mod continuu discul unitate în el însuşi, atunci existămăcar un punct care nu-şi schimbă poziţia.Vom prezenta un caz particular al acestei teoreme (ipoteze întărite) şi anume:Teoremă. Dacă f : D → D este o funcţie cu derivatele de ordinul întâi continue,atunci ea admite măcar un punct fix.Lemă. Nu există nici o aplicaţie f : D → C cu derivatele de ordinul întâi continuecare să lase fixe toate punctele cercului, adică f(x) = x, ∀x ∈ C.Demonstraţie. Presupunem că ar exista o astfel de aplicaţie f şi notăm g(x) =f(x) − x, ∀x ∈ D. Constatăm că g este o funcţie cu derivatele de ordin întâi continueşi, în consecinţă, majorate (în normă) de o constantă k. Din teorema valorii mediiavem ∥g(x) − g(y)∥ ≤ k∥x − y∥, ∀x, y ∈ D, adică distanţa între g(x) şi g(y) estemajorată de distanţa dintre punctele x şi y, multiplicată cu k.Definim aplicaţia f t (x) = x + t · g(x) = (1 − t)x + tf(x), ∀t ∈ [0, 1]. Se verificăfaptul că f t lasă fixe punctele cercului, ∀t ∈ [0, 1]. Arătăm că f t este injectivă pentru0 ≤ t < 1 k . Într-adevăr, dacă f t (x) = f t (y), atunci ∥x − y∥ = t∥g(x) − g(y)∥ ≤tk∥x − y∥, de unde obţinem x = y întrucât tk < 1.În plus, pentru fiecare t ∈ [0, 1] fixat, avem f t(x) ′ = I +tg ′ (x), unde I este aplicaţiaidentică a planului. Pentru 0 ≤ t < 1 k se vede că aplicaţia f t duce discul D în el însuşi,adică f t (D) = D. În continuare să scriem:π = Aria(D) = Aria(f t (D)) =ZDdetf ′ t(x)dx, 0 ≤ t < 1 k .1 Prof.dr., Catedra de matematică, Univ. Tehnică ”Gh. Asachi”, Iaşi94


Deci membrul drept al relaţiei precedente este constant în raport cu t. Mai mult,cantitatea de sub integrală fiind un polinom în t, rezultă că prin integrare se obţinetot un polinom în t. Din faptul că membrul drept este pe de o parte o constantăşi pe de altă parte un polinom în t, urmează că valoarea sa este o constantă pentruorice t ∈ [0, 1], nu numai pentru t ∈ [0, 1 k). Trecând la limită pentru t → 1 în relaţiaprecedentă şi ţinând cont de f(D) = C, găsim:π = Aria D = lim Aria(f t (D)) = Aria(f(D)) = Aria C = 0,t→1ceea ce este fals. Contradicţia ne arată că Lema este adevărată.Demonstraţia Teoremei. Presupunem că aplicaţia din ipoteza teoremei nu arepuncte fixe. Atunci din inegalitatea Cauchy-Buniakowski-Schwarz se obţinef(x) · x ≤ ∥x∥∥f(x)∥ ≤ 1, ∀x ∈ D.Egalitatea s-ar atinge când x şi f(x) ar fi liniar dependente. Cum membrul drept ≤ 1,dacă membrul stâng f(x)·x ar fi = 1, am avea egalitate, în care caz am avea f(x) = αxcu α ∈ R şi ∥x∥ = 1. Deci α = αx · x = 1, de unde f(x) = αx = x; deducem că f aravea puncte fixe, ceea ce contrazice presupunerea făcută. În concluzie, egalitatea nuse atinge şi deci avem x · f(x) < 1, ∀x ∈ D.Construim o aplicaţie h(x) = x −1 − x · x f(x), ∀x ∈ D. Această aplicaţie este1 − x · f(x)bine definită întrucât 1 − x · f(x) > 0. Evident, aplicaţia f are derivatele de ordinulîntâi continue, iar h(x) = x, ∀x ∈ C. Pe de altă parte, avem:Caz 1. Dacă (x · f(x))x = (x · x)f(x), atuncih(x) =x − (x · f(x))x − f(x) + (x · x)f(x)1 − x · f(x)= x − f(x)1 − x · f(x) ≠ 0,deoarece am presupus că f nu are puncte fixe.Caz 2. Dacă (x·f(x))x ≠ (x·x)f(x) rezultă că x şi f(x) nu sunt liniar dependente.După modul cum este definită aplicaţia h, deducem că h(x) ≠ 0 căci în caz contrar xşi f(x) ar fi liniar dependente.În concluzie, constatăm că h nu se anulează şi deci putem defini o aplicaţie H(x) =h(x), ∀x ∈ D. Se vede că H : D → C are derivatele de ordin întâi continue şi∥h(x)∥fixează punctele cercului, adică H(x) = x, ∀x ∈ C. În baza Lemei, însă, am ajuns lao absurditate, întrucât o astfel de aplicaţie H nu există. Demonstraţia este încheiată.Observaţie. O generalizare de la plan la spaţiul cu n dimensiuni se face urmândpas cu pas calea parcursă mai sus. În liceu, elevii întâlnesc teorema lui Brouwer încazul unidimensional: o funcţie f : [0, 1] → [0, 1] continuă are cel puţin un punct fix.Bibliografie1. L.E.J. Brouwer - Über Abbildungen von Mannigfaltigkeiten, Math. Ann., 71(1912),97-115.2. T. Traynor - An Easy Analitic Proof of Brouwer ′ s Fixed Point Theorem, Atti Sem.Mat. Fis. Univ. Modena, XLIV (1996), 479-483.95


Exponentul de triangularitate al unui triunghiDorin MĂRGHIDANU 1Abstract. The following problem is investigated: if a, b, c denote the lengths of the sides ofa triangle, it is required to determine the values of the real and positive exponents α such thatthe powers a α , b α , c α can still be the side lengths of a triangle.It is introduced the notion oftriangularity exponent t and it is proved that t = 2 for the right-angled triangles, t ∈(1, 2) for theobtuse-angled triangles and t > 2 for the acute-angled triangles (Proposition 2).Keywords: triangularity exponent.2000 MSC: 51M15.În această notă dăm un răspuns la următoarea întrebare:Dacă a, b, c sunt lungimile laturilor unui triunghi, pentru care numere α reale şipozitive puterile a α , b α , c a pot forma de asemenea un triunghi?În prima parte a notei prezentăm instrumentul algebric de lucru – o extindere ainegalităţii 11.19 din [1], p.99, iar în cea de-a doua introducem exponentul de triangularitateal unui triunghi, noţiune necesară rezolvării problemei propuse.1. La fel cum a fost demonstrată inegalitatea 11.19 din [1], putem stabili şiPropozţia 1. Dacă pentru numerele reale strict pozitive a, a 1 , a 2 , . . . , a n existăun număr real α > 0 astfel încât a α = a α 1 + a α 2 + . . . + a α n, atunci au loc:(1) 1) a β > a β 1 + aβ 2 + . . . + aβ n, ∀β ∈ R, β > α;(2) 2) a β < a β 1 + aβ 2 + . . . + aβ n, ∀β ∈ R, 0 < β < α.Demonstraţie. Din relaţia de condiţie rezultă că a > a 1 , a > a 2 , . . . , a > a n .1) β > α implică a β−α > a β−α1 , . . . , a β−α > a β−αn şi avema β= a β−α · a α = a β−α (a α 1 + . . . + a α n) = a β−α · a α 1 + . . . + a β−α · a a n> a β−α1 · a α 1 + . . . + a β−αn · a α n = a β 1 + . . . + aβ n.2) β < α implică a β−α < a β−α1 , . . . , a β−α < a β−αn şi, inversând semnul de inegalitateîn calculul de la punctul 1), obţinem (2).Observaţii. 1. Relaţia (1) rămâne valabilă şi în condiţia a α > a α 1 + a α 2 + . . . + a α n.2. Pentru n = 2, α = 2 şi β ∈ N, β > 2, punctul 1) revine la afirmaţia 11.19 [1].Corolar. Dacă a, b, c sunt lungimile laturilor unui triunghi şi este verificată condiţiaa α = b α +c α , atunci (a β , b β , c β ) formează un triunghi dacă şi numai dacă β < α.Vom indica două aplicaţii directe, în geometrie, ale rezultatelor precedente.Aplicaţia 1. Într-un paralelipiped dreptunghic de muchii a, b, c şi diagonală mareD, avem D p > a p + b p + c p , ∀p ∈ R, p > 2.Demonstraţie. Fie d diagonala feţei de laturi a, b. Conform punctului 1) alPropoziţiei 1 (cu n = 2, α = 2), aplicat de două în cazul particular al triunghiuluidreptunghic avem D p > d p + c p > a p + b p + c p .1 Prof. dr., Liceul Teoretic din Corabia, d.marghidanu@gmail.com96


Aplicaţia 2. Fie OABC un tetraedru tridreptunghic cu vârful O şi OA = a,OB = b, OC = c. Atunci AB p + BC p + CA p > 2(a p + b p + c p ), ∀p ∈ R, p > 2.Demonstraţie. Cu Propoziţia 1, aplicată la triunghiurile dreptunghice OAB,OBC şi OCA, obţinem relaţiile AB p > a p + b p , BC p > b p + c p şi CA p > c p + a p care,adunate, conduc la inegalitatea din enunţ.2. Vom introduce o noţiune, cu rol decisiv, prin următoareaDefiniţie. Numărul real pozitiv t se numeşte exponent de triangularitate al triunghuluiABC cu laturi de lungimi a, b, c, dacă tripleta (a s , b s , c s ) formează un triunghipentru orice s ∈ R ∗ +, s < t, şi nu formează un triunghi pentru s ≥ t.Rezolvarea problemei propuse este dată dePropoziţia 2. Sunt adevărate următoarele afirmaţii:1) t = 2, dacă triunghiul este dreptunghic;2) t ∈ (1, 2), dacă este obtuzunghic;3) t > 2, dacă este ascuţiunghic; în acest caz, t este număr real (finit), dacătriunghiul nu-i isoscel sau dacă este isoscel şi unghiul opus bazei sale este strict maimare de 60 ◦ şi t = +∞, dacă triunghiul este isoscel şi are unghiul opus bazei mai micsau egal ca 60 ◦ .Demonstraţie. Fie ABC cu a = max{a, b, c}. Considerăm funcţia continuăf(x) = a x − b x − c x , x > 0.1) Triunghiul fiind dreptunghic, avem a 2 = b 2 + c 2 . Conform Corolarului de maisus, deducem că t = 2.2) În acest caz, f(1) = a − b − c < 0 şi f(2) = a2 − b 2 − c 2 > 0 (triunghiul ABCfiind obtuzunghic). Rezultă că există o valoare t∈(1, 2) (unică, conform Propoziţiei1) astfel încât f(t) = 0, adică a t = b t + c t . Din nou apelând la Corolar, deducem căt este exponentul de triangularitate al triunghiului.3) Avem f(2) = a 2 − b 2 − c 2 < 0 (triunghi ascuţiunghic) şi f(∞) = lim f(x) =x→∞ba‹xlim ax•1 − −cdacă triunghiul nu-i isoscel sau dacă este isoscelx→∞ ax˜=+∞,şi baza sa este mai mare ca laturile sale (echivalent, măsura unghiului din vârf estestrict cuprinsă între 60 ◦ şi 90 ◦ ). Deci, există t ∈ (2, ∞) unic astfel încât a t = b t + c t .Conchidem că t astfel găsit este exponentul de triangularitate în subcazul considerat.În sfârşit, dacă triunghiul este isoscel cu baza mai mică sau cel mult egală culaturile sale (echivalent, unghiul din vârf are măsura mai mică sau cel mult 60 ◦ ),atunci se constată direct că puterile de exponent α ale laturilor acestuia formează untriunghi de acelaşi tip, oricare ar fi α ∈ R. Aşadar, în acest subcaz avem t = +∞.Consideraţiile precedente sugerează examinarea triunghiurilor care verifică condiţiica a 3 = b 3 + c 3 (sau √ a = √ b + √ c etc.), aşa cum s-a făcut în cazul triunghiurilordreptunghice (a 2 = b 2 + c 2 ) sau altor triunghuiuri speciale (triunghiuri mediane,triunghiuri cu laturi în progresie aritmetică etc.).Bibliografie1. O. Bottema, R.Z. Djordjević, R.R. Janić, D.S. Mitrinović, P.M. Vasić –Geometric inequalities, Wolters-Noordhoff, Groningen, 1969.97


Asupra unui şir de integrale RiemannDan POPESCU 1Abstract. This Note is an extension of a former paper–[3] in the reference list. The authorremarks that many problems proposed as topics to various contests are direct consequences of Propositions1 and 2 in the note.Keywords: continuous function, periodic function, Rimann integral.MSC 2000: 26A42.În cele ce urmează, sunt prezentate două condiţii suficiente de convergenţă a şiruluireal de integraleZbf (x) g (nx) ,adxn∈Nunde f şi g sunt funcţii integrabile Riemann care asigură corectitudinea definiriişirului.Un prim rezultat este prezentat în [3]:Propoziţia 1. Fie f : [0, T ] → R o funcţie integrabilă Riemann şi g : [0, ∞) → Ro funcţie cu perioada T >0, astfel încât restricţia g| [0,T ]este integrabilă Riemann.Atunci(1) limn→∞ZTTZTf (x) g (nx) dx = 1 dxZTf (x) g (x) dx.000O aplicaţie directă a acestui rezultat este următoarea:Fie f : R → (0, ∞) o funcţie continuă cu perioada 1. Atuncilim f (x) · f (nx) dxn→∞Z1=Z100f (x)2.(Cristinel Mortici, etapa judeţeană, 2003)În lucrarea [2], se propune tot o consecinţă directă a rezultatului (1):Fie funcţia f : R → R continuă şi periodică cu perioada T > 0. Să se demonstrezecă, pentru a, b ∈ R, a < b, avemlim f (nx) dx =x→∞Zbb − aZTf (x) dx.TaTot ca o consecinţă a rezultatului (1), este prezentată următoarea problemă:Fie funcţia continuă f : [0, 1] → R. Să se demonstreze că şirul (a n ) n≥1, definitde relaţia a n =Z1{nx} 2 f (x) dx, este convergent şi să i se afle limita, în funcţia f,0unde {x} = x − max {k ∈ Z; k ≤ x} ,∀x ∈ R.(Octavian Purcaru, Lista scurtă, O.N.M., 2003)1 Profesor, Colegiul Naţional ”Ştefan cel Mare”, Suceava098


Demonstraţie. Cum funcţia g : R → Rg (x) = {x} 2 este periodică cu perioadaprincipală 1, au loc:=Z1 dxZ1a n = lim f (x) g (nx) dx f (x) {x} 2 dx=3Z11 f (x) dx,n→∞Z10deoareceZ1{x} 2 dx =Z1x 2 dx, funcţiile de integrat fiind egale pe [0, 1) .000În lucrarea [1], apare următoarea problemă, semnată de Mihail Bencze:Să se calculeze limn→∞Z1Un al doilea rezultat util:0t 2 {nt} dt, unde {x}are semnificaţia de mai sus.Propoziţia 2. Fie funcţiile continue g : [0, ∞) → R şi f : [0, a] → R, unde a >0.Dacă lim g (x) = L ∈ R, atuncix→∞(2) limn→∞Za0g (nx) f (x) dx = LZaf (x) dx.000Demonstraţie.Într-adevăr, dacă h (x) = g (x) − L, ∀x ∈ [0, ∞), atunciZa0g (nx) f (x) dx =Za(h (nx) + L) f (x) dx =Zah (nx) f (x) dx + LZaf (x) dx.Notând nx = t, se obţine:000Za0h (nx) f (x) dx =nZna1 h (t) f0tn‹dt. Za0Cum ∃b > 0 astfel încât |f (x)| ≤ b, ∀x ∈ [0, a] ,h (nx) f (x) dx≤ nZnab |h (t)| dt = abDacă H este o primitivă pentru |h| pe intervalul [0, ∞) ,ceea ce asigură că lim0naZna0|h (t)| dt.0|h (t)| dt H (x) − H (0)lim= lim= lim |h (x)| = 0,x→∞Rxxx→∞ xx→∞x→∞Za0h (nx) f (x) dx = 0.Observaţie. Relaţia (2) se poate scrie şi(2 ′ ) limn→∞Za0g (nx) f (x) dx =1lima→∞aZa0dx‹Zag (x) f (x) dx,099


ceea ce justifică prezentarea celor două şiruri de integrale Riemann împreună .O aplicaţie a acestui rezultat:Fie funcţiile f : [0, ∞) → R şi g : [0, 1] → R, astfel ca limdemonstreze că1limn→∞nZn0x→∞f (x) gx= LZ1g (x) dx.ndxDemonstraţie. Cu substituţia x = nt, limita devineşi finalizarea este clară.0f (x) = L ∈ R. Să se(Laurenţiu Panaitopol, etapa judeţeană, 2003)lim f (nt) g (t) dt = LZ1g (x) dxn→∞Z1Propunem cititorului următoarele exerciţii:1. Să se demonstreze că2. Să se calculeze lim0lim en→∞Zπ−x cos nx dx =n→∞Z100021 − e −ππ.{nx} [x] dx, justificându-se întâi existenţa limitei, unde[x] = max {k ∈ Z|k ≤ x} şi {x} = x − [x] , ∀x ∈ R.3. Să se demonstreze că, pentru orice funcţie continuă f : [0, a] → R, are locrelaţialim en→∞Za−nx f (x) dx = 0.Bibliografie01. D.M. Bătineţu - Giurgiu ş.a. – Analiză matematică. Probleme pentru clasa aXII-a, Editura Matrix, 2004.2. R. Miculescu ş.a. – Probleme de calcul integral, Editura GIL, 2005.3. D. Popescu, F. Popovici – O generalizare a lemei lui Riemann, Recreaţii <strong>Matematice</strong>,Iaşi, 4(2002), nr.1, 12-13.100


Dualitatea unor sume combinatorialeAndrei VERNESCU 1Abstract. The main result consist in the combinatorial identity (6), where Ω n is the notationin (1). Some connections of this identity with other combinatorial identities of similar kind, that areknown or are expected to be established, are formulated.Keywords: combinatorial identity.MSC 2000: 05A19.1. În lucrarea [1], din numărul 2/2007 al acestei reviste, s-a aplicat o ideee foarte(2k − 1)!!elegantă pentru calculul unei sume care conţine expresiile , k = 1, 2, 3, . . . , n.(2k)!!În nota de faţă venim în continuarea lucrării citate, prezentând calculul bazatpe aceeaşi idee al altei sume, care ar putea fi considerată, din punctul de vedere almijloacelor folosite, ca fiind duala celei precedente.Pentru facilitarea calculelor care vor urma, vom utiliza notaţia(1) Ω n =(2n − 1)!!(2n)!!(n = 1, 2, 3, . . .).(Am introdus această notaţie în manuscrisul din 1987 al cărţii [11], am publicat-opentru prima dată în articolul [6], am folosit-o în toate ediţiile culegerii de probleme[7], ca şi într-o serie de lucrări: [8], [9], [11] şi [13].)Tot în scopul scrierii mai simple a formulelor şi calculelor care vor urma, estenecesar să definim Ω n şi pentru n = 0. Pentru aceasta, în cazul n ≥ 1, să amplificămfracţia din partea dreaptă a egalităţii (1) cu (2n)!! = 2 n n!. Obţinem Ω n =(2n)!(2 n n!) 2 .Ca această formulă să poată fi prelungită şi pentru n = 0, convenim să punem prindefiniţie(2) Ω 0 = 1.Cu această notaţie, identitatea care formează rezultatul principal al articoluluicitat [1] se scrie concentrat astfel:(3)(−1)nXk=0k CnΩ k k = Ω n .Ea poate fi considerată din două puncte de vedere: ca formulă de însumare combinatorială,precum şi ca identitate referitoare la expresiile Ω k .2. Obţinerea identităţii (3) se efectuează în [1] utilizând integralele:(4) I n =Zπ/2cos n x dx =Zπ/2sin n x dx,01 Conf. dr., Departamentul de Ştiinţe, Univ. ”Valahia”, Târgovişte0101


pentru care I 0 = π 2 , I 1 = 1 şi pe care, folosind notaţia (1) împreună cu definiţia (2),=8>< >(5) I nputem să le mai scriem sub forma::π2 Ω k, dacă n = 2k1 1· , dacă n = 2k + 1.Ω k 2k + 1(Găsirea, pe baza formulei de recurenţă I k = k − 1kI k−2, k ≥ 2, a expresiilor integraleiI n – scrise însă fără a folosi expresia Ω k – este reamintită în [1].)Astfel, cu utilizarea notaţiei menţionate pentru Ω n şi a simbolului de însumareP, putem sintetiza obţinerea rezultatului principal din [1] astfel:π2 Ω n = I 2n =Zπ/2cos 2n x dx =Zπ/200€1 − sin 2 xŠndx ==Zπ/20(−1)nXk=0k Cn k sin 2k x!dx =nXk=0(−1) k Cn‚Zπ/2k sin 2k x dxŒ=0=(−1)nXk=0k CnI k 2k = π 2(−1)nXk=0n CnΩ k k ,adicăπ2 Ω n = π 2(−1)nXk=0k CnΩ k k ,de unde (3).Putem aplica acum exact aceeaşi idee de calcul pornind de la integrala (4) de ordinimpar, I 2n+1 . Vom obţine:1 1·Ω n 2n + 1 = I 2n+1 =Zπ/2cos 2n+1 x dx =Zπ/2€1 − sin 2 t=sin xxŠncos x dx =00Z1t=sin x= − t0€1 2Šn dt =Z10(−1)nXk=0k Cnt k 2k!dt =adică(6)=nXk=0(−1) k C k nZ1nXk=00t 2k dt=nXk=0(−1) k Cnk2k + 1 = 1 1·Ω n 2n + 1 .C k n(−1) k2k + 1 ,Această identitate, care constituie rezultatul principal al lucrării noastre, nu semai referă la expresiile Ω k în interiorul sumei, ci conţine pe Ω n doar la rezultat, fiind102


(spre deosebire de (3), care admite două interpretări) doar o identitate combinatorială.Dar, din punct de vedere al procedeului de deducere folosit, ea constituie o identitateduală celei din [1], adică (3).kÎnsă, ca de multe ori în domeniul identităţilorncombinatoriale, identitatea găsitănu este nouă! Într-adevăr, în cartea lui H. W. Gould [2], la pag. 6, este prezentatăidentitatea(−1)nXk=0kn xx + k = 1x+n(x ∈ R)în care notaţia consacratăαpentru α ∈ R şi k ∈ Nk, ∗ , are semnificaţia:α α(α − 1)(α − 2) · . . . · (α − k + 1),k=1 · 2 · 3 · . . . · kcuα0def= 1, iar dacă α = n ∈ N şi k ≤ n se obţin combinările,nk=C k n.Introducând în identitatea lui Gould x = 1 , se obţine (6).23. Prin prisma identităţilor combinatoriale, egalitatea (6) ocupă poziţia a patradin următoarea succesiune de formule:S ndef=T ndef=nXk=0nXk=0Cnkk + 1 = 2n+1 − 1n + 1 ; S(a)defn =C k n2k + 1 = f(n);T (a)ndef=nXk=0nXk=0(−1) k Ck nk + 1 = 1n + 1 ;C k n(−1) k2k + 1 = 1 1·Ω n 2n + 1 ,unde a desemnează o sumă alternată, iar f(n) este o expresie neprecizată.Găsirea sumelor S n şi S (a) este binecunoscută (o cale fiind integrarea identităţilorn(1+t) n =nXk=0C k nt k , respectiv (1−t) n =(−1)nXk=0k Cnt k k , pe intervalul [0, 1]). Obţinereasumei T n(a) a format obiectul prezentei note, expus în secţiunea 2 şi astfel, se mai puneproblema calculului sumei T n . O altă problemă de studiu ar putea-o constitui calcululunei sume asemănătoare cu cea din (3), dar nealternată, anumenXk=0C k nΩ k .Bibliografie1. A. Corduneanu, Gh. Costovici – Un şir strâns legat de şirul lui Wallis, Recreaţii<strong>Matematice</strong>, Anul IX, Nr. 2 iulie-decembrie 2007, 95-96.2. H. W. Gould – Combinatorial identities, Morgantown Printing and Binding, Co.,1972.3. R. L. Graham, D. E. Knuth, O. Patashnik – Concrete Mathematics, AddisonWesley Longman, Reading MA, 1994.103


4. D. E. Knuth – The art of Computer Programming, Vol. 1, Addison Wesley Longman,Reading, MA 1977 (traducerea la Ed. Teora, Bucureşti).5. J. Riordan – Combinatorial Identities, J. Wiley & Sons, New York, 1968.6. L. Tóth, A. Vernescu – Dezvoltarea asimptotică a şirului lui Wallis, G. M.-A, 11(1990), Nr. 1, 26-29.7. A. Vernescu – Analiză matematică. Probleme. Calcul diferenţial, Editura Pantheon,Bucureşti, 1991.8. A. Vernescu – Ordinul de convergenţă al şirului lui Wallis, G. M.-A, 12 (1991), 7-8.9. A. Vernescu – Asupra unui tip de relaţie de recurenţă, Revista Matematică dinTimişoara (seria a IV-a), nr. 4/2003, 8-14.10. A. Vernescu – Şiruri de Numere Reale, Ed. Univ. Bucureşti, 2004.11. A. Vernescu – Numărul e şi matematica exponenţialei, Ed. Univ. Bucureşti, 2004.12. A. Vernescu – The natural proof of the inequalities of Wallis type, Libertas Mathematica,24 (2004), 183-190.13. A. Vernescu, C. Mortici – New results in discrete asymptotic analysis, GeneralMathematics, to appear.(continuare de la pagina 9)Mă aflu în posesia unei demonstraţii minunate a acestei afirmaţii,dar marginea paginii este prea strâmtă pentru a o cuprinde.Care avea să fie o provocare timp de peste 350 de ani pentru multe generaţii dematematicieni.Abia în 1995 Andrew Wiles a demonstrat afirmaţia, punând capăt provocării luiFermat. Drumul parcurs de lumea matematică până la acest final a fost presărat detentative de demonstraţie, eşecuri, descoperiri epocale, drame şi tragedii individuale,dezvăluiri senzaţionale în mass-media etc.N-au lipsit nici glumele pe marginea provocării lui Fermat:În staţia de metrou de pe Eighth Street, New York a apărut inscripţia:Sau catrenul:x n + y n = z n : nu există soluţie. Am descoperit o demonstraţie cuadevărat remarcabilă a acestui fapt dar nu pot s-o scriu fiindcă-mivine trenul.”Pe untul meu sunt multe litere scrise”,Supărat un client la o masă răcnise;”N-am avut loc, răspunse piccolo-ul Pierre,Nici pe margine, nici pe rafturile din frigider.”(Simon Singh–Marea Teoremă a lui Fermat, Humanitas, Bucureşti, 2005.)104


Asupra problemei C.O: 5004 din G.M.Dan Mihai MOCANU 1Abstract. In this Note, the problem C.O.: 5004 of the journal G.M. – 2/2009, p.103 is generalizedand a couple of simple results in a triangle are established; they deal with isogonal lines andangle trisectors.Keywords: altitudes, angle bisector, isogonal lines, angle trisectors, symmedians.MSC 2000: 51F20.Scopul prezentei note este de a stabili câteva rezultateîntr-un triunghi, legate de cevienele izogonale care sunt înacelaşi timp şi trisectoare.Fie ABC un triunghi oarecare şi A 1 , A 2 ∈ (BC).Se spune că cevienele (AA 1 , (AA 2 sunt izogonale dacăÖBAA 1 ≡ÖCAA 2 şi că sunt trisectoare ale unghiuluibA dacăÖBAA 1 ≡×A 1 AA 2 ≡ÖA 2 AC.Afirmaţia următoare are o demonstraţie imediată.BOA. .HA 1A 2Propoziţie. Dacă (AA 1 şi (AA 2 sunt ceviene izogonale şi una din ele este bisectoareaunghiului dintre o latură şi cealaltă, atunci ele sunt trisectoarele unghiuluibA.Demonstraţie. Să presupunem că (AA 1 este bisectoarea unghiuluiÖBAA 2 (lafel demonstrăm dacă (AA 2 ar fi bisectoarea unghiuluiÖA 1 AC). Avem, deci,ÖBAA 1 ≡×A 1 AA 2 . CumÖBAA 1 ≡ÖA 2 AC (AA 1 şi AA 2 sunt izogonale), rezultă căÖBAA 1 ≡×A 1 AA 2 ≡ÖA 2 AC, ceea ce trebuia demonstrat.Consecinţa 1. (C.O.: 5004 din G.M.-2/2009, p.102). În triunghiul ascuţitunghicABC se consideră înălţimea AH. Să se arate că, dacă bisectoarea unghiuluiÕBAHtrece prin centrul cercului circumscris triunghiului ABC, atunci semidreapta (AHeste trisectoare a unghiuluiÕBAC.Laura ConstantinescuDemonstraţie. Se ştie că înălţimea coborâtă din A şi diametrul ce are A ca oextremitate a sa sunt izogonale. Dar, prin ipoteză, acest diametru este bisectoareaunghiuluiÕBAH. Conform propoziţiei precedente (AH este trisectoare a unghiuluibA).Consecinţa 2. Dacă AH este înălţime, O este centrul cercului circumscris triunghiuluiascuţitunghic ABC, iar (AH este bisectoarea unghiuluiÕOAC, atunci (AOeste trisectoare a unghiuluiÕBAC.Demonstraţie. Se procedează ca în Consecinţa 1.Observaţie. Dacă una dintre (AH sau (AO este trisectoare, atunci şi cealaltăeste trisectoare.1 Elev, cl. a IX-a, Colegiul Naţional, IaşiC105


Următoarele rezultate pot fi privite ca reciproce ale consecinţelor de mai sus.Reciprocă (a Consecinţei 1). Dacă (AA 2 este o trisectoare a triunghiului ascuţit-unghicABC şi (AO este bisectoarea unghiuluiÖBAA 2 , atunci A 2 este înălţime.Demonstraţie. Din condiţiile ipotezei, rezultă că (AA 2 şi (AO fac cu laturiletriunghiului unghiuri de măsură A . Atunci şi (AH, ca izogonală a lui (AO, face cu3AC un unghi de măsură A 3 . Ca urmare (AA 2 coincide cu (AH.Reciprocă (a Consecinţei 2). Dacă (AA 1 este o trisectoare a triunghiului ascuţitunghicABC şi (AH este bisectoarea unghiuluiÖA 1 AC, atunci O ∈ (AA 1 .Demonstraţie. La fel ca pentru reciproca precedentă.Observaţie. În loc de ”înălţime-diametru”, putem lua o altă pereche de cevieneizogonale; de exemplu, ”mediană-simediană”. Trecerea rezultatelor de mai sus lanoua pereche se face cu uşurinţă.În încheiere, vom rezolva o problemă strâns legată de cele de mai sus.Problemă. Fie un triunghi ABC, AB > AC. Fie (AM) mediana relativ lalatura (BC) şi (AN) simediana corespunzătoare. Arătaţi că, dacă (AM sau (ANeste trisectoare a unghiuluiÕBAC, atunci ABAC = 2 cos A 3 .Demonstraţie. Ambele vor fi trisectoare. Cu ajutorul ariilor, avemAB · AM sinÖBAM = AC · AM sinÖCAM,de undeABAC = sinÖCAMsinÖBAM = sin 2A 3sin A 3= 2 cos A 3 .În toamna anului 1999 a apărut primul număr al revistei Recreaţii <strong>Matematice</strong>.În anul acesta se împlinesc 10 ani de la apariţia acesteia.Scrieţi numărul 2009 cu ajutorul numarului 10 şi folosind numai operaţiile deadunare şi împărţire !Care este numărul maxim de operaţii cu care puteţi face acest lucru ?Dar cel minim ?(Nu se acceptă termeni nuli !)Notă. Răspunsurile la aceste întrebări sunt date la pag.106


Matrices à coefficients dans un corps finiAdrien REISNER 1Abstract. It is considered the set A p of matrices of order 2 with their entries in Z p , defined byA p = {a = λM + µI; λ, µ ∈ Z p }, and some properties of this set are presented (Theorems 1,3,6,7,9).Keywords: unitary ring, the order of an element, the field Z p , isomorphism.MSC 2000: 15A33.A étant un anneau unitaire d ′ éléments neutre e on appelle ordre d ′ un élémentinversible a de A le plus petit entier positif n del que a n = e; dans ce cas l ′ ensembleG a = {e, a, a 2 , . . . , a n−1 } forme un sous-groupe du groupe G des éléments inversiblesde l ′ anneau A: l ′ ordre n de l ′ élément a ∈ G est le cardinal du ce sous-groupe G a .Pour toute matrice carrée M à coefficients dans un corps on désigne par T et ∆respectivement les deux applications suivantes: T :M→T (M)=trace de la matrice M,∆ : M → ∆(M)=déterminant de la matrice M.p désignant un nombre premier strictement supérieur à 3, on considère le corpsfini Z p des classes résiduelles modulo p. M et I étant les deux matrices suivantes àcoefficients dans Z p :=b4 b1=b1 b0M−b1 b0, Ib0 b1,on considère l ′ ensemble A p des matrices d ′ ordre 2 à coefficinets dans Z p défini par:A p = {a = λM + µI; λ, µ ∈ Z p }.Théorème 1. L ′ ensemble A p est un anneau commutatif unitaire pour les opérationsusuelles. De plus: Card A p = p 2 .Démonstration. On a immédiatement M 2 =b4M − I; on en déduit compte tenude la structure de l ′ ensemble M 2 (Z p ) que A p est un algèbre associative et unitaire.Enfin, la commutativité se vérifie directement. D ′ autre part, (I, M) étant une basede l ′ algèbre A p , on a A p ≃ Z 2 p et par suite: Card A p = (Card Z p ) 2 = p 2 .La proposition suivante se vérifie immédiatement par le calcul.Proposition 2. Pour toute matrice A = λM + µI ∈ A p , on a:a) T (A) =b2(b2λ + µ), ∆(A) = λ 2 + µ 2 +b4λµ;b) T (A 2 ) = T 2 (A) − 2∆(A).En particulier, T (A 2 ) =b2(b7λ 2 + µ 2 +b4λµ).Théorème 3. Pour A ∈ A p , deux quelconques des conditions suivantes impliquentla troisième: a) T (A) =b0, b) ∆(A) =b1, c) A est une matrice d ′ ordre 4.1 Centre de Calcul E.N.S.T., Paris; e-mail: adrien.reisner@enst.fr107


Démonstration. a) + b) ⇒ c) Le théorème de Cayley-Hamilton appliquée à lamatrice A donne:A 2 − T (A)A + ∆(A)I = O.Il vient alors avec les hypothèses a) et b): A 2 = −I et A 4 = I. L ′ ordre de la matriceA devise donc 4 et comme A 2 ≠ I (p étant impair), 2 n ′ est pas multiple de cet ordre.Finalement, A est d ′ ordre 4.a) + c) ⇒ b) Supposant les conditions a) et c) vérifiées, on a:A 2 = −∆(A)I et I = A 4 = ∆ 2 (A)I.On en déduit: ∆ 2 (A) =b1, ∆(A) ≠ −b1 soit ∆(A) =b1.b) + c) ⇒ a) Compte tenu de b) et c), A 2 = T (A)A − I d ′ oùI = A 4 = T 2 (A)A 2 −b2T (A)A + I = T (A)[T 2 (A) −b2]A + [b1 − T 2 (A)]I.On peut distinguer deux cas: I A et I sout liés, i.e. A = µI. Dans ce cas, A 2 = µ 2 I,A 4 = µ 4 I, d ′ oú, A étant d ′ ordre 4: µ 2 ≠b1, µ 4 =b1, µ 2 = −b1 et A 2 = −I, T (A)A = 0.Si T (A) ≠b0, alors A = O d ′ où T (A) =b0, impossible. Par suite T (A) =b0. II {A, I}est une famille libre. Dans ce cas l ′ égalité I = T (A)[T 2 (A) −b2]A + [b1 − T 2 (A)]Iimplique:b1 =b1 − T 2 (A) d ′ oú T (A) =b0. Le théorème est ainsi démontré.On considère la suite des entiers {Y k } k≥0 définie par Y 0 = 2 et Y k+1 = 2Y 2k − 1,k ≥ 0, dont les premiers terms sont: Y 0 = 2, Y 1 = 7, Y 2 = 97, Y 3 = 18817, . . .Théorème 4. Pour tout k ≥ 0, on a 2Y k ∈ T (M 2k ).Démonstration. Par récurrence sur k. Pour k = 0 la propriété se vérifie trivialmentpuisque 2Y 0 = 4 ∈b4 = T (M). Supposant la propriété vérifiée à l ′ ordre k,démontrons-là pour k + 1. On a immédiatment d ′ une part: 2Y k+1 = 4Yk 2 − 2 =(2Y k ) 2 − 2 et d ′ autre part T (M 2k+1 ) = T [(M 2k ) 2 ] = T 2 (M 2k ) −b2, compte tenude l ′ assertion b) de la Proposition 2 et puisque ∆(M) =b1. Donc, compte tenu del ′ hypothèse de récurrence: 2Y k+1 = (2Y k ) 2 − 2 ∈ T (M 2k+1 ).Théorème 5. La matrice M est d ′ ordre 2 k si et seulement si p|Y k−2 .Démonstration. Supposons la matrice M d ′ ordre 2 k , i.e. M 2k = I. On a:b4M 2 =c15par suite l−b4 −b1et ′ ordre de M ne divise pas 2 soit k ≥ 2. En posantalors A = M 2k−2 on a, M étant d ′ ordre 2 k : A 2 = M 2k−1 ≠ I, A 4 = M 2k = I.A ∈ A p est donc une matrice d ′ ordre 4 vérifiant aussi ∆(A) =b1. Compte tenu duthéorème 3, b) + c) ⇒ a), il vient alors: T (A) =b0 soit d ′ après le théorème précédent:2Y k−2 ∈ T (A) =b0, i.e. p divise 2Y k−2 et finalement p étant impair p|Y k−2 .Réciproquement, avec les mêmes notations si p|Y k−2 i.e. Y k−2 ≡ 0 (mod p), alorsT (A) =b0. Mais, comme ∆(A) =b1, le théorème 3, a)+b) ⇒ c), montre que la matriceA est d ′ ordre 4 soit M 2k = I et M 2k−1 = A 2 ≠ I. L ′ ordre de M divisant 2 k mais non2 k−1 est donc égal à 2 k .108


Théorème 6. Les deux assertions suivantes sont équivalentes:a)b3 n ′ est pas le carré d ′ un élément de Z p ;b) A p est un corps.Démonstration. a) ⇒ b) Sib3 n ′ est pas le carré d ′ un élément de Z p , alors pourA ∈ A p :∆(A) =b0 ⇒ A = O.En effet, supposons ∆(A) =b0. Pour A ∈ A p on a: ∆(A) =b0 = (λ +b2µ) 2 −b3µ 2 .µ ≠b0 impliqueraitb3 = (λµ −1 +b2) 2 ce qui est exclut par hypothèse. Donc µ =b0 et∆(A) =b0 = λ 2 soit: λ =b0 et finalement: A = O. Donc l ′ ensemble A p est formé dela matrice nulle et de matrices inversibles dans M 2 (Z p ).A ≠ O étant une matrice de A p , l ′ application A p → A p définie par X → AX estlinéaire et injective, donc surjective (dim A p = 2). Pour A ≠ O il existe B ∈ A p telleque: AB = 1 soit A −1 = B ∈ A p : les inverse des matrices non nulles de A p sont dansA p .b) ⇒ a) Nous allons montrer que, non a) ⇒ non b). Supposons qu ′ il existe a ∈ Z ptel que a 2 =b3. Dans ce cas: ∆(A) = [λ + (b2 − a)µ][λ + (b2 + a)µ]. Pour la matriceA = (a −b2)M + I(a ≠b2 puisqueb2 2 −b3 =b1) on a ∆(A) =b0 et (Cayley-Hamilton)A(A − T (A)I) = O avec A ≠ O et A ≠ T (A)I. On en déduit que A p n ′ est pas intègreet par suite que A p n ′ est pas un corps.Théorème 7. En supposant queb3 est un carré dans Z p :a) M est semblable à une matrice diagonale;b) A p est isomorphe à l ′ anneau produit Z p × Z p ;c) dans A p il y a p − 1 éléments de déterminantb1 et (p − 1) 2 éléments inversibles.Démonstration. a) L ′ équation caractéristique de la matrice M s ′ écrit X 2 −b4X +b1 = (x −b2) 2 − a 2 = 0 où a est tel que a 2 =b3. M ayant deux valeurs propre distinctesλ 1 =b2 + a et λ 2 =b2 − a, on en déduit que la matrice M est diagonalisable i.e. il=b2 + a b0existe P ∈ GL 2 (Z p ) telle que Mb0 b2 − aP −1 .b) L ′ application R → P −1 RP qui est un automorphisme pour la structure d ′ anneau,transforme toute matrice A = λM +µI de A p en une matrice diagonale. Or l ′ ensembleD p des matrices diagonales de M 2 (Z p ) ayant comme cardinal p 2 (je rappelle que CardA p = p 2 ), l ′ application A p → D p , A → P −1 AP est donc un isomorphisme.b0D ′ autre part, l ′ application Z p × Z p → D p , (α, β) →αétant de façonb0 β,immédiate un isomorphism, on en déduit que: A p ≃ Z p × Z p (cet isomorphisme estmême un isomorphisme d ′ algèbre).c) Compte tenu de l ′ isomorphisme précédent, comme ∆(A) = ∆(P −1 AP ), lenombre de matrices A ∈ A p telles que ∆(A) =b1 est égal aux nombre de couple (α, β)vérifiant αβ =b1. Il y en a p − 1 (choisir d ′ abord α).La démonstration de l ′ implication a) ⇒ b) du Théoreme 6 a montré que ∆(A) ≠b0 ⇒ A −1 ∈ A p . On en déduit que le nombre de matrices A inversibles de A p (i.e. le109


nombre des matrices A ∈ A p telles que ∆(A) ≠b0) est égal aux nombre des couples(α, β) ∈ Z p × Z p vérifiant αβ =b0 soit (p − 1) 2 .Corollaire 8. Dans le cas oùb3 est un carré dans Z p :a) l ′ ordre de la matrice M divise p − 1;b) si p|Y k−2 , alors 2 k |p − 1.Démonstration. a) L ′ ensemble des matrices de A p de déterminantb1 forment unsous-groupe multiplicatif de A ∗ p, groupe des matrices inversibles de A p (si A ∈ A p et∆(A) ≠b0, alors A −1 ∈ A p –voir a) ⇒ b) du théorème 6). Compte tenu de l ′ assertionc) du théorème précédent ce sous-groupe est fini de cardinal p − 1. Or la matriceM appartient à ce sous-groupe. L ′ ordre de M divise par suite p − 1 (ordre de cesous-groupe).b) Si p divise Y k−2 , alors la matrice M est d ′ ordre 2 k d ′ aprés le Théorème 5.L ′ assertion a) de ce corollaire permet alors de conclure.Théorème 9. En supposant queb3 n ′ est pas un carré dans Z p :a) ∆ est un homomorphisme du groupe multiplicatif des éléments non nuls de A pdans celui des éléments non nuls de Z p ;b) il existe k tel que p − 1 = Card (Im ∆) et Card Ker∆ = k(p + 1);c) il existent p + 1 éléments de déterminantb1 dans A p .Démonstration. a) A p étant un corps d ′ après le Théorème 6, l ′ assertion a) estévidente.b) On en déduit l ′ isomorphisme Im ∆ ≃ A ∗ p/Ker∆ et par suite: Card A ∗ p = p 2 −1 =(Card Im ∆)(Card Ker ∆). De plus, Im ∆ est un sous-groupe de Z ∗ p: il existe k telque p − 1 = k Card (Im ∆), d ′ où Card Ker ∆ = k(p + 1). Notons que 1 ≤ k ≤ p − 1.c) Les matrices A de Ker ∆ vérifient ∆(A) =b1. Il s ′ agit de montrer que CardKer ∆ = p + 1. L ′ égalité ∆(A) =b1 entraîne, compte tenu de l ′ assertion a) de laProposition 2: λ 2 + µ 2 +b4λµ −b1 =b0 (1). λ ∈ Z p étant donné, il existe donc 0, 1 ou2 éléments µ ∈ Z p tels que ∆(A) =b1 (Z p étant un corps), donc il existe au plus 2pcouples (λ, µ) vérifiant l ′ équation (1). D ′ autre part, le nombre de tels couples est égalà-voir b)-k(p + 1) = Card Ker ∆. On en déduit finalement que k = 1 et par suiteCard Ker ∆ = p + 1.Corollaire 10. Dans le cas oùb3 n ′ est pas un carré dans Z p :a) l ′ ordre de la matrice M divise p + 1;b) si p|Y k−2 , alors 2 k |p + 1.Démonstration. a) La matrice M de déterminantb1 appartient au sous-groupeKer ∆. L ′ ordre de M divise par suite l ′ ordre p + 1 de ce sous-groupe Ker ∆ -voirl ′ assertion c) du Théorème 9.b) De même qu ′ au Corollaire 8, si p divise Y k−2 , alors la matrice M est d ′ ordre2 k d ′ aprés le Théorème 5. L ′ assertion a) de ce corollaire permet alors de conclure.Remarque. Dans le cas oùb3 n ′ est pas un carré dans Z p , l ′ ordre de toute matriceA de A p vérifiant ∆(A) =b1 divise p + 1 - ordre de sous-groupe Ker ∆. Par suite∀A ∈ A p : A p+1 = I.110


O metodă de rezolvare a problemelorMaria MIHEŢ 1În această notă vom evidenţia o strategie importantă de rezolvare a problemelor:demonstraţia prin exprimarea în două moduri a unor mărimi. Această metodă s-afolosit încă în primele clase de gimnaziu, pentru obţinerea unor ecuaţii. Am întâlnit-o,de asemenea, în demonstraţia formulei pentru suma unghiurilor unui poligon convex:cu ajutorul unui punct O din interiorul poligonului se triangulează poligonul şi seexprimă în două moduri suma unghiurilor triunghiurilor obţinute. Prin exprimareaîn două moduri a ariei sau volumului se pot afla anumite distanţe (ne amintim, deexemplu, cum se poate afla înălţimea din vârful unghiului drept într-un triunghidreptunghic prin exprimarea în două moduri a ariei triunghiului), iar multe identităţicombinatoriale pot fi demonstrate folosind numărarea în două moduri.Am exemplificat metoda prin câteva probleme tip, iar la sfârşitul lucrării amîntocmit o listă de probleme însoţite de indicaţii de rezolvare. Am dori ca cititorii săîncerce mai întîi să le rezolve fără a apela la indicaţii.Această notă se adresează în special elevilor din clasele VI-VIII. Ea poate fi însăcompletată cu multe exemple de nivel liceal.Problema 1. Fie q un număr real, iar n ∈ N, n ≥ 2. Să se calculeze, în funcţiede q, suma S n = 1 + q + q 2 + . . . + q n .Soluţie. Exprimăm S n+1 în două moduri: mai întâi S n+1 = S n + q n+1 , iar, pede altă parte, S n+1 = qS n + 1. Rezultă că S n + q n+1 = qS n + 1, de unde obţinem că(∗) (q − 1)S n = q n+1 − 1.Astfel, dacă q ≠ 1, atunci S n = qn+1 − 1q − 1. Dacă q = 1, prin înlocuire în S n seobţine direct că S n=8


Problema 3. Să se arate că în orice triunghi ABC au loc relaţiile:aa)sin A = bsin B = c (teorema sinusurilor);sin Cb) BDCD = AB unde D este piciorul bisectoarei unghiului A (teorema bisectoarei).ACbSoluţie. a) Vom arăta căsin B = c ; pentru aceasta, fie M mijlocul laturiisin C[BC]. Considerând triunghiurile ABM şi ACM cu înălţimile din A, obţinem căS ABM = S ACM .Pe de altă parte, 2S ABM = AB · BM · sin ∠(ABC), iar 2S ACM = AC · CM ·bsin ∠(ACB). Cum CM = BM, rezultă că b sin C = c sin B, adicăsin B = csin C .b) Considerăm triunghiurile ABD şi ACD. Atunci S ABD= BD · h a= BDS ACD CD · h a CD .Pe de altă parte, deoarece D se află pe bisectoarea unghiului A, înălţimile din Dale triunghiurilor ABD şi ACD sunt egale; prin urmare S ABD= ABS ACD AC . Egalândrapoartele, obţinem teorema bisectoarei.În cele ce urmează, propunem celor interesaţi o serie de probleme care pot firezolvate folosind această strategie.1) Vasul A conţine apă, iar vasul B conţine alcool pur. Se toarnă în A un paharplin de alcool din B, apoi se scoate din A un pahar de amestec, care se toarnă în B.Ce relaţie există între cantitatea de alcool din A şi cantitatea de apă din B?Indicaţie. Exprimaţi în două moduri cantitatea de apă care lipseşte din A dupăcele două operaţii.2) Doi călători au plecat în acelaşi moment din localităţile A şi B, fiecare deplasându-sespre localitatea celuilalt cu viteză constantă. Ei s-au întâlnit la ora 13 şi,continuându-şi drumul, primul a ajuns în B la ora 17, iar cel de-al doilea în A la ora21. La ce oră au plecat cei doi în călătorie?Indicaţie. Calculaţi în două moduri distanţele parcurse de cei doi călători până laîntâlnire.3) O motonavă a plecat într-o cursă pe mare. Când motonava s-a îndepărtat cu180 mile de ţărm, a fost trimis după ea un hidroavion cu un mesaj urgent. Vitezahidroavionului este de zece ori mai mare decât viteza motonavei. La ce distanţa deţărm a fost ajunsă motonava?Indicaţie. Exprimaţi în două moduri timpul până la întâlnire.4) 98 de numere naturale consecutive a 1 < a 2 < . . . < a 98 au suma 137. Aflaţia 2 + a 4 + a 6 + . . . + a 98 .Indicaţie. Exprimaţi în două moduri a 1 + a 2 + a 3 + . . . + a 98 .5) În fiecare din pătrăţelele unui tabel dreptunghiular cu 4 linii şi 5 coloane scriemcâte un număr natural, astfel încât suma numerelor de pe fiecare linie să fie aceeaşi şisuma numerelor de pe fiecare coloană să fie aceeaşi (nu neapărat egală cu cea de pelinii). Fie S suma numerelor din tabel. a) Putem avea S = 30? b) Daţi două exemplede tabele cu S = 20. c) Aflaţi toate tabelele cu S < 20.Indicaţie. Calculaţi S în două moduri.6) Există poligoane convexe cu mai mult de trei unghiuri ascuţite?112


Indicaţie. Calculaţi în două moduri suma unghiurilor poligonului.7) Un număr natural n ≥ 2 cu număr impar de divizori este pătrat perfect.Indicaţie. Scrieţi în două moduri mulţimea divizorilor lui n şi grupaţi divizorii înperechi de formand, n do.8) Fie {d 1 , . . . , d k } mulţimea divizorilor naturali ai numărului natural n ≥ 2.Demonstraţi că (d 1 d 2 . . . d k ) 2 = n k .Indicaţie. Scrieţi în două moduri mulţimea divizorilor lui n.9) Demonstraţi că dacă m ≥ 2, n ≥ 2 sunt numere naturale, iar m divide n, atunci2 m − 1 divide pe 2 n − 1.Indicaţie. Calculaţi în două moduri 1 + 2 + . . . + 2 n−1 .10) Aflaţi cea mai mică valoare pentru max§a + 1 b , b + 1 aª, când a, b ∈ (0, ∞).Indicaţie. Evaluaţi în două moduri suma dintre a + 1 b şi b + 1 a .11) Pe latura (BC) a triunghiului ABC, se consideră punctele D, E astfel încât∠BAD ≡ ∠CAE. Demonstraţi că BDDC · BEEC = AB2AC 2 (Steiner).Indicaţie. Exprimaţi în două moduri S ABDşi S ABE.S ACD S ACE12) În triunghiul ascuţitunghic ABC, înălţimea AA′ şi mediana CM au aceeaşilungime. Aflaţi m(∠MCB).Indicaţie. Exprimaţi în două moduri aria triunghiului MCB.13) Fie n ∈ N. Demonstraţi că1− 1 2 + 1 3 − 1 1+. . .+4 2n − 1 − 12n = 1n + 1 + 1 1+. . .+n + 2 2n (Botez-Catalan).Indicaţie. Calculaţi în două moduri suma: 1+ 1 2 + 1 3 +. . .+ 1 n + 1n+1 + 11n+2+. . .+2n .14) Produsul a două numere de forma x 2 − 2y 2 , x, y ∈ N, este un număr de aceeşiformă.Indicaţie. Exprimaţi în două moduri (a + √ 2b)(c + √ 2d)(a − √ 2b)(c − √ 2d).15) Fie k ≥ 3 un număr natural impar. Arătaţi că dacă x 1 , x 2 , . . . , x k suntnumere întregi astfel încât |x 1 − x 2 | = |x 2 − x 3 | = . . . = |x k−1 − x k | = |x k − x 1 |, iarx 1 + x 2 + . . . + x k = m, atunci k îl divide pe m.Indicaţie. Exprimaţi în două moduri suma (x 1 − x 2 ) + (x 2 − x 3 ) + . . . + (x k−1 −x k ) + (x k − x 1 ).16) Aflaţi numerele N = a 0 a 1 . . . a 9 de 10 cifre (în baza 10) cu proprietatea că a 0este numărul cifrelor de 0 ale lui N, a 1 este numărul de 1-uri din scrierea lui N, . . . ,a 9 este numărul cifrelor de 9 ale lui N.Indicaţie. Calculaţi în două moduri suma cifrelor lui N.17) Numerele a 1 , a 2 , . . . , a n aparţin mulţimii {−1, 1}, iar a 1 a 2 + a 2 a 3 + . . . +a n−1 a n + a n a 1 = 0. Arătaţi că n este multiplu de 4.Indicaţie. Calculaţi în două moduri produsul (a 1 a 2 )(a 2 a 3 ) . . . (a n a 1 ).18) Un tablou cu m linii şi n coloane are toate elementele egale cu 1 sau cu −1,iar produsul elementelor de pe fiecare linie şi fiecare coloană este −1. Demonstraţi căm şi n au aceeaşi paritate.Indicaţie. Calculaţi în două moduri produsul elementelor din tablou.113


19) Într-un tablou cu 25 de linii şi 25 de coloane pentru fiecare i, j ∈ {1, . . . , 25}scriem la intersecţia dintre linia i şi coloana j numărul 1, dacă i divide pe j şi 0, dacăi nu divide pe j. Demonstraţi că numărul zerourilor din tablou este par.Indicaţie. Calculaţi în două moduri suma elementelor tabloului.20) La un turneu de şah au participat 14 şahişti, fiecare jucând câte o partidă cufiecare din ceilalţi 13. La sfârşitul turneului s-a constatat că suma punctelor obţinutede primii trei clasaţi a fost egală cu suma punctelor ultimilor 9. Ştiind că meciuldintre şahiştii de pe locurile 4 şi 5 nu s-a terminat remiză, aflaţi câte puncte a obţinutşahistul clasat pe locul 4 (pentru victorie se primeşte 1p, pentru remiză 1 2p, iar pentruînfrângere niciun punct).Indicaţie. Calculaţi în două moduri suma punctelor primilor trei şahişti.21) Putem scrie pe tablă 17 numere reale astfel încât suma oricăror 7 dintre elesă fie strict pozitivă, iar suma oricăror 11 să fie strict negativă?Indicaţie. Dacă x 1 , . . . , x 17 sunt cele 17 numere, calculaţi în două moduri sumaelementelor din tabloul (cu 11 linii şi 7 coloane):x 1 x 2 . . . x 7x 2 x 3 . . . x 8. . . . . . . . . . . . . . . . . .x 11 x 12 . . . x 17 .22) La o adunare a zeilor în Olimp, fiecare zeu a primit în cupa sa aceeaşi cantitatede ambrozie şi are voie să toarne din cupa sa în cupa altui zeu, însă doar o cantitateade ambrozie egală cu cea pe care o are celălalt zeu. La sfârşitul adunării, toatăambrozia a ajuns în cupa lui Zeus. Demonstraţi că numărul participanţilor a fost oputere a lui 2.Indicaţie. Scrieţi în două moduri cantitatea iniţială de ambrozie din cupa lui Zeus.Bibliografie1. A. Engel - Probleme de matematică – strategii de rezolvare, GIL, Zalău, 2006.2. M. Miheţ, E. Obădeanu - Teste şi probleme comentate pentru Concursul ”TraianLalescu” (gimnaziu), 19 (1988).3. L. Niculescu - Teme de algebră pentru gimnaziu, Ed. Cardinal, 1993.4. G. Polya - Cum rezolvăm o problemă?, Ed. Ştiinţifică, 1965.5. *** - Colecţia RMT, 1996-2009.Vizitaţi noua pagina web a revistei:http://www.recreatiimatematice.ro114


Metoda identificăriiSilviu BOGA 1Ne ocupăm în cele ce urmează de demonstrarea unor identităţi ce permit calcululunor sume sau al unor produse, prin formule de tipulnXk=1a k = b n , ∀n ∈ N ∗ , respectivnYk=1a k = b n , ∀n ∈ N ∗ .În ambele situaţii, cele mai populare strategii de abordare sunt următoarele:• demonstrarea relatiei prin raţionamente sintetic-constructive; considerăm că a-ceastă metodă este superioară calitativ faţă de oricare alta, însă presupunem o anumedexteritate din partea rezolvitorilor în utilizarea unor artificii de calcul şi raţionament;• demonstrarea relaţiei prin inducţie matematică – este metoda aleasă de majoritatearezolvitorilor, alegere motivată de gradul de accesibilitate al raţionamentului şicalculelor în acest caz, comparativ cu aplicarea strategiilor de tip sintetic-constructiv.Vom oferi cititorului spre comparare un procedeu mai puţin cunoscut, pe carel-am denumit metoda identificării, exprimat prin următoareaPropoziţie. a) Dacă a 1 = b 1 , b k −b k−1 = a k , ∀k ≥ 2, atuncinXk=1a k = b n , ∀n ≥ 1.b) Dacă a 1 = b 1 şib kb k−1= a k , ∀k ≥ 2, atuncinYk=1a k = b n , ∀n ≥ 1.Demonstraţie. Într-adevăr, nXk=1a k = b 1 +(b k − b k−1 ) = b 1 + (b 2 − b 1 ) + (b 3 −nXk=2b 2 ) + ... + (b n − b n−1 ) = b n , la sumare efectuând aşa-numitele reduceri telescopice.Faptul că metoda identificării nu apare printre căile frecvent aplicate pentru verificareaunor identităţi este explicabil prin aceea că celelalte metode au o arie de aplicabilitateincomparabil mai vastă. În ceea ce priveşte însă eleganţa raţionamentuluişi simplitatea, următorul exemplu cu rezolvări comparative va convinge, probabil, deavantajele metodei identificării.Problema 1. Demonstraţi că(knXk=12 + 1)k! = n(n + 1)!, ∀n ∈ N ∗ .Soluţia 1 (prin raţionamentele sintetic-constructive). Dacă a k = (k 2 + 1)k! estetermenul general al sumei, căutăm a, b, c astfel încât a k = a · k! + b(k + 1)! + c(k + 2)!;după calcule similare celor din metoda coeficienţilor nedeterminaţi, găsim a = 2, b =−3, c = 1, prin urmare a k = 2 · k! − 3 · (k + 1)! + (k + 2)! Sumând, obţinem căa k = 2 · 1! + (2 − 3) · 2! + (1 − 3)(n + 1)! + (n + 2)! = n · (n + 1)! şi identitaea estenXk=1astfel demonstrată.1 Profesor, Colegiul Tehnic ”I.C. Ştefănescu”, Iaşi115


Soluţia 2 (prin inducţie matematică). Identitatea se verifică pentru n = 1(obţinem 2 = 2, afirmaţie adevărată). Presupunem că egalitatea din enunţ are locpentru n (oarecare) şi demonstam că este adevărată şi pentru n + 1. Avemn+1(kXk=12 + 1)k! =(knXk=12 + 1)k! + ((n + 1) 2 + 1)(n + 1)! == n(n + 1)! + (n 2 + 2n + 2)(n + 1)! == (n + 1)!(n 2 + 3n + 2) = (n + 1)!(n + 2)(n + 1) = (n + 1)(n + 2)!Conform principiului inducţiei complete, identitatea are loc pentru orice n ∈ N ∗ .Soluţia 3 (prin metoda identificării). Identitatea este de tipul celei din Propoziţia1, a k = (k 2 + 1)k! şi b n = n(n + 1)!. Observăm că a 1 = b 1 (= 2), iar b k − b k−1 =k(k + 1)! − (k − 1)k! = (k 2 + 1)k! = a k , ∀k ≥ 2. Rezultă cănXk=1a k = b n = n(n + 1)!Prezentăm încă două probleme, în rezolvarea cărora vom folosi direct metodaidentificării:2Problema 2. Demonstraţi că 1 −k 2 + 3k + 2‹= n + 33(n + 1) , ∀n ∈ N∗ .Soluţie. Identitatea este de tipulnYk=1nYk=1a k = b n , ∀n ≥ 1, cu a k = 1 −2k 2 + 3k + 2 şib n = n + 33(n + 1) . Se observă că a b k1 = b 1 , iar = k + 3b k−1 3(k + 1) : k + 23k = k 2 + 3k(k + 1)(k + 2) =21 −k 2 + 3k + 2 = a k, ∀k ≥ 2. Conform Propoziţiei 1, identitatea este demonstrată.1sin x 2sinProblema 3. Demonstraţi că sin kx =nXk=1nx (n + 1)x2 · sin2sin x , ∀n ∈ N ∗ .2Soluţie. Cu notaţiile din Propoziţia 1, avem evident că a 1 = b 1 , iar b k − b k−1 =sin kx 2sin(k + 1)x2− sin(k − 1)x2sin kx 2‹= 1sin x sin kx 2 · 2 · sin x22 cos kx 2 =2 sin kx 2 cos kx 2 = sin kx = a k, ∀k ≥ 2, prin urmare are loc cerinţa problemei.Propunem cititorului stabilirea următoarelor identităţi valabile pentru ∀n ∈ N ∗ :4. (2k − 1) 2 = n(4n2 − 1)k 2 n(n + 1), 5.=3(2k − 1)(2k + 1) 2(2n + 1) ,6.nXk=1nYk=1Bibliografie1 + p − 1 p(n + 1)k(k + p)‹=n + pnYk=1, 7. cos 2 k x = sin 2n+1 x2 n · sin 2x .1. L. Panaitopol - Inducţia matematică, Gil, Zalău, 2005.2. Gh. Rizescu - Sume şi produse, Ed. Sigma, Bucureşti, 1999.nXk=1116


Bisectoarele exterioarenu sunt ca bisectoarele interioareDumitru MIHALACHE şi Marian TETIVA 1Abstract. It is known the fact that an analogous result to the Steiner-Lehmus Theorem for theexterior bisectors does not hold. The authors of this article present this result in a convenient wayfor the reader who wants to see how a case study should be performed.Keywords: angle bisector, exterior angle bisector, Steiner-Lehmus Theorem.MSC 2000: 97C20.Încă din clasa a şasea elevii învaţă despre triunghiul isoscel. Mai precis, ei învaţăîn primul rînd că dacă un triunghi are două laturi congruente, atunci şi unghiurileopuse acelor laturi sunt congruente. Mai află că, într-un asemenea triunghi, medianelecorespunzătoare laturilor congruente sunt congruente şi acelaşi lucru se întâmplă cuînălţimile corespunzătoare acestor laturi, şi tot congruente sunt şi bisectoarele unghiurilorcongruente. De prisos să mai spunem că privim toate aceste linii importante aletriunghiului ca pe nişte segmente, toată lumea înţelege la ce ne referim. De asemenea,este vorba de bisectoarele unghiurilor interioare ale triunghiului, deoarece despre celeale unghiurilor exterioare nu prea se mai vorbeşte în noile programe. Totuşi n-ar figreu de demonstrat şi congruenţa a două dintre ele dacă le definim corespunzător şitriunghiul este isoscel, dar nu echilateral. Vom avea nevoie de această noţiune, aşa căexplicăm acum la ce ne referim. Să spunem că triunghiul ABC are laturile [AB] şi[AC] de lungimi diferite şi să considerăm bisectoarele unghiurilor sale exterioare careau vârful în A (acestea fiind unghiurile adiacente şi suplementare cu unghiul interiorcare are vârful în A), deocamdată ca semidrepte (se ştie că sunt semidrepte opuse).Una din ele intersectează dreapta BC; dacă AB > AC vedeţi uşor că intersecţia Meste astfel încât C se află între B şi M. Vom numi atunci bisectoare exterioară din A atriunghiului ABC segmentul [AM]. E clar că, dacă AB = AC, bisectoarea exterioarănu poate fi considerată în sensul acestei definiţii.Nu mult după aceea, sau chiar deodată, elevii află că şi reciprocele acestor afirmaţiisunt adevărate. Dar, găsindu-se la începutul studiului geometriei, le va fi mult maigreu să le justifice. Faptul că un triunghi cu două mediane congruente (sau cu douăînălţimi congruente) este isoscel nu prea poate fi demonstrat cu una, cu două, deoarecenecesită cazuri speciale de congruenţă. Iar despre a arăta că un triunghi cu două bisectoarecongruente este isoscel nici nu poate fi vorba atâta vreme cât el face obiectulunei renumite teoreme care a pus la grea încercare chiar mari matematicieni. Dar esteadevărat, ceea ce nu se mai poate afirma în cazul bisectoarelor exterioare (şi despreasta vom vorbi mai departe). Profesorul C.L. Lehmus, în 1840, a cerut pentru primadată o soluţie sintetică a problemei, care a fost găsită, printre alţii, de marele geometruJacob Steiner (se pare că Lehmus însuşi avea să găsească ulterior o demonstraţie);1 Profesori, Colegiul Naţional ”Gheorghe Roşca Codreanu”, Bârlad117


teorema a rămas cunoscută cu aceste nume: teorema Steiner-Lehmus. Mai multedespre istoria ei şi demonstraţia în cadrul geometriei euclidiene clasice se găsesc, de exemplu,pe internet la adresa http://forumgeom.fau.edu/FG2005volume5/FG200525.pdf[2] (ca să dăm un exemplu la îndemâna cititorilor mai tineri; altminteri, orice carteserioasă de geometrie euclidiană elementară cuprinde şi această teoremă).Noi vom începe cu o demonstraţie prin calcul a acestui renumit rezultat, probabilfoarte cunoscută cititorilor. Vom considera un triunghi oarecare ABC în care vomfolosi notaţiile obişnuite: a, b, c sunt lungimile laturilor (opuse vârfurilor A, B, respectivC), l a este lungimea bisectoarei obişnuite (interioare) a unghiului cu vârful înA. Formulala 2 = bc −a2 bc(b + c) 2care furnizează lungimea acestei bisectoare este notorie, nu mai insistăm asupra ei.Ca şi asupra următorului calcul, pe care-l recomandăm celor ce nu-l cunosc încă.Exerciţiul 1. Să se arate că, cu notaţiile de mai sus, aveml 2 a − l 2 bc= (b − a)1 + ab(a2 + b 2 + c 2 + ab + 2ac + 2bc)(a + c) 2 (b + c) 2 .Evident, de aici se obţine imediat teorema Steiner-Lehmus şi chiar mai mult, sevede că avem l a < l b ⇔ a > b (deoarece expresia din paranteză este strict pozitivă).Această proprietate este valabilă şi pentru mediane sau înălţimi, adică m a < m b ⇔a > b şi h a < h b ⇔ a > b (dacă notăm m a şi h a lungimea medianei, respectivînălţimii din A), dar vom vedea că pentru bisectoarele exterioare nu este câtuşi depuţin adevărată. Colateral, vă puteţi ocupa şi deExerciţiul 2. Demonstraţi că m a < m b ⇔ a > b şi h a < h b ⇔ a > b.Acum să vorbim şi despre bisectoarele exterioare. Vom notă cu e a lungimea bisectoareiexterioare din A (dacă b ≠ c). Formulae 2 a =a2 bc(b − c) 2 − bceste şi ea destul de cunoscută (dacă n-o ştiţi, o puteţi obţine fie din relaţia lui Stewart,fie dintr-un calcul cu arii, folosind teorema bisectoarei exterioare pentru a exprima înfuncţie de a, b, c lungimile segmentelor determinate pe [BC] de bisectoare, respectivfolosind expresia lui cos(A/2)). Cu ajutorul ei obţinem imediat, la fel ca mai suse 2 a − e 2 bc= (a − b)1 + ab(a2 + b 2 + c 2 + ab − 2ac − 2bc)(a − c) 2 (b − c) 2 .Desigur, putem considera e a şi e b (şi calcula această expresie) numai într-un triunghineisoscel, deci mai departe presupunem că oricare două dintre a, b, c sunt distincte.Exerciţiul 3. Continuaţi acest calcul pentru a obţinee 2 a − e 2 bc= (b − a)(a + b − c)(c3 − (a + b)c 2 + 3abc − ab(a + b))(a − c) 2 (b − c) 2 .118


E clar acum că nu avem cum să obţinem egalitatea laturilor din egalitatea bisectoarelorexterioare, deşi acesta este unul din cazuri: dacă presupunem e a = e b rezultăcă unul din factorii de la numărător trebuie să fie zero. Dintre aceşti factori doar dea + b − c putem fi siguri că este nenul (pozitiv pentru a, b, c lungimile laturilor unuitriunghi). Rămâne totuşi al treilea factor, care ne permite să demonstrăm următoareaPropoziţie. Pentru orice numere reale pozitive şi distincte a şi b există c pozitivşi diferit de a şi de b astfel încât a, b, c pot fi laturile unui triunghi, iar în acesttriunghi e a = e b .Demonstraţie. Să presupunem, de exemplu, că a < b. Pentru funcţia continuăf definită prinf(x) = x 3 − (a + b)x 2 + 3abx − ab(a + b), ∀x ∈ R,avem f(b − a) = −2a 3 < 0 şi f(b) = ab(b − a) > 0, deci există c ∈ (b − a, b) astfelîncât f(c) = 0. Acest c este diferit şi de a, deoarece f(a) = ab(a − b) < 0; pentru căb > a, b > c şi c > b − a implică b < a + c, triunghiul cu laturile a, b şi c există şi esteneisoscel. Exprimarea de mai sus a diferenţei e 2 a − e 2 bne arată că, în acest triunghi,e a = e b , ceea ce încheie demonstraţia.Se mai observă că semnul acestei diferenţe nu poate fi stabilit numai în funcţie desemnul lui b − a, de aceea un analog al proprietăţii l a < l b ⇒ a > b nu funcţioneazăpentru bisectoarele exterioare. În plus, deşi tocmai am arătat că există triunghiurineisoscele cu două bisectoare exterioare congruente, cititorul va vedea uşor că nu sepoate ca toate bisectoarele exterioare să aibă lungimi egale.Exerciţiul 4. Să se arate că într-un triunghi neisoscel, nu se poate ca toate celetrei bisectoare exterioare să fie congruente.În sfârşit, să ne mai punem o întrebare firească: putem da un exemplu concretde asemenea triunghi (neisoscel, dar care are două bisectoare exterioare congruente)?Căci una e să spui: există un astfel de triunghi, şi alta e dacă afirmi răspicat cătriunghiul cu laturile de anumite lungimi (bine precizate) are respectiva proprietate.Avem un asemenea exemplu!Exerciţiul 5. Să se arate că în triunghiul cu laturile a = 5 − √ 7, b = 5 + √ 7 şic = 6 avem e a = e b .Mai mult, putem găsi toate triunghiurile cu această proprietate.Propoziţie. Un triunghi neisoscel ABC are proprietatea e a = e b dacă şi numaidacă există un număr p din intervalul (0, 1) şi un număr pozitiv c astfel încât lungimilelaturilor sale să fie date decBC =+ 1 −È(1 − p)(p2(p + 1)3p 2 + 3p + 4),cAC =+ 1 +È(1 − p)(p2(p + 1)3p 2 + 3p + 4)şi AB = c.Demonstraţie. Desigur, ca să avem e a = e b într-un triunghi neisoscel trebuie caultimul factor de la numărătorul expresiei e 2 a − e 2 bsă fie zero, adică trebuie să aibă locegalitateac 3 − (a + b)c 2 + 3abc − ab(a + b) = 0.119


Cu notaţiile x = a/c şi y = b/c aceasta se mai scrie (după împărţirea cu c 3 ):1 − (x + y) + 3xy − xy(x + y) = 0 ⇔ 1 − s + 3p − sp = 0,dacă punem s = x + y şi p = xy. Desigur, x şi y sunt soluţiile ecuaţiei de gradul aldoilea t 2 − st + p = 0, iar dacă ştim pe x şi pe y, cunoaştem şi laturile triunghiului:BC = cx, AC = cy şi BC = c. Relaţia de mai sus ne furnizează s = (3p + 1)/(p + 1)şi un calcul rapid ne arată că discriminantul ecuaţiei de gradul al doilea este atunci∆ = s 2 − 4p = 3p + 1 − 4p =p + 1‹2(1 − p)(4p2 + 3p + 1)(p + 1) 2care va fi pozitiv dacă şi numai dacă p ∈ (0, 1) (de la bun început e clar că p trebuiesă fie pozitiv, iar din ∆ > 0 rezultă că p < 1). Dacă ∆ > 0, ecuaţia va aveadouă soluţii pozitive şi distincte care conduc exact la formulele din enunţ pentrulungimile laturilor. Interesant este că aceste valori ale lui x şi y produc întotdeauna(fără să fie nevoie de vreo condiţie suplimentară) lungimile laturilor unui triunghi(în care, credem că e foarte clar, e a = e b ). Într-adevăr, ordinea lor se vede că estea < c < b, ceea ce corespunde inegalităţilor x < 1 < y (care rezultă repede din(x − 1)(y − 1) = p(p − 1)/(p + 1) < 0), caz în care mai trebuie să verificăm doarb < a + c ⇔ b − a < c ⇔ y − x < 1. Darşi demonstraţia este încheiată.y − x = √ ∆ = 1 −4p3(p + 1) 2 < 1Noi am considerat p = 1/2 şi c = 6 pentru a obţine exemplul din Exerciţiul 5, darcititorul are acum la dispoziţie formulele generale care dau lungimile laturilor acestortriunghiuri şi îşi poate construi oricâte exemple doreşte. Întrebare (inevitabilă): găsimtriunghiuri cu toate lungimile laturilor numere întregi în care e a = e b ? (Nu cunoaştemrăspunsul la acestă întrebare.)Probabil aţi observat că titlul, aşa glumeţ-aluziv cum este el spune exact ceam vrut noi să evidenţiem în această notă: proprietăţile bisectoarelor interioare(l a = l b ⇒ a = b, respectiv l a < l b ⇒ a > b care au loc şi pentru celelalte liniiimportante obişnuite ale triunghiului) nu se păstrează deloc pentru bisectoarele exterioare.Lăsăm în seama cititorului interesat (şi mai bun prieten cu geometria euclidianăclasică) să explice pur geometric aceste fapte aparent stranii. Menţionăm căexistenţa triunghiurilor neisoscele cu două bisectoare exterioare congruente e cunoscutăde mult: de exemplu, se vorbeşte despre ele în [1], la pagina 235 (unde suntnumite triunghiuri pseudoisoscele).Bibliografie1. F. G.-M. – Exercices de géométrie, 1912.2. K.R.S. Sastry – A Gergonne Analogue of the Steiner - Lehmus Theorem, ForumGeometricorum, 5 (2005), 191-195.120


Metoda falsei ipoteze - variante de utilizareD.M. BĂTINEŢU-GIURGIU 1În clasele primare elevii întâlnesc mărimi care sunt dependente una de alta. Deexemplu, dacă în produsul F 1 · F 2 creşte (descreşte) unul din factori de un număr deori, acelaşi lucru se întâmplă şi cu produsul, iar dacă unul din factori creşte (descreşte)de un număr de ori şi produsul rămâne neschimbat, atunci celălalt factor descreşte(creşte) de acelaşi număr de ori.Problemele în care apar astfel de mărimi se rezolvă uneori cu metoda falsei ipoteze,care comportă parcurgerea următoarelor etape: 1) se face o ipoteză arbitrară asupramărimilor de aflat, ce va fi, în general, în contradicţie cu datele problemei; 2) seanalizează sursele eşecului şi erorii comise şi se trag concluziile corespunzătoare; 3) semodifică ipoteza (adică valorile date arbitrar mărimilor) pe baza concluziilor punctuluiprecedent, în scopul obţinerii soluţiei; 4) se verifică corectitudinea soluţiei găsite.Deoarece în programa claselor primare nu mai este inclusă această metodă, dămun exempul de problemă pe care vom ilustra această cale de rezolvare.Problemă. Un producător a vândut la piaţă 1200 kg mere de 2 lei/kg, 3 lei/kg şi4 lei/kg, pe care a încasat 3300 lei. Ştiind că producătorul a avut cantităţi egale demere de 2 lei/kg şi 3 lei/kg, să se afle câte kilograme de mere de 2 lei/kg, 3 lei/kg şi4 lei/kg a vândut producătorul.Prezentăm şase variante de rezolvare, utilizând de fiecare dată metoda falseiipoteze, cu o ipoteză sau cu două ipoteze arbitrare.Varianta 1. Presupunem că toate merele sunt de 4 lei/kg. În această ipotezăse încasează 4 × 1200 = 4800 lei. Suma reală este depăşită cu 4800 − 3300 = 1500lei. Această depăşire apare din faptul că am înlocuit x kilograme mere de 2 lei/kg şix kilograme mere de 3 lei/kg cu 2x kilograme mere de 4 lei/kg. Înlocuind 1 kg demere de 2 lei/kg şi 1 kg de mere de 3 lei/kg cu 2 kg mere de 4 lei/kg, suma realăse măreşte cu 8 − (2 + 3) = 3 lei. Avem un număr de 1500 : 3 = 500 înlocuiri, ceeace înseamnă că avem 500 kg mere de 2 lei/kg, 500 kg de mere de 3 lei/kg şi 200 kgmere de 4 lei/kg. Se verifică uşor că aceste cantităţi îndeplinesc condiţiile problemei:500 × 2 + 500 × 2 + 200 × 4 = 3300 lei.Varianta 2. Presupunem că avem numai mere de 2 lei/kg şi de 3 lei/kg. În aceastăipoteză suma încasată de producător este 600 × 2 + 600 × 3 = 3000 lei. Diferenţadintre suma reală şi cea din presupunere este 3300−3000 = 300 lei. Această diferenţă1 Profesor, Colegiul Naţional ”Matei Basarab”, Bucureşti121


provine din faptul că, de x ori am înlocuit câte 2 kg de mere de 4 lei/kg cu 1 kg demere de 2 lei/kg şi 1 kg de mere de 3 lei/kg, adică de câte ori se cuprinde (8−2−3) în300, de unde aflăm că x = 100. Înseamnă că producătorul a vândut 2 × 100 = 200 kgmere de 4 lei/kg, 500 kg de mere de 2 lei/kg şi 500 kg de mere de 3 lei/kg. Verificarease face la fel.Varianta 3. Presupunem că cele trei categorii de mere sunt în cantităţi egale. Înaceastă ipoteză suma încasată este 400 × 2 + 400 × 3 + 400 × 4 = 3600 lei. Diferenţadintre suma presupusă şi cea reală este 3600 − 3300 = 300 lei. Evident, cantităţile nupot fi egale. Trebuie să înlocuim de x ori câte 2 kg mere de 4 lei/kg cu 1 kg de merede 2 lei/kg şi 1 kg de mere de 3 lei/kg. Obţinem x = 300 : (8 − 2 − 3) = 100, ceea ceînseamnă că trebuie să micşorăm cantitatea de mere de 4 lei/kg cu 2 × 100 = 200 kg.Am aflat că producătorul a vândut 200 kg mere de 4 lei/kg, 500 kg mere de 2 lei/kgşi 500 kg mere de 3 lei/kg.Varianta 4. Observăm întâi că producătorul a vândut cantităţi de mere exprimateprin numere naturale pare. Să presupunem că producătorul a vândut 500 kgmere de 4 lei/kg. În acest caz suma încasată este 350 × 2 + 350 × 3 + 500 × 4 = 3750lei. Diferenţa dintre suma din presupunere şi cea reală este 3750 − 3300 = 450 lei.Evident, cantitatea de mere de 4 lei/kg trebuie micşorată. Înlocuim de x ori câte 2 kgde mere de 4 lei/kg cu 1 kg de mere de 2 lei/kg şi 1 kg mere de 3 lei/kg, diminuândsuma presupusă cu 8−(2+3) = 3 lei. Cantitatea de mere de 4 lei/kg trebuie micşoratăcu 450 : 3 × 2 = 300 kg, ceea ce înseamnă că producătorul a vândut 200 kg mere de4 lei/kg, 500 kg mere de 2 lei/kg şi 500 kg mere de 3 lei/kg.Varianta 5. Presupunem că producătorul a vândut 300 kg mere de 2 lei/kg,300 kg mere de 3 lei/kg şi 600 kg mere de 4 lei/kg. În această ipoteză producătorulîncasează 300 × 2 + 300 × 3 + 600 × 4 = 3900 lei. Diferenţa dintre suma presupusă şicea reală este de 3900 − 3300 = 600. Trebuie să înlocuim de x ori câte 2 kg de merede 4 lei/kg cu 1 kg mere de 2 lei/kg şi 1 kg mere de 3 lei/kg, iar suma încasată înaceastă ipoteză se micşorează cu 8 − (2 + 3) = 3 lei. Cantitatea de mere de 4 lei/kgtrebuie micşorată cu 600 : 3 × 2 = 400 kg, ceea ce înseamnă că producătorul a vândut200 kg mere de 4 lei/kg, 500 kg mere de 2 lei/kg şi 500 kg mere de 3 lei/kg.Varianta 6. Prima ipoteză: producătorul a vândut 320 kg mere de 2 lei/kg, 320kg mere de 3 lei/kg şi 560 kg mere de 4 lei/kg. Suma încasată în prima ipoteză este320 × 2 + 320 × 3 + 560 × 4 = 3840 lei. Ipoteza a doua: producătorul a vândut 340 kgmere de 2 lei/kg, 340 kg mere de 3 lei/kg şi 520 kg mere de 4 lei/kg. În această ipotezăsuma încasată este 340 × 2 + 340 × 3 + 520 × 4 = 3780 lei. Se observă că la o creşterede 40 kg mere de 4 lei/kg, prima sumă presupusă se micşorează cu 3840 − 3780 = 60lei. Prima sumă presupusă trebuie să scadă cu 3840 − 3300 = 540 lei. Cantitatea de560 kg mere de 4 lei/kg trebuie să scadă cu 540 : 60 × 40 = 360 kg, ceea ce înseamnăcă producătorul a vândut 200 kg mere de 4 lei/kg, 500 kg mere de 2 lei/kg şi 500 kgmere de 3 lei/kg. Verificarea este imediată.122


Liceul Teoretic ”Garabet Ibrăileanu” 1La 110 ani de la înfiinţareLa prima vedere Liceul Teoretic ”Garabet Ibrăileanu”, cu arhitectura sa uşorconfundabilă cu aceea a şcolilor ridicate în ”epoca de aur”, pare o instituţie şcolarăde dată recentă. Nimic mai fals. Liceul acesta are mai bine de 100 de ani de laînfiinţare şi istoria sa reflectă convulsiile prin care a trecut societatea românească,drama intelectualităţii şi chiar ”drama psihologiei” pentru a prelua titlul unei cărţide Vasile Pavelcu, fost profesor la această şcoală.Legea învăţământului secundar promulgată de Spiru Haret cerea candidaţilor laprofesorat să fi trecut un examen de capacitate. Dar pentru a se putea prezenta laacest examen candidaţii trebuiau să frecventeze un Seminar Pedagogic, să parcurgămodulul pedagogic, am spune noi astăzi, şi implicit să facă un număr de ore depractică pedagogică. De aceea s-a creat Seminarul Pedagogic Universitar cupropria şcoală de aplicaţie. Aceasta se întâmpla la 1 noiembrie 1899, localul şcoliifiind casa Aslan de la vale de Universitatea nouă din Copou. Şcoala de aplicaţie aveaun dublu rol: pentru studenţi sau alsolvenţi era locul unde îşi desfăşurau practicapedagogică, pentru profesorii universitari era locul în care îşi puneau în practicămetodele moderne de predare, un laborator de pedagogie experimentală. Se observădin capul locului coerenţa care exista între activitatea didactică, pregatirea pedagogicăa viitorilor profesori şi activitatea de cercetare a universitarilor. Nu este de mirare căde aici au plecat majoritatea cadrelor didactice din liceele Moldovei. Mulţi absolvenţi1 N. R. Pentru informaţii mai ample recomandăm cartea Magda Negrea – Liceul de Aplicaţieal Semninarului Pedagogic Universitar – Iaşi cu un studiu introductiv de prof.dr. AdrianNeculau.123


ai şcolii de aplicaţie au revenit la SPU ca practicanţi, iar unii chiar ca profesori. Deasemenea unii profesori ai SPU au pus bazele unor şcoli asemănătoare la Cernăuţi(Constantin Narly)) şi Cluj sau a Universităţii din Chişinău (Constantin Fedeleş).Liceul de Aplicaţie al SPU a funcţionat cu succes până la reforma învăţământuluidin 1948. O istorie de jumătate de veac marcată de personalitatea celordoi directori, ambii pedagogi de frunte ai şcolii ieşene: Ioan Găvănescul şi ŞtefanBârsănescu. Ioan Găvănescul, pedagog cu studii în Germania, a pus bazele unuiregulament organic al instituţiei. înainte de deschiderea şcolii a convocat profesorii înşase conferinţe în care a fixat normele şi principiile generale didactice şi de ordine şidisciplină. Ideea călăuzitoare a fost ”educaţia ca scop, instrucţia ca mijloc”. Şcoalatrebuie să dezvolte simţul practic şi spontaneitatea mentală. De aceea în predare se vafolosi metoda activă, intuiţia şi corelaţia între obiectele de studiu. Educaţia nu trebuiesă fie teoretică şi intelectuală ci ”practică şi voluntară”, elevul, asemeni lui RobinsonCrusoe, să ştie în orice împrejurare ”a găsi mijloacele practice pentru a învingeorice greutate”. Pe lângă activitatea teoretică, elevul era antrenat în multe activităţipractice: cartonaj, modelaj, tâmplărie. La SPU din Iaşi s-a introdus pentru primadată lucrul manual la o şcoală de cultură generală. Ioan Găvănescul a fost directorpână în 1932, când a ieşit la pensie, dar a continuat să ţină conferinţe, era membrual partidului conservator, a avut convingeri de dreapta. La venirea comuniştilor laputere, deşi se apropia de 90 de ani a fost arestat şi moare în închisoare în 1951.Între 1932 şi 1938 director este Paul Nicorescu, istoric, fiind ajutat de VasilePavelcu, responsabil cu practica pedagogică şi de E. Briul care răspundea de activitateadiriginţilor şi disciplina şcolară. Sub directoratul lui Paul Nicorescu se construieştearipa de vest a şcolii, care există şi azi la adresa din Toma Cozma, nr. 2.Ştefan Bârsănescu, care îi urmează ca director, continuă să construiască în ciudavremurilor de război. Sub coordonarea lui Ştefan Bârsănescu se construieşte laturadin centru şi se toarnă fundaţia pentru aripa est care ar fi închis acest amsamblu.în această activitate, Ştefan Bârsănescu este secundat de directorul administrativI. Rick şi de directorul adjunct D. Gafiţanu.Odată cu venirea lui Ştefan Bârsănescu la conducerea Seminarului Pedagogic Universitar,începe o perioadă de dezvoltare distinctă a acestei instituţii. Cu studii laUniversitatea din Iaşi şi două doctorate în Germania, Ştefan Bârsănescu are toateatributele unui mare pedagog. Spre deosebire de Ioan Găvănescul, care punea accentpe formarea caracterului şi pregătirea practică, Ştefan Bârsănescu pune accent pelecţie, pe transmiterea de valori. Totul gravitează în jurul ideii de cultură. PentruŞtefan Bârsănescu un ”om cultivat” este acela care se conduce în activitatea sa după”valorile veşnice” îmbogăţindu-şi şi ordonându-şi năzuinţele după adevăr (ştiinţă),bine(morală) şi frumos(artă). În privinţa studenţilor practicanţi, Ştefan Bârsănescuîi roagă pe profesori ”să-i câştige ca pe nişte prieteni, să nu fie descurajaţi, să se simtăca într-o mare familie. Practicanţii asistau la un număr impresionant de ore pânăerau lăsaţi să facă lecţii de probă. Toate lecţiile erau discutate. Fiecare practicant îşiîntocmea un dosar consistent cu observaţii, rapoarte, planuri de lecţii, un adevăratghid cu care pornea în activitatea la catedră.Dar în martie 1944 tot Iaşul pleacă în refugiu. Tot ce s-a putut demonta şi ambalaa fost pus în 14 vagoane şi transportat în Banat. Iaşul est bombardat, trei bombe124


căzând şi pe Liceul de aplicaţie. În martie 1945, Liceul revine din refugiu şi pe 9 apriliese încep cursurile. Zece vagoane din cele 14 sunt recuperate, se fac reparaţii, încâtla 1 oct. 1945 clădirea adăposteşte atât Liceul de aplicaţie cât şi clase de la LiceulInternat şi de la ”Oltea Doamna”. Se fac demersuri pentru a continua construcţiaaripei de est la care de mulţi ani era turnată fundaţia. Totul părea încă posibil,dar reforma învăţământului din 1948 face totul imposibil. Seminarul Pedagogic sedesfiinţează, Liceul de aplicaţie îşi schimbă această menire şi devine Liceul Clasic.Se poate spune că începe drama psihologiei şi pedagogiei. Disciplinile acestea aproapecă dispar din învăţământul universitar, practica pedagogică se reduce ca număr deore şi ca importanţă, dar, în mod paradoxal, se face cu precădere tot la fostul Liceu deaplicaţie. Ştefan Bârsănescu este îndepărtat atât de la liceu cât şi de la Universitate.Abia dacă ocupă un post de bibliotecar. Este reprimit la catedră în 1957. Aveanu numai cultură, ci şi har îşi aminteşte studentul de atunci Adrian Neculau, acumprofesor de psihologia grupului social şi fervent admirator al S.P.U. de altădată.Liceul de aplicaţie fusese doar de băieţi, Liceul Clasic este mixt şi are 12 clase. Din1953 se numeşte Şcoala Medie Clasică, apoi din anul următor se numeste ŞcoalaMedie nr. 2 de fete. Toate aceste schimbări, adevărate convulsii pentru învăţământîn general şi pentru liceul de care ne ocupăm în special, precum şi politizarea în excesa lecţiilor au drept scop ruperea de istoria şcolii, uitarea bunei tradiţii. Din anul 1956,şcolile medii devin mixte şi se numără din nou, liceul nostru semicentenar devenindŞcoala Medie nr. 4. Pentru ca uitarea să se aştearnă şi mai bine pe istoria şcolii, înloc să se termine de construit aripa est, se începe construirea unui nou local pe stradaOastei nr. 1. Şcoala Medie nr. 4, care între timp a căpătat şi numele de Liceul”Garabet Ibrăileanu”, se mută în noul local în 1962. În localul vechi continuă săfuncţioneze internatul pentru mai mulţi ani, apoi aici se instalează dispensarul pentruelevi şi studenţi. Vis-à-vis, în vechea şcoală de învăţătoare, funcţionează Facultateade psihologie şi pedagogie, adică fostul SPU. E doar ironia sorţii sau contratimpultragic care ne marchează pe noi, românii? Vechea clădire din Toma Cozma nr. 2aşteaptă şi acum să fie terminată şi să servească drept şcoală între marile şcoli aleIaşului: Universitate, Facultatea de psihologie, Liceul Internat şi nu departe LiceulNaţional şi fosta ”Oltea Doamna”.În noul local din dealul Copoului, liceul are profilul real - umanist şi din 1967devine şcoală pilot pentru învăţarea limbilor străine, în speţă limba franceză, dinciclul primar. În 1977 îşi schimbă iar numele şi profilul, devenind Liceul IndustrialHidrotehnic. Dar pe fiecare an de studiu se păstrează o clasă de reală şi una defilologie. În 1990 revine la profilul real - umanist. Pe baza tradiţiei de studii clasice,la fiecare an de studiu a existat până în 2004 o clasă de limbi clasice (limba latină şigreaca veche). Actualmente liceul este teoretic şi are clase de informatică, ştiinţe alenaturii, filologie.Povestea Liceului ”Garabet Ibrăileanu” e povestea noastră a tuturor, mereu”sub vremuri”. Şi totuşi spiritul Liceului de aplicaţie, sămânţa bună semănată deacei dascăli de demult, cu dragoste de ţară şi de profesie nu a pierit şi aducerea înmemorie a unui secol de şcoală ne încurajează şi ne obligă.125Prof. Magda NEGREA


LIDIA COHAL(1930-2009)Evenimente a căror împletire este mai presus de dorinţe şi raţiuni ne obligă laoarece zăbavă. Pe lângă numeroasele, nobilele şi acaparantele obligaţii ale profesorilorde matematică, ni se relevă una mai rar evidenţiată, dar pe care o găsim plină desemnificaţii: activitatea în familia substanţial extinsă a matematicienilor.Lidia s-a născut la 30 octombrie 1930 în satul Ciutuleşti, judeţul Bălţi, de parteamai zbuciumată a Prutului. Absolvă şcoala primară în satul natal şi continuă primelepatru clase de liceu la Liceul ”Regina Maria” din oraşul Bălţi. Nu considerăm necesarsă explicăm fracturile sociale care au determinat ca ultimele patru clase să şi ledesfăşoare la Liceul ”Ecaterina Teodoroiu” din Târgu Jiu.Lidia devine studentă la Facultatea de matematică şi fizică la Universitatea ”Al.I. Cuza” Iaşi. Ca recunoaştere a meritelor de la începutul anului IV este numităpreparator. La absolvirea facultăţii, în anul 1952, a fost încadrată ca asistent universitarla Catedra de geometrie, bine reputată inclusiv la nivel internaţional. Împreunăcu colegele şi prietenele Alice Corduneanu şi Ligia Papuc (deja căsătorite cu distinşimatematicieni) se lansează în activităţi ştiinţifice şi didactice, devenind şi o harnicăsecretară a Filialei Iaşi a SSM.În septembrie 1959 i se desface contractul de muncă: originea socială, fiică depreot, era privită drept ”necorespunzătoare”. Conducerea SSM se dovedeşte maipuţin influenţabilă şi decide retribuirea fructuoasei sale activităţi de secretară defilială. Este locul să spunem că Societatea preluase aproape integral sarcinile deperfecţionare a profesorilor de matematică prin şcoli de vară; filiala ieşeană activăîn această directie extrem de susţinut şi materialele respective au fost incorporate înpatru volume de ”Probleme actuale de matematică”.În septembrie 1960 este încadrată şi în învăţământul liceal din Iaşi, păstrându-şişi atribuţiile de ”secretară de filială model”: extrem de precisă găsea mereu zâmbetulprietenesc stimulator.A funcţionat la liceele ieşene ”Naţional”, ”Emil Racoviţă” şi ”Mihai Eminescu”.Din perioada în care era încadrată ca asistent universitar, Lidia începuse documentareaşi lucrul la o carte destinată ”modernizării predării geometriei”. Coordonaeforturile profesorul Adolf Haimovici, preşedinte al filialei Iaşi şi mai colaborau prietenelemai sus numite Alice şi Ligia. Cartea Elemente de geometrie a planului aapărut abia în 1968 la Editura Didactică şi Pedagogică.Lidia s-a pensionat în septembrie 1987.De multe ori se omite amintirea unor realizări în pretenţioasa activitate de soţie;nu vom face aici această greşeală. Prin optimismul ei molipsitor şi prin tonusul eiexcelent, Lidia a potenţat neîntrerupt activitatea soţului ei, distinsul profesor ieşeanTraian Cohal.Unii oameni nu mor de tot.Prof.dr. Dan BRÂNZEIPreşedintele Filialei Iaşi a SSM126


Concursul de matematică “Al. Myller”Ediţia a VII-a, Iaşi, 28 martie 2009Clasa a VII-a1. Determinaţi n ∈ N ∗ cu proprietatea că n!+3·2 n = 6 n−2 , unde n! = 1·2·3 . . . n.Artur Bălăucă2. Fie triunghiul ABC şi punctul D situat pe latura [BC]. Arătaţi căAB · DC + AC · BD ≥ AD · BC.***3. Fie p un număr natural impar. Se ştie că oricare divizor al lui p are ultimacifră diferită de 3 şi 7. Să se arate că numărul 5p + 1 nu este pătrat perfect.Mircea Fianu4. Fie triunghiul echilateral ABC şi punctul D situat pe latura (AC). Bisectoareaunghiului ∠ABD intersectează paralela prin A la dreapta BC în punctul E. Arătaţică AE + DC = BD.Cristian LazărClasa a VIII-a1. Fie un tetraedru regulat cu muchia de lungime 3. Pe suprafaţa acestuia seconsideră 37 de puncte. Arătaţi că printre aceste puncte există două astfel încâtdistanţa dintre ele este cel mult egală cu 1.***2. Determinaţi perechile de numere (a, b) ∈ Z × Z care verifică egalitatea2(a + b) 2 + 3(a + b) + ab + 4 = 0.Petru Răducanu3. Fie a ≥ b ≥ c ≥ d ≥ 0 astfel încât a 2 + b 2 + c 2 + d 2 = 1. Să se arate căa + b ≥ 1 ≥ c + d.Gheorghe Iurea4. Numim piramidă Myller o piramidă SABCD cu baza ABCD, care are SA =SB = SC = SD, ∠ASB ≡ ∠ASD şi ∠BSC ≡ ∠DSC, iar lungimile SA, AB, BC,CD, DA, AC, BD sunt numere naturale nenule. Aflaţi piramida Myller de volumminim.Cristian LazărClasa a IX-a1. Determinaţi n ∈ N ∗ pentru care există o mulţime A ⊂ R cu n elemente, avândproprietatea a(b 3 + 6) ≤ b(a 3 + 6), ∀a, b ∈ A.Gheorghe Iurea127


2. Care este numărul minim de elemente care trebuie eliminate din mulţimea{1, 2, 3, . . . , 100} astfel încât în mulţimea rămasă să nu existe trei elemente x, y, zpentru care xy = z?***3. Fie triunghiul ABC şi punctele M ∈ (AB), N ∈ (AC) astfel încât BM +CN =MN + BC. Notăm ρ raza cercului înscris în triunghiul AMN. Arătaţi căρ( √ bc +È(p − b)(p − c)) ≤ r( √ bc −È(p − b)(p − c)).Dan Brânzei4. Numărul întreg m are proprietatea că, pentru orice număr natural n, existăa n ∈ Z astfel încât |nm − 80a n + 1| < 20. Arătaţi că 80 divide m.Dinu Şerbănescu| {z }n+2 cifreClasa a X-a1. Determinaţi valorile lui n ∈ N ∗ pentru care 41 are un multiplu de formaa00 . . . 0b unde a, b sunt cifre zecimale nenule.Mihai Bălună2. Fie ABC un triunghi, k ∈ R\{1} şi punctele M ∈ BC, N ∈ CA, P ∈ ABastfel încât MBMC = NCNA = P A = k. Se ştie că AM = BN = CP. Demonstraţi căP Btriunghiul ABC este echilateral.XI. V. Maftei3. Pentru x, y ∈ R definim f(x; y) = distanţa de la |x − y| la cel mai apropiatîntreg, iar pentru o mulţime finită M ⊂ [0, 1] definims(M) = f(x, y).x,y∈M,x


) Fie n un număr natural impar şi matricele A, B ∈ M n (C) astfel încât A 2 +B 2 =I n . Să se arate că det(AB − BA) = 0.Andrei Ciupan3. Fie P, Q : R → R două funcţii polinomiale neconstante cu proprietatea căexistă şi este finită ([P (x)] − [Q(x)]). Să se arate că există n ∈ Z astfel încâtlimx→∞P (x) − Q(x) = n, oricare ar fi x ∈ R.Gheorghe Iurea4. Fie A ∈ M n (R) o matrice simetrică cu elementele de pe diagonala principalăegale cu 1 şi cu suma modulelor elementelor de pe fiecare linie mai mică sau egală cu2. Să se arate că detA ≤ 1.Cosmin PohoaţăClasa a XII-a1. Fie f, g două funcţii polinomiale reale, de acelaşi grad, ambele având coeficientuldominant egal cu 1. Dacă g nu are rădăcini reale pozitive, calculaţinZ1lim xn→∞0f(nx)g(nx) − 1‹dx.Radu Gologan2. Fie A un inel. a) Arătaţi că dacă x ∈ A este nilpotent (există k ∈ N ∗ cux k = 0), atunci 1 + x este inversabil.b) Dacă A este finit, numărul elementelor inversabile este un număr prim iarelementele neinversabile sunt nilpotente şi numărul elementelor neinversabile este maimare sau egal cu numărul elementelor inversabile, arătaţi că A are 4 elemente.***3. Să se determine funcţiile continue f : R → R care verifică egalitatea f(arctg x) =(1 + x 2 )f(x), pentru orice x ∈ R.Al. Gabriel Mîrşanu4. Fie p, p > 2, un număr prim şi f un polinom cu coeficienţi întregi, de grad p−1,cu proprietatea că pentru orice a, b numere întregi pentru care p divide f(a) − f(b),rezultă că p divide a − b. Arătaţi că f are coeficientul dominant divizibil cu p.Marian AndronacheRăspunsul la ”recreaţia” de la pag. ... este:Numărul maxim de operaţii cu care se poate scrie 2009 este 4017 :2009 = 10 : 10 + 10 : 10 + · · · + 10 : 10 (2009 termeni).Numărul minim de operaţii este 209:2009 = (10 + 10 + · · · + 10) + (10 : 10 + 10 : 10 + · · · + 10 : 10)( 200 termeni în prima paranteză şi 9 în cea de-a doua).129


Concursul de matematică ”Florica T. Câmpan”Etapa judeţeană, 21 februarie 2009Clasa a IV-a (Subiect elaborat de Dumitru Pârâială şi Cătălin Budeanu)1. Fiica, tatăl şi bunica au împreună 90 de ani. Peste doi ani, tatăl va avea de optori vârsta fiicei, iar bunica de două ori vârsta actuală a tatălui. Aflaţi vârsta fiecăruiaîn prezent.2. Un blocnotes are 100 de pagini, numerotate de la 1 la 100. Se rup din acesta,la întamplare, 30 de foi, apoi se face suma numerelor ce marchează paginile rămase.Este posibil ca această sumă să fie egală cu 800? Justificaţi!3. O cantitate de 120 kg de prune a fost împărţită în mod egal în mai multelăzi. În vederea transportului, pentru a se evita pierderile, s-a transferat câte un sfertdin cantitatea de prune din fiecare ladă, obţinâdu-se astfel o nouă ladă cu tot atâteakilograme de prune câte au rămas în fiecare din lăzile iniţiale.a) Câte lăzi au fost la început?Justificaţi!b) Câte kilograme de prune erau iniţial în fiecare ladă?c) Câţi lei s-au încasat din vânzarea prunelor din lada nou formată, dacă unkilogram de prune de calitatea întâi costă 4 lei, un kilogram de prune de calitatea adoua costă 3 lei, iar la fiecare 2 kilograme de prune de calitatatea întâi există câte3kg de prune de calitatea a doua?Clasa a V-a (Subiect elaborat de Gheorghe Iurea şi Andrei Nedelcu)1. Notăm cu a 1 , a 2 , ....., a 2009 numerele 1, 2, . . . , 2009, scrise în altă ordine.Arătaţi că numărul N = (1 + a 1 ) · (2 + a 2 ) · . . . · (2009 + a 2009 ) este par. Este posibilca N să nu fie divizibil cu 4 ?2. Într-un pătrat 7 × 7 sunt scrise 49 numere naturale diferite. De pe fiecarelinie se consideră cel mai mic număr, iar cel mai mare dintre aceste şapte numere senotează cu x. De pe fiecare coloană se consideră cel mai mare număr, iar cel mai micdintre aceste şapte numere se notează cu y. Justificaţi că x ≤ y.3. a) Arătaţi că numărul 3 · 10 2009 − 8 nu este pătrat perfect.b) Arătaţi că numărul 22 1506 are cel puţin 2009 cifre.Clasa a VI-a (Subiect elaborat de Claudiu Ştefan Popa şi Ciprian Baghiu)1. a) Fie proporţia a b = c , cu toţi termenii numere naturale, astfel încât d esteddivizibil şi cu b, şi cu c, iar b şi c sunt prime între ele. Demonstraţi că bcd este pătratperfect.b) Dacă un extrem al unei proporţii cu toţi termenii numere naturale nenule estedivizibil cu fiecare din mezii ei, arătaţi că suma tuturor termenilor este divizibilă cucelălalt extrem.Claudiu Ştefan Popa130


2. Din cauza crizei economice, preţul unei mărfi se reduce săptămânal cu 50%,timp de n săptămâni, unde n ∈ N, n ≥ 2.a) Considerăm că preţul iniţial al mărfii este de 32 de lei. Ştiind că, atunci cândeste cazul, se procedează la rotunjirea preţului după o anumită reducere, pentru cael să poată fi plătit cu unităţile monetare existente şi că în a n-a săptămână preţulafişat este acelaşi cu cel din săptămâna n – 1, să se afle n.b) Presupunând acum că preţurile succesiv obţinute sunt exprimate fără rotunjireşi că preţul iniţial al mărfii este a lei, arataţi că suma între preţurile mărfii din primelen – 1 săptămâni şi dublul preţului mărfii din a n-a săptămână este a lei.Claudiu Ştefan Popa3. Considerăm triunghiul isoscel ABC, cu AB = AC şi m(∡BAC) < 90 0 . PuncteleM şi N sunt în aşa fel încât AB separă C şi M, AC separă B şi N, m(∡MAB) =m(∡NAC) = 90 0 , iar AM = AN = AB. Arătaţi că:a) MC = NB;b) MB ≠ NB;c) dacă, în plus, triunghiul MNB este isoscel, calculaţi m(∡BAC).Clasa a VII-a (Subiect elaborat de Vasile Nechita şi Ionel Nechifor)1. a) Fie a, b, c numere reale strict pozitive astfel încât 5a + 3c = 4b şi 2b= √ 15ac.Să se arate că numerele a, b, c sunt direct proporţionale cu numerele 6, 15 şi 10.b) Determinaţi x ∈ N pentru care √ x 2 + 9x + 13 ∈ N.2. a) Pe o masă sunt 20 monede. Radu şi Bogdan joacă următorul joc: ia fiecare,pe rând, de pe masă, câte 1, 2 sau 3 monede; câştigătorul este cel care ia ultimamonedă. Cine va câştiga jocul? (justificaţi!) Dar dacă numărul monedelor de pemasă este 21?b) Fiind dat un paralelogram ABCD şi o dreaptă d, care taie dreptele AB, BC,CD, DA în punctele M, N, P, respectiv Q, demonstraţi că MAMB · NBNC · P CP D · QDQA = 1.3. Andrei, Bogdan, Costel şi Dan cumpără trei torturi de îngheţată de formăpătrată, care au fiecare aceeaşi grosime şi compoziţie. Ştim că perimetrele torturilorsunt invers proporţionale cu numerele 1 36 , 1160 , 1, iar suma laturilor celor trei164torturi este 90cm. Explicaţi cum vor împărţi copiii cele trei torturi, pentru a luafiecare porţii egale (fără cântărire).Clasa a VIII-a (Subiect elaborat de Gabriel Popa şi Cristian Lazăr)1. a) Determinaţi numerele întregi a şi b cu proprietatea căa 2 b + ab 2 + a + b + 1 = 0.Recreaţii <strong>Matematice</strong>, 1/2000b) Fie n un număr natural dat. Determinaţi numerele reale strict pozitive x, y, zpentru care x n = yz, y n = zx şi z n = xy.Neculai Hârţan , Recreaţii <strong>Matematice</strong>, 2/20032. Despre un număr natural n vom spune că are proprietatea (P) dacă √ 24n + 1 ∈Q şi vom spune că are proprietatea (Q) dacă √ 24n + k ∈ R\Q, ∀k ∈ {2, 3, 4, ..., 99} .131


a) Daţi exemplu de un număr natural care are proprietatea (P), dar nu are proprietatea(Q).b) Daţi exemplu de un număr natural care are proprietatea (Q), dar nu are proprietatea(P).c) Daţi exemplu de un număr natural care nu are nici proprietatea (P), nici proprietatea(Q).d) Determinaţi (cu justificare) cel mai mic număr natural care are atât proprietatea(P), cât şi proprietatea (Q).e) Găsiţi toate numerele naturale care au atât proprietatea (P), cât şi proprietatea(Q).Cristian Lazăr3. Un zmeu din carton, care are forma unui triunghi ABC, lasă pe pământ oumbră având forma unui triunghi A ′ B ′ C ′ , care este asemenea cu triunghiul ABC.a) Dacă zmeul se află sub razele soarelui la amiază, demonstraţi că ∆ABC ≡∆A ′ B ′ C ′ .b) Dacă zmeul se află sub becul unui stâlp de iluminat, mai rezultă în mod necesarcă ∆ABC ≡ ∆A ′ B ′ C ′ ?Constantin Cocea, Gabriel PopaEtapa interjudeţeană, 21 martie 2009Clasa a IV-a (Subiect elaborat de Dan Brânzei şi Cătălin Budeanu)1. Aflaţi suma resturilor împărţirii a 2009 numere naturale consecutive la 15,ştiind că ultimul se împarte exact la 15.2. Suma a două numere naturale este 26. Împărţind primul număr la al doilea şiapoi pe al doilea la primul se obţine, de fiecare dată, aceeaşi sumă dintre cât şi rest,aceasta fiind cu 8 mai mică decât unul dintre numere. Aflaţi numerele.3. Fie numărul N = 510152025 . . . 725730735 . . . 20020052010.a) Câte cifre are numărul N?b) Care este a 1000-a cifră a lui N?4. (facultativ) Centrul unei pieţe are forma unui pătrat de latură 7 şi este ocupatde o statuie cu un soclu pătrat de latură 1. Se poate acoperi suprafaţa rămasă cupatru dreptunghiuri având dimensiunile 3 şi 4?Clasa a V-a (Subiect elaborat de Gheorghe Iurea şi Andrei Nedelcu)1. La un turneu de fotbal în sală participă patru echipe. Se acordă 2 punctepentru victorie, 1 punct pentru egal şi 0 puncte la înfrângere. Fiecare echipă joacă cufiecare câte un singur meci. În clasamentul final nu sunt două echipe la egalitate depuncte. Care este numărul minim de puncte pe care îl poate avea echipa câştigătoare?Realizaţi o distribuţie a rezultatului meciurilor care să corespundă situaţiei de maisus.132


2. Considerăm mulţimea A = {2 a · 3 b · 5 c |a, b, c ∈ N}. Arătaţi că printre oricare 9elemente ale mulţimii A există cel puţin două a căror produs este pătrat perfect.3. Un pătrat cu latura 5 se împarte în pătrate cu latura 1, care se numerotează cunumere de la 1 la 25. Se calculează sumele de pe fiecare linie şi de pe fiecare coloană.Există o numerotare astfel încât exact o sumă să fie număr par?4. (facultativ) Numărul 31 organizează o petrecere în împărăţia Numerelor Naturale.Dacă x ∈ N, atunci 9x – 2 şi 9x + 2 sunt părinţii lui x, iar 9x + 4 este bunicullui x. Se ştie că, dacă bunicul unui număr se află la petrecere, atunci este invitatşi nepotul său. De asemenea, dacă un număr este invitat la petrecere, atunci suntinvitaţi şi părinţii săi. Arătaţi că numărul 2009 este invitat la petrecere.Clasa a VI-a (Subiect elaborat de Ionel Nechifor şi Ciprian Baghiu)1. Să se arate că oricare ar fi m, n ∈ N, atunci m + 2267 n + 5388


) Fie O intersecţia diagonalelor patrulaterului convex ABCD. Demonstraţi că1AB|| CD dacă şi numai dacă = 1 + 1 .A AOD A ACD A ABDClaudiu Ştefan Popa3. Rezolvaţi în mulţimea numerelor întregi ecuaţia x 2 (y − 1) + y 2 (x − 1) = 1.Gabriel MîrşanuClasa a VIII-a (Subiect elaborat de Gabriel Popa şi Cristian Lazăr)1. Se dă mulţimea M = {1, 2, 3, 4} . Se formează toate sumele cu termeni distincţidin M, luând în seamă inclusiv sumele cu un singur termen. Două sume se considerăa fi distincte dacă diferă fie prin cel puţin un termen, fie prin ordinea termenilor (deexemplu, sumele 4, 1 + 3, 3 + 1, 1 + 2 + 3 sunt distincte).a) Câte sume diferite se pot forma?b) Arătaţi că orice număr cuprins între 1 şi 10 poate fi obţinut ca rezultat în urmacalculării unei astfel de sume.c) Două numere cuprinse între 1 şi 10 vor fi numite înrudite dacă se obţin de acelaşinumăr de ori ca rezultat al unor sume ca cele din enunţ. Determinaţi perechile denumere înrudite.Gabriel Popa2. Fie VABCD şi SABCD două piramide patrulatere regulate, având ca bazăcomună pătratul ABCD de latură 12 √ 2cm, vârfurile V şi S de o parte şi de alta aplanului (ABCD) şi în care înălţimile şi muchiile laterale se exprimă (în centimetri)prin numere întregi.a) Demonstraţi că punctele V, A, S şi C sunt coplanare.b) Aflaţi valorile posibile ale înălţimii VO a piramidei VABCD.c) Arătaţi că patrulaterul VASC este circumscriptibil.d) Dacă patrulaterul VASC este inscriptibil, determinaţi lungimea segmentuluiVS.Cristian Lazăr3. Trei greieri ”sar capra”: un greiere, aflat în punctul A, sare peste un alt greiere,aflat în B, şi ajunge în C, unde C este simetricul lui A faţă de B. Apoi, acelaşi greieresau un altul sare peste un partener de joacă şi tot aşa. Dacă iniţial cei trei greierise aflau în trei dintre vârfurile unui pătrat, se poate ca la un moment dat, în cursuljocului, unul dintre greieri să ajungă în cel de-al patrulea vârf al acelui pătrat?Mircea GangaVizitaţi noua pagina web a revistei:http://www.recreatiimatematice.ro134


Soluţiile problemelor propuse în nr. 2/2008Clasele primareP.154. Dorina are 15 baloane roşii şi albastre. Câte baloane roşii poate avea, dacănumărul acestora este mai mic decât numărul baloanelor albastre şi este cel puţin egalcu 3?(Clasa I )Inst. Maria Racu, IaşiSoluţie. Numărul baloanelor roşii poate fi 3, 4, 5, 6 sau 7.P.155. Dintr-o carte lipsesc câteva pagini, de la numărul 71 la numărul 94. Câtefoi lipsesc din această carte?(Clasa I )Ionela Bărăgan, elevă, IaşiSoluţie. Prima foaie care lipseşte are paginile 71 şi 72, iar ultima are paginile 93şi 94. În total lipsesc 12 foi.P.156. La concursul ”Desene pe asfalt”, elevii claselor I-IV de la Şcoala ”OtiliaCazimir” au acumulat 50 de puncte şi cel puţin 2 premii din fiecare categorie. Careeste cel mai mare număr de premii pe care-l pot obţine elevii, dacă pentru premiulI s-au acordat 10 puncte, pentru premiul al II-lea s-au acordat 6 puncte, iar pentrupremiul al III-lea s-au acordat 4 puncte?(Clasa a II-a)Înv. Elena Porfir, IaşiSoluţie. Numărul maxim de premii se obţine în cazul: 2 premii I, 3 premii II şi3 premii III, în total 8 premii.P.157. Prin golirea unui singur vas, ales dintre cele de mai jos, putem face carestul vaselor să aibă cantităţi egale de lichid. Care vas trebuie golit?(Clasa a II-a)Amalia Cantemir, elevă, IaşiSoluţie. Distribuim lichidul din vasul de 15 litri în primele 6 vase astfel: 5 l, 4 l,3 l, 2 l, 1 l, 0 l. În acest mod, în fiecare vas vom avea 14 litri.P.158. Aflaţi trei numere naturale ştiind că, adunându-le două câte două, obţinem100, 89, respectiv 141.(Clasa a III-a)Inst. Maria Racu, IaşiSoluţie. Obţinem că dublul sumei celor trei numere este 330, deci suma celor treinumere este 330 : 2 = 165. Astfel, primul număr este 165 − 100 = 65, al doilea este165 − 89 = 76, iar al treilea este 165 − 141 = 24.P.159. Se consideră numerele: a = 1 + 4 + 7 + 10 + · · · + 2008, b = 2 + 5 + 8++ · · · + 2009, c = 3 + 6 + 9 + · · · + 2010. Arătaţi că suma a + b + c se împarte exactla 3, fără să calculaţi această sumă.(Clasa a III-a)Iuliana Moldovan, elevă, Iaşi135


Soluţie. Putem scrie a + b + c = (1 + 2 + 3) + (4 + 5 + 6) + (7 + 8 + 9) + . . . +(2008 + 2009 + 2010). În fiecare paranteză avem suma a trei numere consecutive, carese împarte exact la 3, deci a + b + c se împarte exact la 3.P.160. Numărul a este de forma xy0, iar numărul b este de forma uv. Să se aflea şi b ştiind că a + b = 22 zeci.(Clasa a III-a)Dragoş Toma, elev, IaşiSoluţie. Condiţia din problemă se scrie xy0 + uv = 220, din care se deducev = 0, iar xy + u = 22. Pentru perechea (a, b) avem posibilităţile: (210, 10); (200, 20);(190, 30) . . . (130, 90).P.161. Fie a şi b două numere naturale astfel încât diferenţa lor este de 5 orimai mică decât suma lor. Să se arate că numărul cel mai mare se împarte exact la3, iar cel mai mic se împarte exact la 2.(Clasa a IV-a)Diana Tănăsoaie, elevă, IaşiSoluţie. Fie a şi b cele două numere. Din a + b = 5(a − b) obţinem 2a = 3b, decib trebuie să fie par. Înlocuind b = 2c, rezultă că a = 3c, prin urmare a se împarteexact la 3.P.162. Maria are 9 săculeţi cu monede. Cel puţin un săculeţ cântăreşte unkilogram. În orice grupare de 5 săculeţi, cel puţin 3 săculeţi au aceeaşi masă, iar înorice grupare de 6 săculeţi, cel mult 5 săculeţi au aceeaşi masă. Care este cel maimare număr de săculeţi de 1 kg pe care îl poate avea Maria?(Clasa a IV-a)Petru Asaftei, IaşiSoluţie. Nu putem avea mai mult de 5 săculeţi de 1 kg, deoarece am găsi ogrupare de 6 săculeţi de aceeaşi masă. Numărul maxim posibil de săculeţi de 1 kgeste 5 şi poate fi atins, de exemplu, dacă Maria are 5 săculeţi de 1 kg şi încă 4 de oaltă masă, aceeaşi, caz în care ar fi îndeplinită şi prima condiţie (conform principiuluicutiei).P.163. Jumătatea produsului a două numere naturale consecutive, împărţită cu3, nu poate da niciodată restul 2.Recreaţii Ştiinţifice, Anul I (1883)- nr. 4, p.119Soluţie. Dacă unul dintre cele două numere consecutive se împarte exact la3, restul împărţirii din enunţ este 0. În caz contrar, produsul celor două numereconsecutive, care este par, va da restul 2 la împărţirea prin 3, iar jumătatea se va dala împărţirea prin 3 restul 1.Clasa a V-aV.95. Două numere naturale se scriu în baza 10 folosind doar cifrele 1, 4, 6 şi 9.Poate fi unul dintre numere de 2008 ori mai mare decât celălalt?Cătălin Budeanu, IaşiSoluţie. Dacă numerele A şi B se scriu doar cu ajutorul cifrelor 1, 4, 6 şi 9, atunciU(2008A) ∈ {2, 8}, prin urmare 2008A ≠ B. Răspunsul la întrebarea din enunţ estenegativ.V.96. Determinaţi k, n ∈ N ∗ astfel încât (1 + 1 · n)+(2 + 2 · n)+· · ·+(k + k · n) =3 · 4 · 5 · 6.Petru Asaftei, Iaşi136


Soluţie. Deoarece (1 + 1 · n) + (2 + 2 · n) + . . . + (k + k · n) = (1 + 2 + . . . +k(k + 1)k) + n(1 + 2 + . . . + k) = (1 + 2 + . . . + k)(n + 1) şi 1 + 2 + . . . + k = ,2ecuaţia dată este echivalentă cu k(k + 1)(n + 1) = 2 · 3 · 4 · 5 · 6. Atunci k şi k + 1sunt divizori ai numărului 2 · 3 · 4 · 5 · 6, iar k(k + 1) ≤ 2 · 3 · 4 · 5 · 6. Găsim soluţiile(k, n) ∈ {(1, 359); (2, 119); (3, 59); (4, 35); (5, 23); (8, 9); (9, 7); (15, 2)}.V.97. Arătaţi că numărul N = 17 n + 21 n + 25 n , n ∈ N, nu poate fi pătrat perfect.Virginia Grigorescu, CraiovaSoluţie. Deoarece (Ma+1) n = Ma+1, deducem că 17 n = M4+1, 21 n = M4+1,25 n = M4 + 1, deci N = M4 + 3, prin urmare N nu poate fi pătrat perfect.V.98. Fie n ∈ N ∗ . Să se demonstreze că numărul N = 5050 . . . 505 (2n + 1 cifre)se scrie ca sumă a 4n + 2 pătrate perfecte distincte.Veronica Plăeşu şi Dan Plăeşu , Iaşi| {z }n cifreSoluţie. Observăm că N = 5·10 2n +5·10 2n−2 +. . .+5·10 2 +5. Cum 5 = 2 2 +1 2 ,iar 5 · 10 2k = 500 · 10 2k−2 = (16 2 + 12 2 + 8 2 + 6 2 ) · (10 k−1 ) 2 = (16 · 10 k−1 ) 2 + (12 ·10 k−1 ) 2 + (8 · 10 k−1 ) 2 + (6 · 10 k−1 ) 2 , cerinţa problemei este demonstrată.V.99. Se consideră numărul N = 1 + 11 + 101 + 1001 + · · · + 100 . . . 01.a) Pentru n ∈ N, n ≥ 5, arătaţi că 5 | N ⇔ 5 | n.b) Precizaţi care dintre propoziţiile ”3 | n ⇒ 3 | N” şi ”3 | N ⇒ 3 | n” esteadevărată pentru orice n ≥ 3.Temistocle Bîrsan, Iaşi| {z }ncifre| {z }n| {z }ncifre| {z }n−1 termeni| {z }ncifreSoluţie. a) Cum N = 1+10+100+. . .+100 . . . 0 + 1 + 1 + . . . + 1 = 11 . . . 11 +(n−1) = 11 . . . 10 +n. Cum 5| 11 . . . 10, rezultă cerinţa a).b) Niciuna dintre implicaţii nu este adevărată. Într-adevăr, pentru n = 3 avem căN = 1 + 11 + 101 = 113 . ./ 3. Apoi, pentru n = 5 avem că N = 1 + 11 + 101 + 1001 +10001 = 11115 . .3, în timp ce 5 . ./ 3.V.100. Determinaţi numerele naturale nenule a şi b pentru care există n ∈ Nastfel încât a b = 3n + 2 şi 3a + 2b < 100.7n + 5Gheorghe Iurea, IaşiSoluţie. Dacă d = (3n + 2, 7n + 5), atunci d|3n + 2 şi d|7n + 5, de unde d|3(7n +5) − 7(3n + 2), cu alte cuvinte d = 1, deci fracţia 3n + 2 este ireductibilă. Deoarece7n + 5ab = 3n + 27n + 5 , deducem că a = k(3n + 2) şi b = k(7n + 5), unde k ∈ N∗ , iar inegalitatea3a + 2b < 100 devine k(23n + 16) < 100. Pentru n = 0 obţinem k ∈ {1, 2, 3, 4, 5, 6},a = 2k şi b = 5k; pentru n = 1 avem k ∈ {1, 2}, a = 5k şi b = 12k. Dacă n = 2,atunci k = 1, a = 8, b = 19, iar pentru n = 3, k = 1, a = 11, b = 26. Pentru n ≥ 4,problema nu admite soluţii.137


V.101. Considerăm fracţia an + bcn + d , unde n, a, b, c, d ∈ N∗ , astfel încât b şi d auparităţi diferite, iar a şi c au aceeaşi paritate. Arătaţi că, dacă ad − bc = 2 k , k ∈ N ∗ ,atunci fracţia este ireductibilă.Cosmin Manea şi Dragoş Petrică, PiteştiSoluţie. Presupunem prin absurd că fracţia este reductibilă. Fie p un divizorcomun al numerelor an+b şi cn+d, p ∈ N ∗ , p ≥ 2; rezultă că p|(a·n+b) şi p|(c·n+d),de unde p|ad − bc, deci p|2 k şi atunci p este par. Totodată, p|n(a + c) + b + d şi cumn(a + c) + b + d este număr impar, rezultă că p este impar, contradicţie.Clasa a VI-aVI.95. Determinaţi numerele naturale nenule a 1 , a 2 ,. . . , a 2008 , ştiind că a 1a 21 · 2 =a 2 a 32 · 3 = · · · = a 2007a 20882007 · 2008 , iar a 1 + a 2008 = 2009.Gheorghe Iurea, IaşiSoluţie. Considerând rapoartele două câte două, obţinem că a 11 = a 33 = . . . =a 20072007 , iar a 22 = a 44 = . . . = a 20082008 . Astfel, relaţia a 1 + a 2008 = 2009 devine a 1 +1004a 2 = 2009 şi cum a 1 , a 2 ∈ N ∗ , atunci a 1 = 1, a 2 = 2. Deducem că a i = i,∀i = 1, 2008.VI.96. Determinaţi p ∈ N pentru care numerele p, p + 12, p + 22, p + 52, p + 72,p + 102 şi p + 132 sunt prime.Damian Marinescu, TârgovişteSoluţie. Pentru p = 7, numerele sunt 7, 19, 29, 59, 79, 109, 139 şi sunt toate prime.Dacă p ∈ {2, 3, 5}, obţinem numerele compuse 2 + 12, 3 + 12, respectiv 5 + 22. Dacăp ≥ 11, considerând p = M 7 + r, cu r ∈ {0, 1, . . . , 6}, găsim în fiecare caz câte unnumăr compus printre cele date. În concluzie, singura valoare convenabilă este p = 7.VI.97. a) Dacă a, b, c, d, e, f ∈ N ∗ sunt astfel încât (a, b) = (c, d) = (e, f) =(b, d) = 1, iar t = a b − c d + e ∈ N, arătaţi că f = bd.f4b) Determinaţi a, b ∈ N pentru care2a + 1 − 1 2 b + 7 6 ∈ N.Cosmin Manea şi Dragoş Petrică, PiteştiSoluţie. a) Deoarece t · bd = ad − bc + bd · ee∈ N, rezultă că bd · ∈ N şi, cumf f(e, f) = 1, deducem că f|bd. Pe de altă parte, din t · bf = af − bf · c + ed ∈ N şidt · df = df · ac a− cf + de ∈ N, rezultă că bf · şi df · sunt numere naturale. Folosindb d bfaptul că (b, a) = (d, b) = (d, c) = 1, obţinem că d|f şi b|f şi, cum (b, d) = 1, atuncibd|f. Din f|bd şi bd|f, rezultă că f = bd.b) Suntem în condiţiile punctului precedent; deducem că 6 = (2a + 1) · 2 b , de undea = b = 1. Pentru aceste valori, vom avea t = 2 ∈ N.VI.98. Determinaţi cel mai mic număr natural n cu proprietatea că numărulzerourilor în care se termină numărul (n + 10)! este cu 2008 mai mare decât numărulzerourilor în care se termină n! (unde n! = 1 · 2 · 3 · · · n).Cătălin Budeanu, Iaşi138


Soluţie. Ipoteza problemei revine la faptul că numărul (n + 1)(n + 2) . . . (n + 10)se termină în 2008 zerouri, ceea ce se întâmplă atunci când produsul (n + 1)(n +2) . . . (n + 10) se divide cu 5 2008 şi nu se divide cu 5 2009 . Printre factorii produsuluiprecedent, există exact doi care sunt divizibili cu 5, iar dintre aceştia unul nu mai estedivizibil cu nicio altă putere a lui 5, deci celălalt se va divide cu 5 2007 . Cum dorim nminim, atunci n + 10 = 5 2007 , prin urmare n = 5 2007 − 10.VI.99. Un patrulater convex are două laturi opuse congruente şi diagonalelecongruente. Arătaţi că patrulaterul este trapez isoscel sau dreptunghi.Ioan Săcăleanu, HârlăuSoluţie. Presupunem că (AB) ≡ (CD) şi (AC) ≡ (BD). Din congruenţa triunghiurilorABC şi DCB, rezultă căÕABC ≡ÕDCB, iar din congruenţa triunghiurilorABD şi DCA, rezultă căÕBAD ≡ÕCDA. Cum suma unghiurilor unui patrulater este360 ◦ , deducem că m(ÕABC) + m(ÕBAD) = 180 ◦ , deci AD∥BC.Dacă AB∥CD, atunci ABCD este un paralelogram cu diagonalele congruente,deci este un dreptunghi. Dacă AB ∦ CD, atunc ABCD este un trapez cu diagonalelecongruente, adică un trapez isoscel.VI.100. Fie △ABC cu m(bA) ≥ 90 ◦ . Să se arate că m(ÒB) = 2m(ÒC) dacă şinumai dacă există M ∈ [BC] astfel încât AB = AM = MC.Petru Asaftei, IaşiSoluţie. Presupunem că m(ÒB) = 2m(ÒC) şi fie M intersecţia mediatoarei laturii[AC] cu BC; cum m(bA) ≥ 90 ◦ , vom avea că M ∈ [BC]. AEvident atunci că △MAC este isoscel cu MA = MC şicumÖAMB este unghi exterior, deducem că m(ÖAMB) =2m(ÒC) = m(ÒB). Astfel, △ABM va fi isoscel cu AB =AM, ceea ce încheie justficarea afirmaţiei directe. B M CReciproc, dacă există M ∈ [BC] cu AB = AM = MC, din triunghiurile isosceleABM şi MAC obţinem că m(ÒB) = m(ÖAMB) = 2m(ÒC).VI.101. Fie ABC un triunghi dreptunghic cu m(bA) = 90 ◦ şi CD bisectoareaunghiuluiÒC, D ∈ (AB). Perpendiculara din D pe bisectoarea unghiuluiÒB intersecteazăipotenuza BC în E. Dacă P este punctul de intersecţie a bisectoarelor unghiurilortriunghiului ABC, iar M este punctul de intersecţie dintre EP şi AC, arătaţicăÖMP A ≡ÕP BE.Nela Ciceu, Bacău şi Titu Zvonaru, ComăneştiSoluţie. Din enunţ rezultă că triunghiul BDE este isoscel şi că BP este mediatoareasegmentului DE. Totodată, m(ÕCDE) = m(ÕCDB) − Cm(ÕEDB) = 180 ◦ − 1 2 m(ÒC) − m(ÒB) − 1 2 [180◦ − m(ÒB)] = 45 ◦ .Prin urmare, triunghiul DP E este dreptunghic isoscel. Astfel,CP ⊥ ME şi cum CP este bisectoarea unghiuluiÒC, rezultăcă triunghiul CME este isoscel. Deducem că m(ÖMP A) =180 ◦ − m(ÖP MA) − 45 ◦ = 135 ◦ − 180 ◦ + m(ÖCME) = −45 ◦ +12 [180◦ − m(ÒC)] = 1 2 [90◦ − m(ÒC)] = 1 2 m(ÒB) = m(ÕP BE).139MAPDEB


Clasa a VII-aA MVII.95. Fie ABCD pătrat, M un punct oarecare pe (AB),iar N ∈ (BC) este astfel încât MN ⊥ MD. Arătaţi că AM ·AB + CN · CB = DM 2 .Ovidiu Pop, Satu MareGh. Szöllösy, Sighetul MarmaţieiSoluţie. Fie a latura pătratului x = AM şi y = CN; atunciDDM 2 +MN 2 = DN 2 ⇔ a 2 +x 2 +(a−x) 2 +(a−y) 2 = a 2 +y 2 ⇔2a 2 − 2ax + 2x 2 − 2ay = 0 ⇔ a 2 + x 2 = ax + ay ⇔ DM 2 = AM · AB + CN · CB.VII.96. Fie [AD] mediană în △ABC, M mijlocul lui [AD],{E} = BM ∩ AC, iar punctul F pe dreapta AB este astfel încâtCF ∥ AD. Demonstraţi că punctele D, E şi F sunt coliniare.Mirela Marin, IaşiSoluţie. Cum ADCF este trapez, M este mijlocul bazei mici,iar B este punctul de intersecţie a laturilor neparalele, înseamnăcă {P } = BM ∩ CF este mijlocul lui [CF ]. Cu reciproca teoremeiliniei mijlocii se arată că A este mijlocul lui [BF ], prin urmareCA şi BP sunt mediane în △BCF , iar E va fi centrul de greutateal acestui triunghi.triunghiului este F D.BA.ME.DFBNC. PColiniaritatea dorită rezultă observând că a treia mediană aVII.97. Fie C 1 (O 1 , r 1 ) şi C 2 (O 2 , r 2 ), r 1 < r 2 , două cercuri tangente exterior.Considerăm punctele A ′ ∈ C 1 , B ′ ∈ C 2 , de aceeaşi parte a dreptei O 1 O 2 , astfel încâtA ′ O 1 ∥ B ′ O 2 . Dacă AB este tangentă comună exterioară a cercurilor (A ∈ C 1 ,B ∈ C 2 ), demonstraţi că dreptele AB, A ′ B ′ şi O 1 O 2 sunt concurente.Romanţa Ghiţă şi Ioan Ghiţă, BlajSoluţie. Notăm {P } = AB ∩ O 1 O 2 , {P ′ } = A ′ B ′ ∩ O 1 O 2 . Din asemănările△P O 1 A ∼ △P O 2 B şi △P ′ O 1 A ′ ∼ △P ′ O 2 B ′ obţinem că P O 1= P ′ O 1P O 2 P ′ = r 1< 1.O 2 r 2BAO 2O 1B A P. .Astfel, punctele P şi P ′ se află de aceeaşi parte cu O 1 pe dreapta O 1 O 2 şi împartsegmentul [O 1 O 2 ] în acelaşi raport, prin urmare P = P ′ şi de aici rezultă concurenţadorită.VII.98. Să se determine numerele naturale nenule a şi b, ştiind că sunt directproporţionale cu b − 6 şi a şi invers proporţionale cu a + 12 şi b.Constantin Apostol, Rm. SărataSoluţie. Din datele problemei obţinem căb − 6 = b a şi a(a+12) = b2 . Din primaecuaţie rezultă că a 2 − b 2 = −6b, iar din a doua a 2 − b 2 = −12a, deci −6b = −12a.140PC


Atunci b = 2a şi, folosind prima ecuaţie, găsim că a = 4 şi apoi b = 8.VII.99. Fie a, b ∈ Z şi numerele A = 119a 5 + 5b 3 − 4a şi B = 119b 5 + 5a 3 − 4b.Să se arate că A se divide cu 120 dacă şi numai dacă B se divide cu 120.Dan Nedeianu, Drobeta-Tr. SeverinSoluţie. Avem că A + B = 120(a 5 + b 5 ) − (a 5 − 5a 3 + 4a) − (b 5 − 5b 3 + 4b). Cuma 5 − 5a 2 + 4a = (a − 2)(a − 1)a(a + 1)(a + 2) se divide la 120, rezultă că A + B . .120şi concluzia de impune.VII.100. Arătaţi că 2a 2 + 15b 2 + 7c 2 ≥ 10ab − 6ac + 20bc, ∀a, b, c ∈ R.Alexandru Negrescu, student, IaşiSoluţie. Inegalitatea se scrie: 4a 2 − 4a(5b − 3c) + 30b 2 + 14c 2 − 40bc ≥ 0 ⇔(2a − 5b + 3c) 2 + 30b 2 + 14c 2 − 40bc − (5b − 3c) 2 ≥ 0 ⇔ (2a − 5b + 3c) 2 + 5(b − c) 2 ≥ 0,evident adevărat. Egalitate avem pentru a = b = c.VII.101. Pentru n ∈ N ∗ , notăm cu d (n) numărul divizorilor primi ai lui n.a) Determinaţi cardinalul mulţimii A = {n ∈ N ∗ | n ≤ 208, d (n) = 3}.b) Aflaţi cel mai mic şi cel mai mare element al mulţimiiB = {k ∈ N | ∃n ∈ N ∗ , n ≤ 2008, a.î. d (n) = k} .Gabriel Popa, IaşiSoluţie. a) Observăm că n ∈ A dacă şi numai dacă n = p α 11 · p α 22 · p α 33 , cup 1 < p 2 < p 3 numere prime, iar α 1 , α 2 , α 3 ∈ N ∗ . Cum 5 · 7 · 11 = 385 > 208, înseamnăcă p 1 ∈ {2, 3}, prin urmare(p 1 , p 2 , p 3 ) ∈ {(2, 3, 5); (2, 3, 7); (2, 3, 11); (2, 3, 13); (2, 3, 17); (2, 3, 19); (2, 3, 23);(2, 3, 29); (2, 3, 31); (2, 5, 7); (2, 5, 11); (2, 5, 13); (2, 5, 17); (2, 5, 19);(2, 7, 11); (2, 7, 13); (3, 5, 7); (3, 5, 11); (3, 5, 13)}.Pentru tripletele subliniate, în produsul n = p α 11 · p α 22 · p α 33 vom avea obligatoriuα 1 = α 2 = α 3 = 1; obţinem astfel 13 elemente ale lui A.Dacă (p 1 , p 2 , p 3 ) = (2, 3, 5), în n = 2 α1 · 3 α2 · 5 α3 putem avea α 3 = 1, (α 1 , α 2 ) ∈{(1, 1); (2, 1); (3, 1); (1, 2); (2, 2)} sau α 3 = 2, α 1 = α 2 = 1; găsim încă 6 elemente dinA. Dacă (p 1 , p 2 , p 3 ) = (2, 3, 7), în n = 2 α1 · 3 α2 · 7 α3 este obligatoriu ca α 3 = 1, iar(α 1 , α 2 ) ∈ {(1, 1); (2, 1); (3, 1); (1, 2)}; obţinem 4 noi elemente ale lui A.Dacă (p 1 , p 2 , p 3 ) = (2, 3, 11), atunci α 3 = 1, (α 1 , α 2 ) ∈ {(1, 1); (2, 1); (1, 2)}, deciîncă 3 elemente. Dacă (p 1 , p 2 , p 3 ) ∈ {(2, 3, 13); (2, 3, 17); (2, 5, 7)}, vom avea în fiecarecaz (α 1 , α 2 , α 3 ) ∈ {(1, 1, 1); (2, 1, 1)}, adică încă 3 × 2 = 6 elemente.În total, |A| = 13 + 6 + 4 + 3 + 6 = 32.b) Evident că min B = 0, atins pentru n = 1.Arătăm că max B = 4: dacă ar exista n ≤ 2008 cu d(n) ≥ 5, atunci n ≥p 1 p 2 p 3 p 4 p 5 ≥ 2 · 3 · 5 · 7 · 11 = 2310 > 2008, absurd, iar pentru n = 2 · 3 · 5 · 7avem d(n) = 4.Clasa a VIII-aVIII.95. Pentru a, b, c ∈ R ∗ , notăm α = a b + b c + c a , β = a c + c b + b a . Calculaţinumărul x = a3b 3 + b3c 3 + c3în funcţie α şi β.a3 ElenaNicu, Malu-Mare (Dolj)141


Soluţie. Fie x = a b , y = b c , z = c ; atunci xyz = 1, x + y + z = α, iaraxy + yz + zx= 1 xyz x + 1 y + 1 = β, de unde xy + xz + yz = β. Folosind identitateazx 3 + y 3 + z 3 − 3xyz = (x + y + z) 3 − 3(x + y + z)(xy + xz + yz),rezultă că x 3 + y 3 + z 3 = α 3 − 3αβ + 3.VIII.96. Rezolvaţi în numere naturale ecuaţia x 2 + y 2 + xy = x 2 y 2 .Mihail Bencze, BraşovSoluţie. Putem presupune că x ≤ y. Dacă x = 0, obţinem că y = 0. Dacă x = 1,deducem că y = −1 /∈ N. Pentru x = 2, găsim că 3y 2 − 2y − 4 = 0, ecuaţie care nu aresoluţii naturale. În cazul în care x ≥ 3, atunci y ≥ 3 şi vom avea că x2 − 1 > x + 1,y 2 −1 > y+1, inegalităţi care, înmulţite, conduc la (x 2 −1)(y 2 −1) > (x+1)(y+1). Însă(x+1)(y+1) > xy+1, prin urmare (x 2 −1)(y 2 −1) > xy+1, adică x 2 +y 2 +xy < x 2 y 2 .În concluzie, singura soluţie a ecuaţiei date este (x, y) = (0, 0).Soluţia 2. Vom rezolva ecuaţia în mulţimea numerelor întregi. Fie (x, y) o soluţie.Scriind ecuaţia sub forma (x + y) 2 = xy(xy + 1), deducem că xy(xy + 1) este pătratperfect. Cum pentru xy ≥ 1, (xy) 2 < xy(xy + 1) < (xy + 1) 2 , iar pentru xy ≤ −2(xy + 1) 2 < xy(xy + 1) < (xy) 2 , nu avem soluţii în aceste cazuri. Rezultă că xy = 0sau xy = −1. Obţinem soluţiile: (0, 0); (1, −1) şi (−1, 1).Soluţia 3 (Gheorghe Iurea). Fie (x, y) o soluţie cu x, y ∈ Z. Ecuaţia esteechivalentă cu (x + y) 2 = (xy) 2 + xy ⇔ 4(x + y) 2 = (2xy + 1) 2 − 1 ⇔ [2xy + 1 − 2(x +y)][2xy + 1 + 2(x + y)] = 1, deci 2xy + 1 − 2(x + y) = 2xy + 1 + 2(x + y) ∈ {−1, 1}.Obţinem soluţiile (0, 0); (1, −1) şi (−1, 1).VIII.97.Fie d 1 , d 2 , d 3 , d lungimile diagonalelor feţelor, respectiv diagonaleiunui paralelipiped dreptunghic. Dacă d 2 1 = 2d2 2d 2 3d 2 2 + , să se arate că paralelipipedul ared2 3o muchie de lungime cel puţin egală cu d√ 33 . Gheorghe Molea, Curtea de ArgeşSoluţie. Fie d 2 1 = a 2 + b 2 , d 2 2 = b 2 + c 2 , d 2 3 = a 2 + c 2 , d 2 = a 2 + b 2 + c 2 , unde a, b, csunt lungimile muchiilor paralelipipedului. Relaţia din ipoteză se scrie succesiv:d 2 1 = 2d2 2d 2 3d 2 2 + ⇔ a 2 + b 2 = 2(b2 + c 2 )(c 2 + a 2 )d2 3b 2 + c 2 + c 2 + a 2 ⇔ a 4 + b 4 = 2c 4 .Însă 2(a 4 + b 4 ) ≥ (a 2 + b 2 ) 2 , deci 4c 4 ≥ (a 2 + b 2 ) 2 ⇔ 2c 2 ≥ a 2 + b 2 ⇔ 3c 2 ≥a 2 + b 2 + c 2 ⇔ 3c 2 ≥ d 2 , deci c ≥ d√ 3Û. Avem egalitate pentru a = b = c, deci într-un3Ûcub.VIII.98. Fie VABCD piramidă patrulateră regulată. Notăm u=m(Û(V BC),(ABC)),v = m( (V BC) , (V CD)) şi t = m( (V BC) , (V AD)). Arătaţi că u + v + t > 180 ◦ .Claudiu Ştefan Popa, IaşiSoluţie. Fie M şi N mijloacele laturilor [BC], respectiv [AD], iar P proiecţia142


lui B pe V C; se arată că u = m(ÖV MN), v = m(ÕBP D), iart = m(ÖMV N). În △V MN avem că u + u + t = 180 ◦şi atunci concluzia problemei rezultă dacă am arăta căv > u. Evident că u < 90 ◦ , prin urmare dacă v ≥ 90 ◦ ,demonstraţia este încheiată. Presupunem că v < 90 ◦ ;atunci v > u ⇔ sin v > sin u ⇔ BD · P OBP 2P O = V O · BDV M · BC, iar BP => V OV M . Cum, ultima inegali-2V CV Ctate revine laBD2 · V C2BC 2 · V M > 1. Însă BD2 = 2BC 2 , iarV C > V M şi astfel soluţia problemei este completă.A.NV..PD.CO MBVIII.99. Pentru n ∈ N ∗ , considerăm A =1 2 , 2 2 , 3 2 , . . . , n 2 . Determinaţi n,ştiind că există o funcţie f : A → A astfel încât f (x) − f (y) = √ x − √ y, ∀x, y ∈ A.Cristian Lazăr, IaşiSoluţie. Avem că f(x) − √ x = f(y) − √ y, ∀x, y ∈ A, deci ∃k ∈ R astfel încâtf(x) − √ x = k, ∀x ∈ A, de unde f(x) = √ x + k, ∀x ∈ A. Deducem că, pentrux < y din A, avem că f(x) < f(y) şi atunci, cum A este finită, rezultă că f(1 2 ) = 1 2 ,f(2 2 ) = 2 2 , . . . , f(n 2 ) = n 2 , de unde 1 + k = 1 şi n + k = n 2 . Deci, k = 0 şi n = 1.VIII.100. Rezolvaţi în în N 2 ecuaţia x 2 − 8 n + 1287 = 0.Mihai Crăciun, PaşcaniSoluţie. Dacă n este impar, atunci 8 n = (9 − 1) n = M 9 − 1. Cum x 2 dă laîmpărţirea prin 9 unul dintre resturile 0, 1, 4 sau 7, iar 1287 . .9, cantitatea din membrulstâng al ecuaţiei dă la împărţirea prin 9 unul dintre resturile 1, 2, 5 sau 8 şi se ajungela o contradicţie.Dacă n = 2k, ecuaţia se scrie: (8 k − x)(8 k + x) = 3 2 · 11 · 13. Analizând pe rândcazurile care se obţin, găsim soluţie doar când 8 k − x = 11, 8 k + x = 3 2 · 13, când vomavea k = 2, x = 53. Deci, singura soluţie a ecuaţiei date este (n, x) = (4, 53).VIII.101. Se calculează suma cifrelor pentru fiecare dintre numerele de la 1la n, n > 10. Pentru fiecare sumă dintre cele n se calculează din nou suma cifrelor,repetându-se această operaţie până când obţinem n numere formate din câte o singurăcifră. Să se afle n, ştiind că în mulţimea astfel obţinută cifrele 1, 2, 3 şi 4 se repetăde câte 101 ori fiecare, iar cifrele 5, 6, 7, 8 şi 9 de câte 100 ori fiecare.Mihai Haivas, IaşiSoluţie. Restul împărţirii unui număr prin 9 este egal cu restul împărţirii sumeicifrelor sale prin 9. Atunci, cifra 1 se obţine din acele numere care dau restul 1la împărţirea prin 9, adică din numerele 1, 10, 19 . . . . Pentru a obţine 1 de 101 ori,trebuie să avemhn9i+1 = 101, deci n ∈ {900, 901, . . . , 904}. Se verifică faptul cădoar n = 904 satisface şi celelalte condiţii din ipoteză.Clasa a IX-aIX.91. Fie a, b, c, p ∈ R, p > 0. Dacăax 2 + bx + c≤p, ∀x ∈ [−1, 1], atuncicx 2 + bx + a≤2p, ∀x ∈ [−1, 1].Dorin Mărghidanu, Corabia143


Soluţie. Fie f(x) = ax 2 + bx + c, g(x) = cx 2 + bx + a, x ∈ [−1, 1]. Din ipotezăse obţine că |a + b + c| = |f(1)| ≤ p, |a − b + c| = |f(−1)| ≤ p, iar |c| = |f(0)| ≤ p.Folosind inegalitatea modulului, avem:|g(x)| = |cx 2 + bx + a| = |c(x 2 − 1) + (a + b + c) · x + 1 + (a − b + c) · 1 − x | ≤22≤ |c| · |x 2 |x + 1||1 − x|− 1| + |a + b + c| · + |a − b + c| · ≤22≤ p · |x 2 |x + 1| |1 − x|− 1| + +‹=p 1 − x 2 + x + 1 + 1 − x ‹=2 2 2 2= p(2 − x 2 ) ≤ 2p, ∀x ∈ [−1, 1].IX.92.(1 + α) · · · (n + α)n + α + βFie n ∈ N, n ≥ 3, iar α, β ∈ R astfel încât n + α + β ≠ 0. Arătaţi căn−2− (1 + α) · · · (n − 1 + α) + (−1) i+1 (1 + α) · · · (n − i − 1 + α)××β (β + 1) · · · (β + i − 1) + (−1) n β · · · (β + n − 2) = (−1) n β (β + 1) · · · (β + n − 1).n + α + βGheorghe Costovici, IaşiSoluţie. Notăm cu S n membrul stâng al egalităţii. Pentru n = 3, avem căS 3 =Xi=1(1 + α)(2 + α)(3 + α)− (1 + α)(2 + α) + (1 + α) · β −3 + α + ββ(β + 1)(β + 2)−β(β + 1) = . . . = − ,3 + α + βcalculele fiind de rutină. Fie n ≥ 4, iar σ n−2 = S n − (−1) n β(β + 1) . . . (β + n − 2).Ar fi suficient să dovedim că(∗) σ k = (−1) k−1 1(1+α) . . . (n−k−1+α)β(β+1) . . . (β+k)· , ∀k ∈ 1, n − 2n + α + βodată demonstrată (∗), vom obţine căS n = σ n−2 + (−1) n β(β + 1) . . . (β + n − 2) == (−1) n β(β + 1) . . . (β + n − 2) 1 − 1 + αn + α + β‹== (−1) n ·adică tocmai ceea ce trebuia demonstrat.Justficăm (∗) prin inducţie după k:β(β + 1) . . . (β + n − 1),n + α + βσ 1 =(1 + α) . . . (n + α)− (1 + α) . . . (n − 1 + α) + (1 + α) . . . (n − 2 + α)β =n + α + β=−β(1 + α) . . . (n − 1 + α) · + (1 + α) . . . (n − 2 + β) · β =n + α + β=(1 + α) . . . (n − 2 + α)β(β + 1),n + α + β144


deci (∗) este adevărată pentru k = 1. Presupunem (∗) adevărată pentru k ∈ {1, 2, . . . , n−3}; atunciσ k+1 = σ k + (−1) k (1 + α) . . . (n − k − 2 + α)β(β + 1) · . . . · (β + k) == (−1) k (1 + α) . . . (n − k − 2 + α)β · . . . · (β + k) 1 − n − k − 1 + αn + α + β‹== (−1) k (1 + α) . . . (n − k − 2 + α)β · . . . · (β + k + 1) ·1n + α + β ,deci (∗) este adevărată şi pentru k + 1 şi astfel soluţia problemei este completă.IX.93. Fie △ABC dreptunghic cu m(bA) = 90 ◦ şi ABAC = 3 , iar D mijlocul lui2[AC]. Notăm cu E punctul de intersecţie a cercurilor C 1 (A, AD) şi C 2 (B, BC), aflatde aceeaşi parte a dreptei AB ca şi punctul C. Determinaţi măsura unghiuluiÕCAE.Cătălin Ţigăeru, SuceavaSoluţie. Fie {M, N} = AB ∩ C 1 , cu N ∈ (AB), iar a = AD; atunci AC = 2a,AB = 3a, BC = a √ 13, BD = a √ 10, iar BM = 4a. CCum BD < BC < BM, înseamnă că cercurile C 1 şiC 2 sunt secante, iar 90ED..◦ < m(ÕBAE) < 180 ◦ . Aplicămteorema cosinusului în △ABE : BE 2 = AB 2 +AE 2 −2AB · AE · cos(ÕBAE) ⇔ 13a. .2 = 9a 2 + a 2 − 2 · 3a · a ·cos(ÕBAE), de unde obţinem cos(ÕBAE) = − 1 M A N B2 , adicăm(ÕBAE) = 120 ◦ . Deducem că m(ÕCAE) = 120 ◦ − 90 ◦ = 30 ◦ .IX.94. În △ABC, I este centrul cercului înscris, iar {M} = AI∩BC. Demonstraţică bisectoarea unghiuluiÖAMC, BI şi AC sunt trei drepte concurente dacă şi numaidacă m(bA) = 120 ◦ .Vlad Emanuel, student şi Andrei Cozma, elev, BucureţiSoluţie. Notăm cu a, b, c lungimile laturilor şi fie {P } = BI ∩ AC; avem căAM =2bcAb + c·cos A 2 , MC = ab AP, iarb + c P C = c a . Atunci:BI, AC şi bisectoarea unghiuluiÖAMC sunt concurenteP⇔ MP este bisectoarea luiÖAMC ⇔ APIP C = AMMC ⇔ca = 2ca cos A 2 ⇔ cos A 2 = 1 2 ⇔ m(bA) = 120 ◦ .B MCIX.95. Dacă x i ∈ [0, a], i = 1, n şi x n+1 = x 1 , demonstraţi căx i+1 (a − x i )


(1aria poligonului, atunciObţinem cănXi=12 sin π n cos π n · 12 ·de unde rezultă concluzia problemei.Clasa a X-aX.91. Arătaţi căS i < S, iar S i = x i+1(a − x i )· sin 2π 2 n , S = na24 sin π · cos π n .nnXi=1x i+1 (a − x i ) < na24 sin π n· cos π n ,arctg 1 72+arctg 3 42+π 2162= 2arctg 1 74+arctg 3 44+π 4256.D. M. Bătineţu-Giurgiu, BucureştiSoluţie. Se demonstrează relativ simplu identitatea[x 2 + y 2 + (x + y) 2 ] 2 = 2[x 4 + y 4 + (x + y) 4 ],cunoscută sub numele de identitatea lui G. Candido. Concluzia problemei rezultăobservând că, dacă x = arctg 1 7 şi y = arctg 3 tg x + tg y, atunci tg(x + y) =4 1 − tg x tg y = 1,deci x + y = π 4 (deoarece x + y < π 2 ).X.92. Fie a, b ∈ C. Demonstraţi că ecuaţia z 2 − az + b = 0 are ambele soluţii demodul 1 dacă şi numai dacă |b| = 1 şi |a| 2 +a 2 − 4b=4. (În legătură cu X.77 dinRecMat - 1/2007.)Marian Tetiva, BârladSoluţie. Putem proceda ca în soluţia problemei X.77, care poate fi consultatăîn RecMat 1/2008, pp. 65. În cele ce urmează, vom prezenta o soluţie carefoloseşte scrierea trigonometrică a numerelor complexe. Presupunem întâi că ecuaţiaare soluţiile z 1 , z 2 de modul 1, deci z k = cos t k + i sin t k , t k ∈ [0, 2π), k = 1, 2. Atuncia = z 1 + z 2 = 2 cos t 1 − t 22cos t 1 + t 22+ i sin t 1 + t 22‹, b = z 1 z 2 = cos(t 1 + t 2 ) +i sin(t 1 + t 2 ), iar a 2 − 4b = −4 sin 2 t 1 − t 2(cos(t 1 + t 2 ) + i sin(t 1 + t 2 )). Evident că2|b| = 1, iar |a| 2 + |a 2 − 4b| = 4 cos 2 t 1 − t 2+ sin 2 t 1 − t 22‹=4.2Reciproc, fie |b| = 1 şi |a| 2 + |a 2 − 4b| = 4. Atunci |a 2 | + |4b − a 2 | = 4 = 4|b| =|a 2 + 4b − a 2 |, prin urmare există t ≥ 0=Êastfel încât 4b − a 2 = ta 2 sau avem a 2 = 0. Înal doilea caz, ecuaţia devine z 2 + b = 0 şi, cum |b| = 1, ambele soluţii vor fi de modul1. În primul caz, discriminantul este ∆ = a2 − 4b = −ta 2 , de unde z 1,2 = a · 1 ± i√ t,2+ t + t)acu modulul |z 1 | = |z 2 | = |a| ·É1 2=È|b| = 1, ceea ce trebuia44demonstrat.146


X.93. Dacă a 1 , a 2 , . . . , a n ∈(0, 1) sau a 1 , a 2 , . . . , a n ∈(1, ∞), iar f, g : {1, 2, . . . , n}→ {1, 2, . . . , n} sunt funcţii injective, să se arate cănXk=1log aka f(k)a g(k)‹ nXk=1a k‹≥n 2 .Dan Popescu, SuceavaÏnQSoluţie. Se înmulţesc membru cu membru următoarele două inegalităţi, carelog aka f(k)rezultă din inegalitatea mediilor: a k ≥ n ·nÌnYk=1a k şi≥a gknXk=1nXk=1log aka f(k)nÊnQnÊnQn· n k=1nn== (am ţinut seama de faptul că funcţiilea g(k) a g(k) a knQl=1f şi g sunt chiar bijective).k=1k=1X.94. a) Să se arate căpx 2n + y 2n + x n y n + √ x 2n + z 2n + x n z n ≥py 2n + z 2n + y n z n , ∀x, y, z ∈R ∗ , ∀n∈N.b) Demonstraţi că, dacă n este par, inegalitatea este strictă, iar dacă n este impar,atunci există x, y, z ∈ R pentru care se atinge egalitatea.Bogdan Victor Grigoriu, FălticeniSoluţie. a) Considerând în plan punctele A(x n , 0), B‚− yn 2 , yn√ 32ŒşiC‚− zn 2 , −zn√ 3Œ, observăm că AB =px2 2n + y 2n + x n y n , AC = √ x 2n + z 2n + x n z n ,iar BC =py 2n + z 2n + y n z n . Concluzia rezultă aplicând inegalitatea triunghiului.b) Dacă n este par, atunci numerele x n , y n şi z n sunt strict pozitive, prin urmareB se va situa în cadranul II, iar C în cadranul III, în timp ce A se va afla pe semiaxapozitivă Ox. În acest caz, punctele A, B şi C nu pot fi coliniare, prin urmare inegalitateade la a) este strictă. Dacă n este impar, egalitatea se atinge, de exemplu,pentru x = 1, y = z = − n√ 2.X.95. Considerăm funcţia f : R 3 → R,f (x, y, z) = sin x + sin y + sin z + sin (x − y) + sin (y − z) + sin (z − x) .Determinaţi maximul şi minimul funcţiei f.Cătălin Calistru, IaşiSoluţie. Cu substituţiile x = x 1 + x 2 , y = x 2 + x 3 , z = x 3 + x 1 , avem:f(x, y, z) = sin(x 1 + x 2 ) + sin(x 2 + x 3 ) + sin(x 3 + x 1 ) + sin(x 1 − x 3 )++ sin(x 2 − x 1 ) + sin(x 3 − x 2 ) = 2 sin x 1 cos x 3 + 2 sin x 2 cos x 1 + 2 sin x 3 cos x 2 .Prin urmare, 3+f(x, y, z) = (sin x 1 +cos x 3 ) 2 +(sin x 2 +cos x 1 ) 2 +(sin x 2 +cos x 2 ) 2 ≥ 0şi 3 − f(x, y, z) = (sin x 1 − cos x 3 ) 2 + (sin x 2 − cos x 1 ) 2 + (sin x 3 − cos x 2 ) 2 ≥ 0, deci147


−3 ≤ f(x, y, z) ≤ 3, ∀(x, y, z) ∈ R 3 . Cum f( π 2 , π 2 , π 2 ) = 3, iar f(−π 2 , −π 2 , −π 2 ) = −3,rezultă că max f = 3 şi min f = −3.Clasa a XI-aXI.91. Fie matricele A, B, C, D ∈ M n (R) astfel încât AC + BD = I n , iarAD = BC. Demonstraţi că CA + DB = I n şi DA = CB.I. V. Maftei, Bucureşti şi Mihai Haivas, IaşiSoluţie. Din datele problemei obţinem că (A−iB)(C+iD) = I n ; atunci matriceleA − iB şi C + iD sunt una inversa celeilalte, prin urmare (C + iD)(A − iB) = I n .Deducem că (CA + DB) + i(DA − CB) = I n şi, cum CA + DB, DA − CB ∈ M n (R),atunci CA + DB = I n şi DA − CB = O n .XI.92. Determinaţi matricele X ∈ M 2 (R) pentru care X 2 + X = 1 11 1‹.Adrian Reisner, ParisSoluţie. Fie X = a bc d‹∈M 2 (R); atunci X 2 = (a + d)X − (ad − bc)I 2 şiecuaţia devine X(a + d + 1) − (ad − bc)I 2 = 1 1 Obţinem sistemul:1 1‹.8>:a(a + d + 1) − (ad − bc) = 1 (1)b(a + d + 1) = 1 (2)c(a + d + 1) = 1 (3)d(a + d + 1) − (ad − bc) = 1 (4)Din ecuaţiile (1) şi (4) rezultă că (a + d + 1)(a − d) = 0 şi cum a + d + 1 ≠ 0,1atunci d = a. Din ecuaţiile (2) şi (3) obţinem că b = c =a + d + 1 = 12a + 1 .Folosind acum ecuaţia (1), avem că a(2a + 1) − (a 2 1− ) = 1, echivalent cu(2a + 1)2a(a + 1)(4a 2 + 4a − 3) = 0, deci a ∈ {− 3 2 , 1 , −1, 0}. Găsim soluţiile2X 1 = 0 11 0‹, X 2 =−1 −1−1 −1‹, X 3 = 1 21 11 1‹, X 4 = − 1 23 11 3‹.XI.93. Studiaţi convergenţa şirului (u n ) n≥1definit prin u 1 ≥ 0, u n+1 = u n + 1u 2 n + 1 ,∀n ∈ N ∗ .Gheorghe Costovici şi Adrian Corduneanu, IaşiSoluţie. Vom arăta că (u n ) este convergent către 1. Dacă u 1 ∈ {0, 1}, afirmaţiaeste imediată. Cazurile u 1 ∈ (0, 1) şi u 1 ∈ (1, ∞) tratându-se asemănător, ne vomfixa atenţia asupra primului. Evident că u n > 0, ∀n ∈ N şi se arată uşor prininducţie că u 2n−1 < 1 şi u 2n > 1, ∀n ∈ N ∗ . Vom demonstra acum că u 2n+2 < u 2n şi148


u 2n+1 > u 2n−1 , ∀n ∈ N ∗ ; efectuăm calculele doar pentru prima afirmaţie:uu 2n+2 < u 2n ⇔ u 2n+1 + 1u 2 2n+1 + 1 < u 2n ⇔ u 2n + 1u 2 2n + 1 + 1‹Á< u 2n ⇔ u4 2n + u 3 2n + 3u 2 2n + u 2n + 2u 4 2n + 3u2 2n + 2u 2n + 212n + 11‹2u 2 2n + +< u 2n ⇔ u 5 2n − u 4 2n + 2u 2 2n−−u 2 2n + u 2n − 2 > 0 ⇔ (u 2n − 1)(u 4 2n + 2u 2 2n + u 2n + 2) > 0,adevărat. În concluzie, subşirul (u 2n−1 ) n≥1 este crescător şi mărginit, iar subşirul(u 2n ) n≥1 este descrescător şi mărginit inferior de 1; înseamnă că ambele sunt convergentespre α, respectiv β. Trecând la limită în relaţia de recurenţă, obţinem căα = β + 1β 2 + 1 , iar β = α + 1α 2 + 1 . Înlocuind, găsim pentru α ecuaţia α5 − α 4 + 2α 3 − α 2 +α − 2 = 0, deci (α − 1)(α 4 + 2α 2 + α + 2) = 0 şi, cum α > 0, înseamnă că α = 1.Deducem apoi că β = 1, prin urmare lim u n = 1.n→∞Notă. În aceeaşi manieră, autorii problemei au stabilit că pentru orice a, b ∈(0, ∞), şirul (u n ) n≥1 definit prin u 1 ≥ 0, u n+1 = ua n + bu a+1 este convergent către 1.n + bXI.94. Să se demonstreze că pentru orice n ∈ N ∗ , există numerele distinctex 1 , x 2 , . . . , x n ∈ (1, 2), aşa încât x 1 x 2 · · · x n =4 .enDan Plăeşu , IaşiSoluţia 1 (a autorului). Considerăm funcţia f : [1, 2] → R, f(x) = x ln x − x.Aplicând teorema lui Lagrange funcţiei f pe intervalele•1 + k − 1n , 1 + n˜,k k =1, 2, . . . , n, determinăm x k ∈ 1 + k − 1n , 1 + n‹, k k = 1, 2, . . . , n, astfel încât f 1 +n‹−kf 1 + k − 1 ‹= 1 n n f ′ (x k ), k = 1, 2, . . . , n. Rezultă că f ′ (x 1 ) + f ′ (x 2 ) + . . . + f ′ (x n ) =n · f 1 +n‹−f k 1 + k − 1 ‹‹=f(2) − f(1). Folosind faptul că f ′ (x) = ln xnnPk=1şi că f(2) − f(1) = ln 4 , din ultima relaţie obţinem concluza problemei.eSoluţia 2 (Gabriel Popa). Construcţia şirului (x n ) n≥1 se poate face inductiv:luăm x 1 = 4 e ∈ (1, 2), iar dacă presupunem existenţa numerelor x 1, x 2 , . . . , x n ∈ (1, 2)pentru care x 1 x 2 . . . x n = 4 4atunci numerele x 1 , . . . , x n−1 , k n x n şie‹n,k n e , unde4(k n ) n≥1 este un şir de numere din (1, 2) cu limita 1, au produsul şi pot fie‹n+1făcute distincte, alegând convenabil k n (posibil, întrucât orice vecinătate la dreaptaa lui 1 este nenumărabilă).Soluţia 3 (Gheorghe Iurea). Căutăm numerele x k , k = 1, n de forma x k =4e‹a kcu ak ∈ 0, 3 2‹, k = 1, n. Condiţia problemei devine a 1 + a 2 + . . . + a n = n.149


Pentru n = 2p putem alege numerele 1 − 1 3 , 1 + 1 3 , 1 − 1 4 , . . . , 1 − 1p + 2 , 1 + 1p + 2 , iarpentru n = 2p+1 putem alege numerele 1, 1− 1 3 , 1+ 1 3 , 1− 1 4 , . . . , 1− 1p + 2 , 1+ 1p + 2 .XI.95. Calculaţi lim +n→∞1 1n α+1 + 1α12 α + · · · +1 + 1α1n α − n, unden nα ≥ 1 este fixat. (În legătură cu L83 din RecMat-1/2005.)Marius Olteanu, Rm. VâlceaSoluţie. Notăm cu (x n ) n≥1 şirul a cărui limită o căutăm. Cum 1 + 1 k α >n α‹11, ∀k = 1, n, deducem că x n > 0, ∀n ∈ N ∗ . Din inegalitatea lui Bernoulli, obţinem că1 + 1α‹1k α < 1 + 1n n α · 1k α , deci x n < 1n α 1 + 12 α + . . . + 1n α‹, ∀n ∈ N ∗ . Din teoremaCesarò-Stolz, lim1n→∞ n α 1 + 12 α + . . . + 1n α‹= 1limn→∞ (n + 1) α · 1(n + 1) α − n α .Dacă vom dovedi că această din urmă limită este 0, conform criteriului cleştelui rezultăcă lim x n = 0.n→∞Aplicând teorema lui Lagrange funcţiei f : [n, n+1] → (0, ∞), f(x) = x α , deducemcă (n + 1) α − n α = α · c α−1 , cu c ∈ (n, n + 1). Atunci 1 α · 1(n + 1) 2α−1 < 1(n + 1) α ·1(n + 1) α − n α < 1 α · 1(n + 1) α , ceea ce, prin trecere la limită, conduce la faptul· nα−1 1că limn→∞ (n + 1) α · 1(n + 1) α − n α = 0.Clasa a XII-aXII.91. Prove thatZ1(1 + x) e x(1+ex) dx = e e − 1.Ze00Zdravko Starc, Vr˘sac, SerbiaSoluţie. Facem substituţia xe x = t; atunci (x+1)e x dx = dt şiZ1(1+x)e x(1+ex) dx =e t dt = e te0 = ee − 1.XII.92. Fie b > a > 0, iar f : [a, b] → R o funcţie continuă pe [a, b] şi derivabilăpe (a, b); să se arate că există c ∈ (a, b) astfel încât bZcf (x) dx = c (b − c) f (c).Dan Nedeianu, Drobeta Tr. SeverinSoluţie. Aplicăm teorema lui Rolle funcţiei g : [a, b] → R, g(x) = x − bZxf(t)dtx aşi ţinem cont că g ′ (x) = b2Zxf(t)dt + x − bx axf(x).2 sin x 2 cos 1XII.93. Demonstraţi că există c ∈ (2, π) pentru careZπdx ≤ .1 x cConstantin Micu, Melineşti (Dolj)150a0


Soluţie.Se constată uşor că funcţia f :h1, π 2i→R, f(x) = sin x este strictcrescătoare, iar g :h1, π g(x) =2i→R, 1 xinegalităţii lui Cebîşev,este strict descrescătoare; atunci, conform1·Zπ2π2 − 1 1sin xx ≤ 1€π22·Zπ− 1Š21sin xdx ·Zπ211x dx,2 sin xln π − ln 2de undeZπdx ≤ 2 cos 1 · . Aplicând teorema lui Lagrange funcţiei1 x π − 2h : [2, π] → R, h(x) = ln x, găsim c ∈ (2, π) pentru care 1 ln π − ln 2= şi astfelc π − 2soluţia problemei este completă.nZ2n x a + bXII.94. Calculaţi lim √ dx, unde a ∈ (0, ∞) şi b ∈ R.n→∞ n x2a+4+ 1Soluţie. Fie f : [0, ∞) → R, f(x) =x ≥ 0. Cum 0 ≤ F (x) ≤Zxcă lim F (x) = 0. Atuncix→∞xZ2xlimx→∞ x= limx→∞0x a + b√x2a+4+ 1t a + bt a+2 dt = 1 x − 12x +Liviu Smarandache, Craiovat şi F (x) =Zx a + b√t2a+4+ 1 dt,ba + 101x a+1 − 1(2x) a+1‹, obţinemt a + bF (2x) − F (x)√ dt = lim x(F (2x) − F (x)) = limt2a+4+ 1 x→∞ x→∞ 1=x2f(2x) − f(x)− 1 x 2= limx→∞ (x2 f(x) − 1 2 (2x)2 f(2x)) = 1 − 1 2 = 1 2 ,x a+2 + bx 2deoarece limx→∞ x2 f(x) = lim √x→∞ x2a+4+ 1 = 1. Prin urmare, limita cerută este 1 2 .XII.95. Fie (A, +, ·) un inel în care 0 ≠ 1 şi 1 + 1 + 1 + 1 + 1 = 0. Să se aratecă, dacă x 3 y 2 = y 2 x 3 , ∀x, y ∈ A, atunci inelul este comutativ.I.V. Maftei, Bucureşti şi Mihai Haivas, IaşiSoluţie. Fie x, y ∈ A, arbitrare. Din x 3 y 2 = y 2 x 3 şi x 3 (y + 1) 2 = (y + 1) 2 x 3 ,găsim că x 3 (y + y) = (y + y)x 3 . Cum 1 + 1 + 1 + 1 + 1 = 0, rezultă că 5y = 0, decix 3 (5y) = (5y)x 3 . Deducem că x 3 (3y) = (3y)x 3 , prin urmare x 3 y = yx 3 . Folosindaceastă relaţie pentru x şi x + 1, obţinem x 3 y = yx 3 şi (x + 1) 3 y = y(x + 1) 3 , deunde (3x 2 + 3x + 1)y = y(3x 2 + 3x + 1). Înlocuind pe x cu x + 1 şi ţinând cont că3(x+1) 2 +3(x+1)+1−(3x 2 +3x+1) = 6x+6 = x+1, rezultă că (x+1)y = y(x+1),deci xy = yx. Cum x, y sunt arbitrare din A, concluzia problemei se impune.151


Soluţiile problemelor pentru pregătireaconcursurilor propuse în nr. 2/2008A. Nivel gimnazialG146. Fie x, y, z ∈ (0, ∞) astfel încât xyz = 1. Arătaţi căxy 3x 4 + y + z + yz 3y 4 + z + x + zx 3z 4 + x + y ≥ 1.X =X ≥X Liviu Smarandache şi Lucian Tuţescu, CraiovaSoluţie. Avem că x 3 + y 3 ≥ xy(x + y), cu egalitate pentru x = y. Atuncixy xy 3y 3x 3x 4 + xyz(y + z) x 3 + yz(y + z) x 3 + y 3 + z 3 = 1.3x 4 + y + z =XEgalitatea se atinge pentru x = y = z = 1.G147. Fie n ∈ N, n ≥ 2, fixat, iar a, b, c sunt numere naturale astfel încâtna + (n + 1) b + 2nc = n 2 + 1. Arătaţi că n −hn − 12i≤a + b + c ≤ n.Gheorghe Iurea, IaşiSoluţie. Cum b − 1 = n(n − a − b − 2c), înseamnă că b − 1 . .n. Însă b < n, altfelna + (n + 1)b + 2nc > n 2 + 1 şi atunci b − 1 = 0, deci b = 1. Condiţia din enunţ devinena + n + 1 + 2nc = n 2 + 1 ⇔ a + 2c = n − 1 ⇔ a + b + c = n − c. Suma a + b + ceste maximă când c este minim, adică pentru c = 0; deducem că (a + b + c) max = n,maxim atins când a = n − 1, b = 1, c = 0. Suma a + b + c este minimă când c este− 1− 1maxim, deci pentru c =•n˜; obţinem că (a + b + c) min = n −•n˜, minim2 2− 1− 1atins când a = n − 1 − 2•n˜, b = 1, c =•n˜.2 2G148. Fie a 1 a 2 . . . a p ∈ N. Să se arate că orice număr natural are un multiplude forma a 1 a 2 . . . a p a 1 a 2 . . . a p . . . a 1 a 2 . . . a p 0 . . . 0.Marian Panţiruc, IaşiSoluţie. Fie n ∈ N; considerăm numerele: a 1 a 2 . . . a p , a 1 a 2 . . . a p a 1 a 2 . . . a p , . . . ,a 1 a 2 . . . a p a 1 a 2 . . . a p . . . a 1 a 2 . . . a p , în număr de (n + 1). Prin împărţirea acestora lan obţinem (n + 1) resturi şi, cum resturile posibile sunt 0, 1, . . . , n − 1, rezultă căcel puţin două resturi sunt egale. Fie a = a 1 a 2 . . . a p . . . a 1 a 2 . . . a p (format din k 1numere a 1 a 2 . . . a p şi b = a 1 a 2 . . . a p . . . a 1 a 2 . . . a p (format din k 2 numere a 1 a 2 . . . a p )două dintre numerele de mai sus, care dau acelaşi rest la împărţirea cu n. Diferenţaacestora se divide cu n, deci a−b = a 1 a 2 . . . a p a 1 a 2 . . . a p . . . a 1 a 2 . . . a p 00 . . . 0 verificăcerinţa problemei.G149. a) Determinaţi două numere prime p, q astfel încât p < q, iar p 2 − 1 aremai mulţi divizori naturali decât q 2 − 1.b) Determinaţi toate numerele prime p pentru care p 2 − 1 are exact opt divizorinaturali.Dan Popescu, Suceava152


Soluţie. a) De exemplu, putem lua p = 19, q = 23.b) Dacă p ≠ 2, atunci p 2 ≡ 1 (mod 8). Dacă p ≠ 3, atunci p 2 ≡ 1 (mod 3).Cum (3, 8) = 1, înseamnă că dacă p /∈ {2, 3}, atunci p 2 ≡ 1 (mod 24), prin urmarep 2 − 1 . .24. Pentru p ≥ 7, avem, că p 2 − 1 > 24 şi, cum 24 are opt divizori naturali,p 2 −1 va avea mai mult de opt divizori. Pentru p ∈ {2, 3}, p 2 −1 are mai puţin de optdivizori. Dacă p = 5, atunci p 2 − 1 = 24 are exact opt divizori, deci singurul numărcare satisface condiţiile din enunţ este 5.ªG150. Fie m şi n numere naturale nenule cu proprietatea că m ≤ 1 + 2 + · · · + n.Să se arate că m poate fi scris ca suma câtorva numere distincte dintre 1, 2,. . . , n.Marian Tetiva, Bârladn(n + 1)Soluţia 1 (Dan Mocanu, elev, Iaşi). Acoperim mulţimea§1, 2, 3, . . . ,2cu următoarele sume având termenii distincţi: 1, 2, 3, . . . , n, n+1, n+2, . . . , n+(n−1),n + (n − 1) + 1, n + (n − 1) + 2, . . . , n + (n − 1) + (n − 2), . . . , n + (n − 1) + . . . + 1;concluzia problemei este acum imediată.Soluţia 2 (a autorului). Demonstrăm prin inducţie după n. Pentru n = 1, decim = 1, nu avem nimic de demonstrat; de asemenea se verifică uşor cazul n = 2 (decim = 1, 2 sau 3). Vom presupune mai departe n ≥ 2 şi că afirmaţia este adevăratăpentru n − 1 şi o demonstrăm pentru n.Dacă m este unul dintre numerele 1, 2, . . . , n nu avem ce arăta. Dacă m ≥ n + 1,avem că 1 ≤ m − n ≤ 1 + 2 + . . . (n − 1) şi, conform ipotezei de inducţie, m − n estesuma unor termeni distincţi din mulţimea {1, 2, . . . , (n − 1)}. Rezultă că m este sumadintre aceşti termeni şi n, ceea ce încheie demonstraţia.G151. Bazele unei prisme sunt poligoane cu 2008 vârfuri. Numerotăm cu 1,2,. . . , 2008 vârfurile bazei inferioare şi, corespunzător, cu a 1 , a 2 ,. . . , a 2008 vârfurilebazei superioare, unde {a 1 , a 2 , . . . , a 2008 } = {1, 2, . . . , 2008}.a) Demonstraţi că putem găsi o numerotare pentru baza superioară astfel încâti + a i. . 8, ∀i ∈ {1, 2, . . . , 2008}.b) Demonstraţi că nu putem găsi o numerotare pentru baza superioară astfel încâti + a. i 9, ∀i ∈ {1, 2, . . . , 2008}.Gabriel Popa şi Gheorghe Iurea, IaşiSoluţie. a) De exemplu, putem lua a 1 = 2007, a 2 = 2006, . . . ,a 2007 = 1, a 2008 =2008.b) Dacă ar exsta o numerotare pentru care i + a i. .9, ∀i ∈ {1, 2, . . . , 2008}, atunci(1 + a 1 ) + (2 + a 2 ) + . . . + (2008 + a 2008 ) . .9, prin urmare 2(1 + 2 + . . . + 2008) . .9. Amobţine astfel că 2008 · 2009 . .9, contradicţie.G152. În triunghiul isoscel ABC (AB = AC) notăm cu B′ , C ′ picioarele înălţimilordin B, respectiv C. Dacă AB = 2 B ′ C ′ , să se determine unghiurile triunghiului.Nela Ciceu, Bacău şi Titu Zvonaru, ComăneştiSoluţie. Vom analiza două cazuri, după cum unghiulbA este ascuţit sau obtuz.a) Fie M mijlocul laturii [AB]; atunci [B ′ M] va fi mediană în △ABB ′153


dreptunghic şi deducem că B ′ M = 1 2 AB. Avem şi că B′ C ′ = 1 2 AB,prin urmare B ′ C ′ = B ′ M, deciÖB ′ MB ≡ÖB ′ C ′ A. ÎnsăÖB ′ C ′ A ≡ÒB (deoarece B ′ C ′ ∥BC) şi atunci m(ÒB) = m(×B ′ MC ′ ) = 180 ◦ −2m(ÖMBB ′ ) = 180 ◦ −2[90 ◦ −m(bA)] = 2m(bA), relaţie care împreunăcu m(bA)+2m(ÒB) = 180 ◦ conduce la m(bA) = 36 ◦ , m(ÒB) = m(ÒC) =72 ◦ .b) DacăbA este obtuz, atunci △B ′ MA va fi isoscel şicu un raţionament asemănător celui de mai sus obţinemcă m(ÒB) = m(×B ′ C ′ M) = 180 ◦ − 2m(ÖB ′ AB) = 180 ◦ − 2 ·2m(ÒB) = 180 ◦ − 4m(ÒB), prin urmare m(ÒB) = m(ÒC) = 36 ◦ ,iar m(bA) = 108 ◦ .CBBBCG153. În triunghiul ABC, M este mijlocul laturii [BC], m(ÕABC) = 30 ◦ şim(ÕACB) = 105 ◦ . Perpendiculara din C pe AM taie AB în Q. Calculaţi valoarearaportului QAQB .Neculai Roman, Mirceşti (Iaşi)Soluţie. Ducem CE⊥AB, E ∈ (AB). Folosind triunghiul CEM echilateralşitriunghiul AEC dreptunghic isoscel, găsim că ME = EC = EA, deci triunghiulAEM este isoscel cu m(AEM) = 150 ◦ . Atunci m(ÖEAM) = 15 ◦ , deci m(ÖMAC) = 30 ◦şi m(ÖAMC) = 45 ◦ . Prin urmare, m(ÕBCQ) = 45 ◦ şi m(ÕACQ) = 60 ◦ . Obţinem căQAQB = A ACQ BC · CQ sin BCQ AC sin 60◦= =A BCQ AC · CQ sinÕACQ BC sin 45 ◦ .Dar AC = EC √ 2 = BC√ 2şi atunci QA√32QB = 2 .G154. Fie D mijlocul laturii [BC] a triunghiului echilateral ABC de latură 1,iar P un punct mobil pe [CD]. Notăm cu M şi N proiecţiile pe AP ale punctelor B,respectiv C. Aflaţi aria locului geometric descris de segmentul [MN].Marius Olteanu, Rm. VâlceaSoluţie. Observând că patrulaterele ABDMAşi ACND sunt inscriptibile, deducem că puncteleM şi N se află pe cercurile C 1 (circumscris triunghiuluiABD), respectiv C 2 (circumscris triunghiuluiACD). Dacă E este mijlocul laturii F E[AC], se constată că M parcurge arcul mic÷DE alcercului C 1 , în timp ce N parcurge arcul mic÷CDMP Cal cercului C 22 . Astfel, locul geometric măturat de[MN] este suprafaţa haşurată în figură. Observăm B D CC 1 Ncă segmentele de disc mărginite de C 1 şi [DE], respectivde C 2 şi [CD], sunt congruente; atunci√aria locului geometric va fi egală cu3aria triunghiului echilateral CDE, adică16 .154MMAACBC


G155. Fie C cercul circumscris △ABC ascuţitunghic. Notăm cu P punctul deintersecţie al tangentelor duse la cerc în B şi C, {D} = AP ∩ C, iar M şi N suntmijloacele arcului mic BC, ⌢respectiv arcului mare BC. ⌢Să se arate că dreptele AM,DN şi BC sunt concurente.Gabriel Popa, IaşiSoluţie. Fie {T } = AM ∩BC. Cum AM este bisectoarea unghiuluiÕBAC, rezultăcăBTN(1)T C = ABAC .ACVom arăta că DT este bisectoare pentruÕBDC; atunciD, T, N vor fi coliniare şi de aici concluzia problemei.CumÕBAP ≡ÕP BD, rezultă că △P AB ∼ △P BD,TBC. . deci P BP D = AB . Analog se arată că △P AC ∼ △P CD,BDde unde.P CP D = ACAB. Însă P B = P C, prin urmareCD BD =ACD MCD ⇔ ABAC = BDCD . Ţinând cont de (1), obţinem căBTT C = BDDC , adică DT este bisectoarea luiÕBDC, ceea ceîncheie rezolvarea.PB. Nivel licealL146. În plan se consideră dreptele d 1 , d 2 ,. . . , d n+1 , oricare două neparalele.Notăm cu α k = m(Ød k , d k+1 ), α k ≤ 90 ◦ , k = 1, n. Pe d 1 se consideră un segmentde lungime 2 care se proiectează pe d 2 , apoi segmentul obţinut se proiecteazăpe d 3 şi tot aşa, până când pe d n+1 se obţine un segment de lungime 1. Ştiind cătg€min¦α i | i = 1, √ n©Š=Èn4 − 1, determinaţi unghiurile α k , k = 1, n.Cristian Săvescu, student, BucureştiSoluţie. Fie A 1 B 1 segmentul de lungime 2 de pe d 1 , iar A k = pr dk A k−1 ,B k = pr dk B k−1 , k = 2, n + 1. Cum A k+1 B k+1 = A k B k · cos α k , atunci A n+1 B n+1 =A 1 B 1 cos α 1 cos α 2 . . . cos α n , de unde cos α 1 cos α 2 . . . cos α n = 1 2 . Fie α p = min{α i |i =1, n}; cum cosinusul este descrescător pe0, π 2i, avem cos α p ≥ cos α k , ∀k = 1, n şiastfel cos n α p ≥ 1 2 , deci cos α p ≥ 1 √ √ n2 , apoi sin α 4 − 1p =È1 − cos 2 α p ≤Ènn√ , prin 2urmare tg α p ≤Èn √ 4 − 1. Conform ipotezei, rezultă că se atinge egalitatea; acestlucru are loc pentru cos α 1 = cos α 2 . . . = cos α n , deci când α 1 = α 2 = . . . = α n =arccos 1n√2.L147. Se consideră un poligon convex cu n laturi, n ≥ 4, având proprietatea căoricare două diagonale nu sunt paralele şi oricare trei nu sunt concurente în punctediferite de vârfurile poligonului. Se notează cu n i numărul punctelor de intersecţie a155


diagonalelor interioare poligonului şi cu n e cel al punctelor de intersecţie exterioarepoligonului.a) Să se arate că există exact opt poligoane care verifică relaţia n i > n e .b) Să se arate că există exact trei poligoane pentru care n i + n e = kn 2 , k ∈ N ∗ .Mihai Haivas, IaşiSoluţie. Fiecare punct interior de intersecţie a diagonalelor este unic determinatde cele două diagonale ce-l conţin, deci de patru vârfuri ale poligonului; rezultă căn i = Cn. 4 n(n − 3)Cum sunt diagonale, care se intersectează în C 2 n(n−3) puncte,22fiecare vârf al poligonului fiind numărat de Cn−3 2 ori (se obţine ca intersecţie a oricaredouă diagonale care trec prin acel vârf), obţinem că numărul total de intersecţii, fărăvârfuri, este n i + n e = C 2 n(n−3)2n(n − 3)(n − 4)(n − 5)n l = .− nCn−3 2 = n(n − 3)(n2 − 7n + 14). Prin urmare,812a) Din condiţia n i > n e rezultă că n 3 −15n+38 < 0, cu soluţiile n ∈ {4, 5, 6, . . . , 11}.b) Condiţia n i + n e = kn 2 este echivalentă cu n 3 − 10n 2 + (35 − 8k)n − 42 = 0,deci n ∈ {7, 14, 12}, cărora le corespund valorile k ∈ {1, 11, 33}.Notă. Într-o manieră asemănătoare a rezolvat problema dl. Daniel Văcaru,Piteşti.L148. Pe latura (AB) a triunghiului ABC considerăm punctul D astfel încâtAB = 4 AD. De aceeaşi parte a laturii AB ca şi punctul C, luăm un punct Pastfel încâtÕP DA ≡ÕACB şi P B = 2 P D. Demonstraţi că patrulaterul ABCP esteinscriptibil.Nela Ciceu, Bacău şi Titu Zvonaru, ComăneştiSoluţia 1 (a autorilor). Considerăm punctul Q astfel încât AB separă PAQşi Q, AB = 2AQ şiÕQAB ≡ÕBP D. AtunciAAB =P DPP B= 1 obţinem că △AQB ∼ △P DB, de unde2‹şiQ ÕAQB = ÕP DB. Rezultă că m(ÕAQB) + m(ÕACB) = Dm(ÕP DB) + m(ÕP DA) = 180 ◦ , ceea ce asgură inscriptibilitateapatrulaterului AQBC. Din AB = 2AQ şi AB =BC4AD deducem că AQ 2 = AD · AB, astfel scris AQAB = AD , prin urmare △AQD ∼AQ△ABQ. Obţinem căÕADQ ≡ÕAQB şi cum avem şiÕAQB ≡ÕP DB, rezultă că puncteleQ, D şi P sunt coliniare. Ne amintim căÕQAB ≡ÕQP B, deci patrulaterul AQBPeste inscriptibil. Inscriptibilitatea patrulaterelor AQBC şi AQBP arată că puncteleA, Q, B, C, P sunt conciclice, de unde concluzia problemei.Soluţia 2 (Daniel Văcaru, Piteşti). Folosind relaţia lui Stewart în △P AB,obţinem că P A 2 · BDAB −P D2 +P B 2 · ADAB = AD ·DB, deci P A2 · 34 −P D2 +P B 2 · 14 =316 AB2 şi, cum P B = 2P D, deducem că P A 2 · 34 = 3 16 AB2 , adică P A = 1 2 AB.Aplicăm acum teorema cosinusului în △ADP : P A 2 = AD 2 +DP 2 −2AD·DP ·cos C,156


putem alege x 1 > 0, x 2 < 0. Aria triunghiului P T 1 T 2 va fixS P T1 T 2= 1 1 1 + x 2k22 1 x 1 y = 1 1 21 x 2 y 2Gândim această arie ca o funcţie în nedeterminata x 1 , derivata acestei funcţii este34 · ax 4 x 2 1 − k , iar punctele critice ale funcţiei sunt x1a‹2′ 1 =É−ka şix ′′1 = −É−ka . Studiind semnul derivatei, se observă că x′ 1 este punct de minimpentru arie şi obţinem că x p = 0. În concluzie, punctul P căutat este intersecţiadreptei date cu Ox.· x 2 1 + k a‹k− x 1‹k − a ax 1 2x 2 1 +k2a 2 x 2 1.L150. Fie tetraedrul A 1 A 2 A 3 A 4 , iar P un punct în interiorul său. Notămcu A ij ∈ (A i A j ) proiecţiile ortogonale ale lui P pe muchiile A i A j ale tetraedrului.Demonstraţi căV P A12A 13A 23+ V P A12A 14A 24+ V P A13A 14A 34+ V P A23A 24A 34≤ 1 4 V A 1A 2A 3A 4.Când se atinge egalitatea?Marius Olteanu, Rm. VâlceaSoluţie. Fie P 1 = P r (A2 A 3 A 4 )A 1 ; din reciproca teoremei celor trei perpendiculare,obţinem că P 1 A 23 ⊥ A 2 A 3 , P 1 A 24 ⊥ A 2 A 4 , P 1 A 34 ⊥ A 3 A 4 , prin urmare A 23 A 24 A 34este triunghiul podar al punctului P 1 în raport cu △A 2 A 3 A 4 . Aria s 1 a acestuitriunghi este cel mult un sfert din aria S 1 a △A 2 A 3 A 4 . Dacă mai notăm h 1 = A 1 P 1 ,V P A23 A 24 A 34x 1 = P P 1 , atunci= x 1s 1≤ 1 V A1A 2A 3A 4h 1 S 1 4 · x1 . Introducem analog x 2 , x 3 , x 4h 1şi h 2 , h 3 , h 4 ; putem scrie încă trei inegalităţi analoage celei precedente. Concluziaproblemei se obţine adunând cele patru inegalităţi şi ţinând seama de relaţia luix 1Gergonne + x 2+ x 3+ x 4= 1‹.h 1 h 2 h 3 h 4Deoarece 4s i = S i doar atunci când P i coincide cu centrul cercului circumscrisfeţei care se opune vârfului A i , rezultă că egalitatea se atinge când P este centrulsferei circumscrise tetraedrului.L151. Să se demonstreze că nu există numere naturale n şi k astfel încâth€2 + √ 3Š2n+1i=h€4 + √ 15Ški.Cosmin Manea şi Dragoş Petrică, PiteştiSoluţie. Avem că (2 + √ 3) 2n+1 = 2 2n+1 + C2n+12 1 2n√ 3 + C2n+12 2 2n−1 · 3 +. . . + C2n+1 2n+1 (√ √3) 2n+1 = a n + b n 3, cu an , b n ∈ N, iar (2 − √ √3) 2n+1 = a n − b n 3.Astfel, [(2 + √ 3) 2n+1 ] = [(2 + √ 3) 2n+1 + (2 − √ 3) 2n+1 − 1 + 1 − (2 − √ 3) 2n+1 ] =[2a n − 1 + 1 − (2 − √ 3) 2n+1 ] = 2a n − 1, deoarece 1 − (2 − √ 3) 2n+1 ∈ (0, 1). La felstabilim că (4 + √ √ √ √15) k = c k + d k 15, (4 + 15) k = c k − d k 15, ck , d k ∈ N, iar[(4 + √ 15) k ] = 2c k − 1. Dacă presupunem că există n şi k cu proprietăţile cerute,158


avem a n = c k , deci 2 2n+1 + M 3 = 4 k + M 15 ; rezultă că 3|2 · 4 n − 4 k . Dar 2 · 4 n − 4 k =2(M 3 + 1) − (M 3 + 1) = M 3 + 1 şi obţinem contradicţia 3|1, prin urmare nu există nşi k cu proprietatea cerută.L152. Pentru a, b, c ∈ R şi x ∈ R + , demonstraţi inegalitatea9a 2 + b 2 + c 2 ≤ 3 (x + 1) 2 (a + b + c) 4”3(x 2 + 1)(a 2 + b 2 + c 2 ) + 2x(a + b + c) 2—(ab + bc + ca) 2 ≤ 1 a 2 + 1 b 2 + 1 c 2 .I. V. Maftei şi Dorel Băiţan, BucureştiSoluţie. Utilizând sumarea ciclică, inegalitatea din stânga se scrie 3[3(x 2 +1)(Xa 2 ) + 2x(Xa) 2 ] · (Xab) 2 ≤ (x 2 + 1)(Xa) 4 (Xa 2 ) şi aceasta se obţineadunând inegalităţile(1) 9(Pab) 2 ≤ (Pa) 4 ; 3(Pab) 2 ≤ (Pa) 2 (Pa 2 ),prima multiplicată prin (x 2 + 1)(Xa 2 ), iar a doua prin 2x(Xa) 2 .Inegalitatea din dreapta se scrie sub forma(x + 1) 2 a 2 b 2 c 2 (Xa) 4 ≤ (Xa 2 b 2 )(Xab) 2 [(x 2 + 1)(Xa 2 ) + 2x 3 (Xa) 2 ]şi se poate obţine adunând inegalităţile(2) a 2 b 2 c 2 (Pa) 4 ≤ (Pa 2 b 2 )(Pab) 2 (Pa 2 ); a 2 b 2 c 2 (Pa) 2 ≤ 1 3 (Pa 2 b 2 )(Pab) 2 ,prima multiplicată prin (x 2 + 1), iar a doua prin 2x(Xa) 2 .Justificarea inegalităţilor (1) şi (2) revine la demonstrarea inegalităţilor 9(Xab) 2 ≤(Xa) 4 ; (Xa) 2 ≤ 3(Xa 2 ); 3abc(Xa) ≤ (Xab) 2 ; abc(Xa) ≤Xa 2 b 2 , care suntrelativ uzuale.L153. Găsiţi toate funcţiile f : R → R cu proprietatea căf€x 2 + xy + yf (y)Š=xf (x + y) + f 2 (y) , ∀x, y ∈ R.Adrian Zahariuc, student, PrincetonSoluţie. Căutăm un z ≠ x astfel încât x 2 +xy = z 2 +zy; găsim z = −x−y. Atuncixf(x + y) + f 2 (y) = f(x 2 + xy + yf(y)) = f(z 2 + zy + yf(y)) = zf(z + y) + f 2 (y) =(−x−y)f(−x)+f 2 (y). Rezultă că xf(x+y) = −(x+y)f(−x), pentru orice x, y ∈ R.Deducem că f(x) (x ≠ 0) este constantă, deci f(x) = cx, c ∈ R. Cum pentru x = 0xşi y ≠ 0 obţinem f(0) = 0, rezultă că f(x) = cx, c ∈ R, pentru orice x ∈ R.Se verifică uşor că toate aceste funcţii au proprietatea dorită.Notă. O soluţie corectă, dar ceva mai laborioasă, a fost primită din partea d-luiDaniel Văcaru, Piteşti.L154. Fie P ∈ R [X] un polinom de gradul n şi p : R → R funcţia polinomialăasociată. Ştiind că mulţimea {x ∈ R | p (x) = 0} are k elemente (distincte), iar159


funcţia f : R → R, f (x) = |p (x)| este derivabilă pe R, arătaţi că numărul maxim derădăcini complexe nereale ale lui P este egal cu 2hn2i−2k.Vlad Emanuel, student, BucureştiSoluţie. Studiem derivabilitatea funcţiei f. Dacă x 0 ∈ R şi p(x 0 ) ≠ 0, atunci pare semn constant într-o vecinătate a lui x 0 . Pe această vecinătate f(x) = p(x) (sauf(x) = −p(x). Rezultă că f este derivabilă în x 0 . Dacă x 0 ∈ R şi P (x 0 ) = 0, atuncif s(x ′ |P (x)| − |P (x 0 )|0 ) = limx→x 0 x − xxx 00= − lim= limx→x 0xx 0P (x) − P (x 0 )=−|Px − xs(x ′ 0 )| = −|P ′ (x 0 )| şi0P (x) − P (x 0 )=|Pd ′ x − x (x 0)| = |P ′ (x 0 )|.0În concluzie, f este derivabilă în x 0 dacă şi numai dacă −|P ′ (x 0 )| = |P ′ (x 0 )| ⇔P ′ (x 0 ) = 0.Prin urmare, f este derivabilă pe R dacă şi numai dacă orice rădăcină a lui P estecel puţin dublă. Cum P are k rădăcini reale distincte, rezultă că P are cel puţin 2krădăcini reale.Pentru n = 2p, deducem că P are cel mult 2p − 2k rădăcini complexe nereale, iarpentru n = 2p + 1, P are cel mult 2p + 1 − (2k + 1) rădăcini complexe nereale (amfolosit faptul că rădăcinile complexe sunt perechi).În final avem cel mult 2hn2i−2krădăcini complexe nereale.Cum pentru n = 2p, P = (X − 1) 2 . . . (X − k) 2 (X 2 + 1) p−k verifică ipotezeleproblemei şi are 2p − 2k rădăcini complexe nereale, iar pentru n = 2p + 1, P =(X − 1) 3 (X − 2) 2 . . . (X − k) 2 (X 2 + 1) p−k verifică ipotezele şi are 2p − 2k rădăcininereale, conclucionăm că numărul căutat este egal cu 2hn2i−2k.L155. Fie A, B ∈ M 2 (C) două matrice astfel încât matricea AB − BA să fieinversabilă. Să se arate că urma matricei (I 2 + AB) (AB − BA) −1 este egală cu 1.Florina Cârlan şi Marian Tetiva, BârladSoluţie. Considerăm plinomul f(x) = det[I 2 +AB+x(BA−AB)], care are graduldoi (coeficientul lui x 2 este egal cu det(BA − AB) = det(AB − BA) ≠ 0). Observămcă f(0) = f(1), ceea ce arată că punctul de extrem al funcţiei f este x = 1 2 .Pe de altă parte, f(x) = det(BA − AB)det[(I 2 + AB)(BA − AB) −1 + xI 2 ] =det(BA − AB)(x 2 + xtr(P ) + detP ), unde P = (I 2 + AB)(BA − AB) −1 , ceea ceînseamnă că punctul de extrem este − 1 2 tr(P ) = −1 2 tr[(I 2 + AB)(BA − AB) −1 ]. Prinurmare, − 1 2 tr[(I 2 + AB)(BA − AB) −1 ] = 1 2 , de unde tr[(I 2 + AB)(AB − BA) −1 ] = 1.Notă. Soluţie corectă a dat dl. Daniel Văcaru, Piteşti.160


Probleme propuse 1Clasele primareP.174. Mirela are un măr, o pară şi o portocală. Mama îi spune să aşeze fructelepe două farfurii astfel încât pe fiecare farfurie să fie cel mult două fructe. În câtemoduri poate aşeza Mirela cele trei fructe?(Clasa I )Inst. Maria Racu, IaşiP.175. Scrieţi toate numerele mai mici ca 27 care se pot descompunesub forma indicată alăturat.(Clasa I )Diana Tănăsoaie, elevă, Iaşia a+1 a+2 a+3P.176. Într-o bombonieră sunt cinci bomboane cu fructe şi şapte bomboane cuciocolată. Care este cel mai mic număr de bomboane pe care trebuie să-l luăm dinbombonieră, fără să ne uităm, pentru a avea cel puţin două bomboane cu ciocolată?(Clasa a II-a)Alexandru Dumitru Chiriac, elev, IaşiP.177. Cum măsurăm 1litru de apă folosind două vase negradate, unul de 5litru,iar celălalt de 8litri?(Clasa a II-a)Mariana Nastasia, elevă, IaşiP.178. Arătaţi că, dacă restul este o cincime din scăzător, atunci descăzutul seîmparte exact la 6. Care este cel mai apropiat descăzut de numărul 100 cu aceastăproprietate?(Clasa a III-a)Mirela Cucoranu, elevă, IaşiP.179. Se dau produsele: a × b = 60, a × c = 70, a × d = 95. Ştiind că b + c + deste de 9 ori mai mare decât a, să se afle valoarea lui a.(Clasa a III-a)Andreea Amarandei, elevă, IaşiP.180. Arătaţi că din şirul 7, 28, 31, 46, 61, 100 nu putem extrage patru numere acăror sumă să se împartă exact la trei.(Clasa a III-a)Dragoş Iacob, elev, IaşiP.181. Un triunghi şi un pătrat au acelaşi perimetru, exprimat printr-un numărnatural. Care este cea mai mică valoare a perimetrului? Câte valori posibile aleperimetrului sunt cuprinse între 100 şi 200?(Clasa a IV-a)Andreea Alexa, elevă, IaşiP.182. Aflaţi cea mai mică valoare a lui k astfel încât 1 21 + 2 21 + 3 21 + · · · + k 21să fie un număr natural.(Clasa a IV-a)Ionela Bărăgan, elevă, IaşiP.183. Se consideră nouă numere naturale a, b, c, . . . , i. Media aritmetică a numerelora şi b este 1, media numerelor c, d şi e este 5, iar media numerelor f, g, h şi ieste 11. Aflaţi media aritmetică a numerelor a, b, c, . . . , i şi 9.(Clasa a IV-a)Ionel Nechifor, Iaşi1 Se primesc soluţii până la data de 31 iunie 2010.161


Clasa a V-aV.109. Aflaţi câtul şi restul împărţirii numărului 3 · 2 2009 la 5 · 2 2007 .Damian Marinescu, TârgovişteV.110. Determinaţi patru numere naturale x, y, z, t cu proprietatea că 2 x−1+ 3 · 2 2y+1 + 5 · 2 3z+2 + 11 · 2 5t+1 = 2009.Cătălina Drăgan, GalaţiV.111. Demonstraţi că numărul 20 200 are 261 de cifre la scrierea în baza 10.Geanina Hăvârneanu, IaşiV.112. Demonstraţi că mulţimea A =§x = 3n + 4 ∈ N, 1000 ≤ n ≤ 2009ªare4n + 3n 1010 elemente.Daniela Munteanu, IaşiV.113. Dacă S = 1 + 1 2 + 1 3 + . . . + 113, demonstraţi că S >2009 2 .Al. Gabriel Mîrşanu, IaşiV.114. Se consideră în plan cinci drepte distincte, care împart planul în maimulte regiuni. Arătaţi că oricum am alege 2009 puncte din plan, vor exista cel puţin126 de puncte dintr-o aceeaşi regiune.Nicolae Ivăşchescu, CraiovaV.115. O mulţime de numere naturale A = {a 1 , a 2 , . . . , a 9 } are elementelearanjate strict crescător; media aritmetică a primelor două elemente este 1, mediaurmătoarelor trei este 5, iar media ultimelor patru este 11. Câte astfel de mulţimiexistă?Ionel Nechifor şi Gabriel Popa, IaşiClasa a VI-aVI.109. Determinaţi a, b, c, d ∈ N pentru care 2a + 3b + 5c + 7d = 87, dacă:a) a, b, c, d sunt numere prime;b) a, b, c, d sunt pătrate perfecte.Nicolae Ivăşchescu, CraiovaVI.110. Determinaţi perechile de numere naturale care au suma 2009 şi produsulmultiplu al numărului 2009.Dan Popescu, SuceavaVI.111. Demonstraţi că numărul A = 40! · 1 + 1 2 + . . . + 1 40‹este natural,divizibil cu 2009 · 7 2 (unde 40! = 1 · 2 · . . . · 40).VI.112. Fie a, b, c, d ∈ N ∗ astfel încât ad + bc = bd. Demonstraţi căa 2009b 2009 + a2008 cb 2008 d + . . . + a2 cb 2 d + acbd + c d ∈ N.162Mihai Haivas, IaşiCătălin Budeanu, Iaşi


VI.113. După două reduceri succesive, preţul unui frigider scade de la 2000 lei la1620 lei. Ştiind că cele două reduceri sunt proporţionale cu preţurile rămase în urmalor, aflaţi preţul frigiderului după prima reducere.Ciprian Baghiu, IaşiVI.114. Pe laturile [BC], [AC], [AB] ale triunghiului isoscel ABC (AB = AC)considerăm punctele D, E, respectiv F , astfel încât m(ÕBAD) = 2m(ÕEDC) şi m(ÕDAC)= 2m(ÕF DB). Demonstraţi că △AEF este isoscel.Doru Buzac, IaşiVI.115. Dreptele a şi b sunt perpendiculare pe segmentul [AB] în A, respectivîn B. Considerăm punctele C ∈ (AB), M ∈ a, N, P ∈ b astfel încât între oricaredouă dintre triunghiurile ACM, BCN şi BCP există câte o congruenţă. Ştiind căm(ÕBP C) = 25 ◦ , determinaţi măsurile unghiurilor triunghiului MNP.Andrei Nedelcu, IaşiClasa a VII-aVII.109. Fie ABCD dreptunghi, O mijlocul lui [AC], M ∈ (AO), N ∈ (OC),{P } = BM ∩ AD şi {Q} = BN ∩ CD. Demonstraţi că O este centrul de greutate altriunghiului BP Q dacă şi numai dacă OM = ON = 1 6 AC.Petru Asaftei, IaşiVII.110. Măsurile unghiurilor A, B şi C ale triunghiului ABC sunt direct proporţionalecu 5, 4 şi 3, iar BC = (2 + 2 √ 2 + 2 √ 3) cm. Demonstraţi că perimetrul şi ariatriunghiului sunt numeric egale.Constantin Apostol, Rm. SăratVII.111. Fie ABC un triunghi şi punctele D ∈ (AC), E ∈ (AB), {P } = BD ∩CE. Dacă DAP CD= k, demonstraţi că k − (k + 1)PDC P E P B = 1.Neculai Roman, Mirceşti (Iaşi)VII.112. Fie ABCD trapez cu baza mare [AB], {E} = AD ∩ BC, {O} =AC ∩ BD, iar OP ∥AB, cu P ∈ (AD). Demonstraţi că CP şi CE sunt bisectoarele(interioară, respectiv exterioară) unghiuluiÕACD, dacă şi numai dacă AB = AC.Claudiu Ştefan Popa, IaşiVII.113. a) Demonstraţi că √ √3 · bcb 2 − bc + c 2 ≥ √b2 + bc + c , ∀b, c ∈ 2 R∗ +.b) Considerând un triunghi ABC cu m(bA) = 120 ◦ , AB = c, AC = b, interpretaţigeometric inegalitatea de la a).Dan Mocanu, elev, IaşiVII.114. Demonstraţi că produsul a două numere naturale nenule consecutivenu poate fi egal cu produsul altor patru numere naturale consecutive.Mihai Crăciun, Paşcani1VII.115. Demonstraţi căn + 1 + 1n + 2 + · · · + 14 k · n > k, ∀n, k ∈ N∗ .Cosmin Manea şi Dragoş Petrică, Piteşti163


Clasa a VIII-aVIII.109. Fie ABCDA ′ B ′ C ′ D ′ un cub de muchie a. Notăm cu E, F, G, H, K, Lmijloacele muchiilor AB, BC, CC ′ , C ′ D ′ , D ′ A ′ , respectiv A ′ A. Calculaţi volumul poliedruluiBÛ′ EF GHKL.Û ÛAdrian Corduneanu, IaşiVIII.110. Fie V ABCD piramidă patrulateră regulată. Notăm cuu = m( (V BC), (ABC)), v = m( (V BC), (V CD)) şi t = m( (V BC), (V AD)). Stabiliţidacă printre numerele u, v, t pot exista perechi de numere egale. (În legătură cuVIII.98 din RecMat 2/2008.)Claudiu Ştefan Popa, IaşiVIII.111. Fie ABC un triunghi de laturi a, b, c, astfel încât b + c = a √ 2.Demonstraţi că triunghiul este ascuţitunghic dacă şi numai dacă b şi c sunt distincteşi se află în intervalul‚a √ 24 , 3a√ 2Œ.4Romanţa Ghiţă şi Ioan Ghiţă, BlajVIII.112. Fie x, y ∈ R ∗ astfel încât xy, x y şi yÈx2 + (x + 1) 2 + x 2 (x + 1) 2 a‹sunttoate numere raţionale. Demonstraţi că x şi y sunt tot numere raţionale.Dan Nedeianu, Drobeta Tr. SeverinVIII.113. Dacă a, b, c ∈ R ∗ +, demonstraţi că 1 a +1 b +1 c = 2 1a + b + 1b + c + 1c +dacă şi numai dacă a = b = c.D.M. Bătineţu-Giurgiu, BucureştiVIII.114. Demonstraţi că oricare ar fi numerele naturale impare m, n cu m >n + 2, există numere naturale x, a, b astfel încât x = a(a + m) = b(b + n).Titu Zvoranu, ComăneştiVIII.115. Demonstraţi că 5(a 2 + b 2 ) 2 ≤ 4a 4 b 4 + (a + b) 4 , ∀a, b ∈ [1, +∞).Lucian Tuţescu şi Ion Vişan, CraiovaClasa a IX-aIX.101. Prin inducţie matematică se arată că are loc inegalitate a lui Bernoulli(1) (1 + x) n ≥ 1 + nx,∀n ∈ N, n ≥ 2 şi ∀x ∈ [−1, ∞), egalitatea fiind atinsă pentru x = 0. Arătaţi că:a) dacă n = 2k, k ∈ N ∗ , atunci (1) are loc ∀x ∈ R;b) dacă n = 2k + 1, k ∈ N ∗ , atunci (1) are loc ∀x ∈ [−2, ∞);c) dacă n = 3, atunci (1) are loc ∀x ∈ [−3, +∞), cu egalitate când x ∈ {−3, 0},iar pentru x ∈ (−∞, −3), (1) are loc cu sens contrar.Dorin Dutkay, Orlando (U.S.A.) şi Florin Popovici, BraşovIX.102. Rezolvaţi în R 3 sistemul:x+y+z = 2− 1 x − 1 y − 1 z ; x2 +y 2 +z 2 = 6− 1 x 2 − 1 y 2 − 1 z 2 ; x3 +y 3 +z 3 = 2− 1 x 3 − 1 y 3 − 1 z 3 .164Vasile Chiriac, Bacău


IX.103. Fie x, y, z ∈ R cu 0 ≤ x ≤ y ≤ z. Dacă α ∈ R este astfel încâtαx + (1 − α)z ≥ 0, demonstraţi că αx + (1 − α)y ≥ 0 şi αy + (1 − α)z ≥ 0.Ovidiu Pop, Satu MareIX.104. Fie A, B, C, D patru puncte ale cercului C(O, r), {M} = AB ∩ CD,N şi P mijloacele coardelor [AB], respectiv [CD], iar Ω cel de-al patrulea vârf alparalelogramului NMP Ω.a) Arătaţi că −−→ MA + −−→ MB + −−→ MC + −−→ MD = 2 −−→ MΩ.b) Paralelele prin C şi D la AB şi paralelele prin A şi B la CD se taie două câtedouă în patru puncte ce determină un paralelogram de centru Ω.c) Ω = O dacă şi numai dacă AB ⊥ CD.Diana Vrânceanu, elevă şi Dumitru Mihalache, BârladIX.105. Într-un triunghi, cu notaţiile uzuale, demonstraţi echivalenţa condiţiilor:(i) R = r a ; (ii) cos A = cos B + cos C.Temistocle Bîrsan, IaşiClasa a X-aX.101. Calculaţi suma S =nXk=11arctg √ 2k+1arcsin √ 2k+1k+1Bencze Mihály, BraşovX.102. Rezolvaţi ecuaţia 12 x + log 2 x − 7 4‹+ 7 4 = 0. Eugen Jecan, DejX.103. Fie S, U, A trei puncte distincte. Rotind vectorul −→ SA în jurul lui S, cuun arc α ∈ (−π, π), obţinem punctul S ′ ; rotind apoi −→ UA în jurul lui U, cu un arcβ ∈ (−π, π), obţinem U ′ , U ′ ≠ S ′ . Fie M ∈ S ′ U ′ astfel încât −−→ S ′ −−→M = k · MU ′ , undek ∈ R\{0, −1}. Demonstraţi că poziţia punctului M nu depinde de A atunci şi numaiatunci când k = 1, β = α ± π * .Diana Vrânceanu, elevă şi Dumitru Mihalache, BârladX.104. Fie p, l a , l b , l c semiperimetrul, respectiv lungimile bisectoarelor unui triunghi.Determinaţi numerele reale α şi β, în funcţie de p, ştiind că soluţiile ecuaţieix 3 − p √ 3 · x 2 + αx − β = 0 sunt l a , l b şi l c .Cătălin Calistru, IaşiX.105. Determinaţi cel mai mare număr real α astfel încât inegalitateax+3ysin x + sin y sin4+ sin 3x+y4≤ α ·22să fie adevărată pentru orice x, y ∈ [0, π].Clasa a XI-aXI.101. Pentru a ∈ R ∗ +, calculaţi limx→∞ x1.+ (1 − α) sin x + y2Marian Tetiva, Bârlad+xx a − e a.D.M. Bătineţu-Giurgiu, Bucureşti* Generalizare a problemei comorii din insulă a lui G. Gamow, din One, Two, Theree . . . Infinity.165


XI.102. Determinaţi funcţiile continue f : R → R cu proprietatea că f(x 2 ) −f(y 2 ) = (x + y)f(x − y), ∀x, y ∈ R.Gheorghe Iurea, IaşiXI.103. Fie (x n ) n≥1 ⊂ R ∗ + astfel încât lim n(x n+1 −x n ) = a ∈ (1, +∞); definimn→∞n ln nşirul (y n ) n≥1 prin y n =. Calculaţi limx 1 + x 2 + . . . + x (y n) n .n n→∞Cosmin Manea şi Dragoş Petrică, PiteştiXI.104. Calculaţi limn→∞1 55 1 + 255 2 + 355 3 + . . . + n55 n.Neculai Roman, Mirceşti (Iaşi)XI.105. Considerăm matricele A k , B k ∈ M k (C), k ∈ N\{0, 1}, astfel încâtdetA k = α ∈ C ∗ , ∀k ∈ N\{0, 1}. Studiaţi convergenţa şirului (a n ) n≥2 definit prindet(A k x + B k )a n = limx→∞ k! · xnXk=2k .Cătălin Calistru, IaşiClasa a XII-aXII.101. Rezolvaţi ecuaţia x 2 + x + 1 = 0 în Z 13 şi în Z 19 , apoi deduceţi că 247divide (3 7n−1 − 1)(7 4n−1 − 1), oricare ar fi n ∈ N.Mihai Haivas, Iaşi şi I.V. Maftei, BucureştiXII.102. Determinaţi primitivele funcţiei f :0, π cos 2xf(x) =4→R,(tg x + ctgx) 2009 .Nicoleta Bran, CraiovaXII.103. Demonstraţi că există c ∈0, π 4pentru careZπ40(e tg x − 1)dx ≤ π232 ·e tg ccos 2 c .Bogdan Victor Grigoriu, FălticeniXII.104. Determinaţi funcţiile derivabile f : [0, 1] → R pentru care f(0) = 0, iarf ′ (x) = f(ax), ∀x ∈ [0, 1], cu a ∈ [0, 1] fixat.Gheorghe Iurea, Iaşi+ y f(x) + f(y)XII.105. Fie f : (0, ∞) → R o funcţie semiconvexă (fx≤ ,2 2∀x, y ∈ (0, ∞)).a) Demonstraţi că pentru orice x ∈ (0, ∞), şirul (f n (x)) n≥1 definit prin f n (x) =n•fx + 1 n‹−f(x)˜, este monoton.b) Deduceţi că pentru orice x ∈ (0, ∞), şirul (e n (x)) n≥1 definit prin e n (x) =1 +nx‹n1 , este crescător.Dan Ştefan Marinescu şi Viorel Cornea, Hunedoara166


Probleme pentru pregătirea concursurilorA. Nivel gimnazialG166. Demonstraţi că următoarele propoziţii sunt adevărate.a) ∀n ∈ N, n ≥ 2, ∃x 1 , x 2 , . . . , x n ∈ N ∗ astfel încât x 1 x 2 + x 2 x 3 + . . . + x n x 1 =x 1 x 2 . . . x n .b) ∀n ∈ N, n ≥ 5, ∄x 1 , x 2 , . . . , x n ∈ 2N ∗ astfel încât x 1 x 2 + x 2 x 3 + . . . + x n x 1 =x 1 x 2 . . . x n .c) ∃x 1 , x 2 , . . . , x n ∈ 2N + 1 astfel încât x 1 x 2 + x 2 x 3 + . . . + x n x 1 = x 1 x 2 . . . x n ⇔n ∈ 2N ∗ + 1.Dan Popescu, SuceavaG167. Fie 1 = d 1 < d 2 < . . . < d k = n toţi divizorii pozitivi ai numărului naturaln. Dacă există i, j cu j > i > 13 şi d 2 7 + d 2 i = 2 d2 j , arătaţi că n este multiplu de 8.Titu Zvonaru, ComăneştiG168. Pentru x, y, z ∈ R ∗ +, demonstraţi că are loc inegalitateax(y + z) y(x + z) z(x + y)+ +x + yz y + xz z + xy ≤ x 2x + yz +y2y + xz +z2z + xy.Ştefan Gavril, Piatra NeamţG169. Demonstraţi că există o infinitate de numere iraţionale α cu proprietateacă α 3 şi α 2 + α sunt, de asemenea, iraţionale.Gabi Ghidoveanu şi Dumitru Mihalache, BârladG170. O mulţime A ⊂ R, de cardinal 2009, are proprietatea că fiecare elemental ei este mai mare decât o zecime din suma celor 2008 numere rămase. Arătaţi că Aconţine cel puţin 12 numere negative.Andrei Nedelcu, IaşiG171. Punctele planului care au, în raport cu un reper ycartezian, ambele coordonate numere naturale, le parcurgemîn sensul indicat de săgeţi în figură, pornind din origine.Notăm cu a n,k poziţia punctului de coordonate (n, k) în şirulobţinut (de exemplu, a 0,0 = 1, a 0,2 = 4, a 2,2 = 13 etc.).Exprimaţi numărul a n,k în funcţie de n şi de k.Lucian Georges Lăduncă, Iaşi0xG172. O tablă dreptunghiulară m × n, m, n ≥ 2, are pătrăţelele unitate de laintersecţiile liniilor de ordin impar cu coloanele de ordin impar colorate în negru,restul pătrăţelelor fiind albe. A recolora o linie (coloană) înseamnă a schimba culoriletuturor pătrăţelelor acelei linii (coloane). Arătaţi că tabla nu poate fi transformatăîntr-una complet albă prin recolorarea câtorva linii şi coloane.Răzvan Ceucă, elev, IaşiG173. Notăm cu T (a, b, c) triunghiul care are laturile de lungimi a, b şi c. Demonstraţică triunghiurile T (b, 2c, 2m b ) şi T (c, 2b, 2m c ) pot fi confecţionate (pe rând)dintr-o aceeaşi bucată de carton, fără pierdere de material.Petru Asaftei, Iaşi167


G174. Se consideră triunghiul ABC isoscel cu m(bA) = 40 ◦ . Să se arate cănu există puncte P ∈ Int ABC pentru care m(ÕP AB) = 30 ◦ , m(ÕP BC) = 10 ◦ şim(ÕP CA) = 35 ◦ .Gabriel Popa şi Paul Georgescu, IaşiG175. Fie ABCD un patrulater înscris în cercul de rază R. Demonstraţi căAB · AD + CB · CD ≤ 4R 2 .Gheorghe Costovici, IaşiB. Nivel licealL166. Fie ABCD un dreptunghi, iar C un cerc prin A, care intersectează (AB),(AC) şi (AD) în M, N, respectiv P . Arătaţi că AM · AB + AP · AD = AN · AC.Gheorghe Iurea, IaşiL167. Fie ABC un triunghi cu AB > AC. Cercul înscris în triunghi este tangentlaturilor BC şi AC în D, respectiv E. Considerăm T un punct pe latura [BC] şi notămcu J centrul cercului înscris în △ABT. Dacă DE trece prin mijlocul segmentului [CJ],demonstraţi că triunghiul AT C este isoscel.Titu Zvonaru, ComăneştiL168. Demonstraţi că în orice triunghi, cu notaţiile uzuale, are loc inegalitateaab + c + bc + a + ca + b ≥ 11p2 − 15r 2 − 60Rr6p 2 − 6r 2 ≥ 3 − 24Rr 2 . Marius Olteanu, Rm. VâlceaL169. Care este probabilitatea ca razele cercurilor exînscrise unui triunghi alesaleator, să fie laturile unui nou triunghi?Petru Minuţ, IaşiXL170. Fie n ∈ N, n ≥ 2 şi a 1 , a 2 , . . . , a n ∈ R + cu a 1 +a 2 +. . .+a n = S. Considerămk ∈ N, 1 ≤ k ≤ n − 1 şi α 1 , α 2 ∈ R + cu α 1 + α 2 = 1. Demonstraţi inegalitatea(a i1 + a i2 + . . . + a ik ) α 1(S − a i1 − . . . − a ik ) α 2≤ kα 1(n − k) α 2C knnS.1≤i 1


) Există funcţii f : (a, b) ∩ Q → R cu proprietatea că |f(x) − f(y)| ≥ c, ∀x, y ∈(a, b) ∩ Q, unde c este o constantă pozitivă?Geanina Hăvârneanu, IaşiL174. Fie a 1 , a 2 , . . . , a n , b 1 , b 2 , . . . , b n numere reale pozitive şi a =nXi=1b =nXi=1√bi!2√ai!2. Arătaţi că există x 0 > 0 astfel încât"nXi=1pa i x + b i#−[ √ ax + b] ∈,{0, 1}, ∀x > x 0 .L175. Arătaţi că[ n 2 Xk=0]Cn 2k Ω k = 2 n Ω n , n ∈ N,Marian Tetiva, Bârladunde Ω k =(2k − 1)!!, k ∈ N ∗ (se convine ca Ω 0 = 1).(2k)!!Gheorghe Costovici, IaşiTraining problems for mathematical contestsA. Junior highschool levelG166. Prove that the following assertions are true:a) ∀n ∈ N, n ≥ 2, ∃ x 1 , x 2 , . . . , x n ∈ N ∗ such that x 1 x 2 +x 2 x 3 +x n x 1 = x 1 x 2 . . . x n .b) ∀n ∈ N, n ≥ 5, ∄ x 1 , x 2 , . . . , x n ∈ 2 N ∗ such that x 1 x 2 + x 2 x 3 + x n x 1 =x 1 x 2 . . . x n .c) ∃ x 1 , x 2 , . . . , x n ∈ N ∗ such that x 1 x 2 +x 2 x 3 +x n x 1 = x 1 x 2 . . . x n ⇔ n ∈ 2 N ∗ +1.Dan Popescu, SuceavaG167. Let d 1 < d 2 < . . . d k = n be all the positive divisors of the natural numbern. Assuming that the subscripts i, j with j > i > 13 exist such that d 2 7 + d 2 i = d2 j ,show that n is a multiple of 8.2 Titu Zvonaru, ComăneştiG168. For any x, y, z ∈ R ∗ +, prove that the following inequality holds :x(y + z) y(x + z) z(x + y)+ +x + y z y + x z z + x y ≤ x 2x + y z +y2y + x z +z2z + x y.α 3Ştefan Gavril, Piatra NeamţG169. Prove that an infinity of irrational numbers α exist with the property thatand α 2 + α are irrational numbers as well.Gabi Ghidoveanu and Dumitru Mihalache, Bârlad169


G170. A subset A ⊂ R, of cardinal number 2009, has the property that eachof its elements is greater than one tenth of the sum of the remaining 2008 numbers.Show that A contains at least 12 negative numbers.Andrei Nedelcu, IaşiG171. The points in the plane whose both coordinates, yin a Cartesian system of coordinates, are natural numbers,visited along the path indicated by the arrows in the figure atright, starting from the origin. We denote by a n,k the positionof the point of coordinates (n, k) in the sequence thus obtained(for instance, a 0,0 = 1, a 0,2 = 4, a 2,2 = 13, etc.). Express thenumber a n,k in terms of n and k.x0Lucian Georges Lăduncă, IaşiG172. A rectangular board of size m × n, m, n ≥ 2, has unit squares at theintersections of odd-order rows with odd-order columns, colored in black while theother squares remain white. To re-color a row (column) means to change the colorof all the squares on that row (column). Show that the board cannot be turned to acompletely white board by recoloring a couple of rows and columns.Răzvan Ceucă, school student, IaşiG173. We denote by T (a, b, c) the triangle whose side lengths are a, b, c. Provethat the triangles T (b, 2c, 2m b ) and T (c, 2b, 2m c ) can be successively manufacturedfrom a single (the same) cardboard sheet without losses of material.Petru Asaftei, IaşiG174. The isosceles triangle ABC is considered with m(bA) = 40 0 . Show thatno points P ∈Int(ABC) exist such that m(ÕP AB) = 30 0 , m(ÕP BC) = 10 0 andm(ÕP CA) = 35 0 .Gabriel Popa and Paul Georgescu, IaşiG175. Let ABCD be a quadrilateral inscribed in the circle of radius R. Provethat AB · AD + CB · CD ≤ 4R 2 .Gheorghe Costovici, IaşiB. Highschool levelL166. Let ABCD be a rectangle, and C a circle passing through A whichintersects the lines (AB), (AC) and (AD) at the points M, N and P , respectively.Show that AM · AB + AP · AD = AN · AC.Gheorghe Iurea, IaşiL167. Let ABC be a triangle with AB > AC. The circle inscribed in the triangleis tangent to the sides BC and AC at D and respectively E. We consider a point Ton the side [BC] and denote by J the center of the circle inscribed in △ABT . IfDE passes through the midpoint of the segment [CJ], prove that the triangle AT Cis isosceles.Titu Zvonaru, Comăneşti170


"nXi=npa i x + b i#−”√ax + b—∈{0, 1} , for ∀x > x 0 .L168. Prove that in any triangle, with the usual notations, the following inequalityholds :ab + c + bc + a + ca + b ≥ 11 p2 − 15 r 2 − 60 R r6 p 2 − 6 r 2 ≥ 3 − 24 R r 2 .Marius Olteanu, Rm. VâlceaL169. What is the probability that the radii of the excircles to a randomly chosentriangle be the sides of a new triangle?Petru Minuţ, IaşiPL170. Let n ∈ N, n ≥ 2 and a 1 , a 2 , . . . , a n ∈ R + with a 1 + a 2 + · · · + a n = S.We consider k ∈ N, 1 ≤ k ≤ n − 1 and α 1 , α 2 ∈ R with α 1 + α 2 = 1. Prove theinequality(a i1 + a i2 + · · · + a ik ) α 1(S − a i1 − · · · − a ik ) α 2≤1≤ i 1 < i 2 0 exists such thatL175. Show that[ n 2 ]Cn Xk=02k Ω k = 2 n Ω nMarian Tetiva, Bârladwhere Ω k =(2k − 1)!!, k ∈ N ∗ ; by convention Ω 0 = 1.(2k)!!Gheorghe Costovici, Iaşi171


Pagina rezolvitorilorCOVASNAŞcoala cu clasele I-VIII ”Avram Iancu”. Clasa a V-a (prof. LÁZÁR Emese).BÂRLĂ Oana: P(161,163), V(95,96), VI.96; PĂTRÂNJEL Andrada-Maria: P(161,163), V(95,96), VI.96; TIMARU Carmen-Ioana: P(161,163), V(95,96), VI.96; UŢĂIoana: P(161, 163), V(95,96), VI.96; VLĂDEA Diana: P(161,163), V(95,96), VI.96.CRAIOVALiceul Teoretic ”Tudor Arghezi”. Clasa a V-a (prof. DRĂCEA Dorina). VÎRLANLeonard: P.161, V(95-97).IAŞIŞcoala nr. 3 ”Al. Vlahuţă”. Clasa I (inst. MAXIM Gabriela). CUCURUZRaluca: P(154,155, 157, 164-167); DASCĂLU Lorena: P(154,155,157,164-167); PO-PESCU Alexandru: P(154,157,164-167); ROBU Carmen: P(154,157,164-167); ŞER-BĂNOIU Alexandru: P(154,157,164-167); TORAC George: P(154,157,164-167);NICA Daniel: P(154,155,157,164-167). Clasa a II-a (inst. CRĂCIUN Marilena).CREANGĂ Adrian: P(154,155,157,164-167); FILIP Vlad: P(154,155,157,164-167);POPESCU Claudia: P(154,155,157,164-167). Clasa a II-a (inv. MĂRIUŢĂ Valentina).ENEA Codruţ Alexandru: P(154, 155, 157, 164-167); GHEORGHIU Beatrice-Elena: P(154,155,157,164-167); HERGHELEGIU Mădălina: P(154,155,157,164-167);HREAPCĂ Alin: P(154,155,157,164-167); HUHU Paula: P(154,155,157,164-167);POPOVICIU Teodor-Andrei: P(154,155,157,164-167); ROMILĂ Andreea-Maria:P(154,155,157,164-167). Clasa a VI-a (prof. MARIN Mirela). MARCU Ana:V(105,106), VI(102,104,105); RUSU Mădălina-Andreea: V(105,106), VI(102,104,105);TIBA Ştefana-Alexandra: V(105,106), VI(102,104,105).Şcoala nr. 13 ”Alexandru cel Bun”. Clasa I (inst. COJOCARIU Ana). ACA-TRINEI Andra: P(154,164-167); PERDUN Patricia-Maria: P(154,164-167); PRISE-CARU Alexandru-Julian: P(154,164-167), SAMSON Constantin-Cătălin: P(154,164-167); ŞTEFAN Tudor: P(154,164-167); ZAHARIA Ştefan-Eusebiu: P(164,164-167).Şcoala nr. 14 ”Gh. Mârzescu”. Clasa a IV-a (inst. NUŢĂ Elena). BACIU Tudor:P(164-167,170); CHIRILUŢĂ George-Ştefan: P(164-167,169-171); NEDELEUIulia: P(164-167,170); POSTUDOR Georgiana-Mădălina: P(164-167,170); ŢÂMBA-LARIU Ioana-Vasilica: P(164-167,170).Şcoala nr. 22 ”B.P. Haşedeu”. Clasa I (inv. BLĂJUŢ Cristina). SAVA CosminaIoana: P(164-167,169). Clasa a IV-a (inv. ŞTEFAN Liviu). BLĂJUŢ Cristin-Marian: P(167-168).Şcoala nr. 26 ”George Coşbuc”. Clasa a II-a (inst. VÂRLAN Elena). AMARIEIRomeo: P(154-157,164-167); GHEBAN Andreea: P(154-157,164-167); PĂVĂLUCAna-Maria: P(154-157,164-167); PICHIU Cosmin: P(154-157,164-167); TĂTARUAlice: P(154,157,164-167); ŢIPLEA Iulian: P(154-157,164-167). Clasa a III-a(inv. BUCATARIU Rica). BARHAN Ştefana-Adina: P(164-166,168,169,171,172);CHIRIAC Alexandra: P(164-166,168,169,171,172) CUPEŢ Valeria: P(164-166,168,169,171,172); FRUNZĂ Andrei-David: P(164-166,168,169,171,172); FRUNZĂ Diana-Mihaela: P(164-169,171,172); IVANOV Alexandra: P(164-166,168,169,171,172);172


MÎNDRU Liana: P(164-169,171,172). Clasa a IV-a (inst. RACU Maria). APA-CHIŢEI Aura-Georgiana: P(164-171); BURA Emma-Andreea: P(164-170); CRĂ-CIUN Ioana-Daniela: P(164-171); LEŞOVSCHI Alexandra-Ioana: P(164-169); LUPURoxana-Elena: P(164-169).Şcoala SAM ”M. Kogălniceanu”, Ţigănaşi. Clasa a II-a (înv. GALIA Paraschiva).CAZADOI Ioana-Cristina: P(154,155,157,164-167); DUCA Cristina Mihaela: P(154-155,157,164-167); SANDU Rebeca: P(154-155,157,164-167).SFÂNTU GHEORGHE (Tulcea)Şcoala generakă cu clasele I-VIII. Clasa a II-a (înv. GAVRILĂ Elena). HALCHINIoana: P(154-158); SIDORENCU Adrian: P(154-158). Clasa a IV-a (înv. GAVRILĂElena). BĂLAN Silviu: P(154-161); CLADIADE Bogdan-Robert: P(154-161); CUCUDelia: P(154-161); EFIMOV Cosmin Alexandru: P(154-161). Clasa a VI-a (prof.SĂILEANU Sorin). SIDORENCU Andrei: V(95,96), VI.96, VII(96,98), VIII.96.Şcoala nr. 26 ”G. Coşbuc”, IaşiElevi rezolvitori premiaţi1. LEŞOVSCHI Alexandra-Ioana (cl. a IV-a): 2/2008(6pb), 1/2009(6pb), 2/2009(6pb).2. LUPU Roxana-Elena (cl. a IV-a): 2/2008(6pb), 1/2009(6pb), 2/2009(6pb).Vizitaţi noua pagina web a revistei:http://www.recreatiimatematice.ro173


IMPORTANT• În scopul unei legături rapide cu redacţia revistei, pot fi utilizate următoareleadrese e-mail: t birsan@yahoo.com şi profgpopa@yahoo.co.uk . Peaceastă cale colaboratorii pot purta cu redacţia un dialog privitor la materialeletrimise acesteia, procurarea numerelor revistei etc. Sugerăm colaboratorilorcare trimit probleme originale pentru publicare să le numerotezeşi să-şi reţină o copie xerox a lor pentru a putea purta cu uşurinţă o discuţieprin e-mail asupra acceptării/neacceptării acestora de către redacţia revistei.• La problemele de tip L se primesc soluţii de la orice iubitor de matematicielementare (indiferent de preocupare profesională sau vârstă). Fiecare dintresoluţiile acestor probleme - ce sunt publicate în revistă după un an - va fiurmată de numele tuturor celor care au rezolvat-o.• Adresăm cu insistenţă rugămintea ca materialele trimise revisteisă nu fie (să nu fi fost) trimise şi altor publicaţii.• Rugăm ca materialele tehnoredactate să fie trimise pe adresa redacţieiînsoţite de fişierele lor (de preferinţă în L A TEX).• Pentru a facilita comunicarea redacţiei cu colaboratorii ei, autorii materialelorsunt rugaţi să indice adresa e-mail.174

Hooray! Your file is uploaded and ready to be published.

Saved successfully!

Ooh no, something went wrong!